Chuyền đề Số học

120 5 0
Chuyền đề Số học

Đang tải... (xem toàn văn)

Tài liệu hạn chế xem trước, để xem đầy đủ mời bạn chọn Tải xuống

Thông tin tài liệu

Phần nguyên là một lĩnh vực hay và độc đáo của toán sơ cấp, cao cấp và ứng dụng.Có nhiều bài toán hay về phần nguyên đã được sử dụng làm đề thi học sinh giỏi các cấp, trong đó có rất nhi[r]

(1)

CHUYÊN ĐỀ SỐ HỌC

(2)(3)

Lời nói đầu

1 Bước nhảy Viete Mở đầu

Lời giải nguyên thủy toán vấn đề liên quan

Gợi ý cho số toán 14

Các toán thử sức 15

2 Vận dụng phương pháp LTE vào giải toán số học 17 Một số khái niệm 17

Hai bổ đề 18

Lifting The Exponent Lemma (LTE) 18

Một số ví dụ 21

Bài tập vận dụng 27

3 Các toán số học hoán vị vịng quanh 29 Phương pháp đối xứng hóa 29

Phương pháp dùng bất đẳng thức 32

Bài tập tự luyện 36

4 Dãy số số học 39 Dãy số nguyên tính chất số học 39

Dãy số nguyên tính phương 47

5 Một số hàm số học ứng dụng 61 Hàm tổng ước số số ước số 61

Kiến thức cần nhớ 61

Ví dụ áp dụng 62

Bài tập có hướng dẫn, gợi ý 65

Bài tập tự giải 66

Một số hàm số khác 68

Hàm phần nguyên 68

(4)

Hàm Euler 69

Bài tập tổng hợp 70

6 Thặng dư bình phương 73 Tính chất thặng dư bình phương kí hiệu Legendre 73

Bài tập ví dụ 76

Kí hiệu Jakobil 79

Bài tập ví dụ 80

Khai thác bổ đề 81

Bài tập đề nghị 83

7 Cấp nguyên thủy 85 Cấp số nguyên dương 85

Căn nguyên thủy 97

8 Hàm phần nguyên phần lẻ 101 Định nghĩa, tính chất tập 101

Định nghĩa 101

Các tính chất quen thuộc 101

Bài tập 103

Ứng dụng định lý Hermite định lý Legendre 105

Hàm có chứa phần nguyên 109

Hàm phần nguyên việc tính tổng chữ số 115

Định nghĩa 115

Tính chất 115

Bài tập ví dụ 116

(5)

“Số học bà hoàng Tốn học”

Có thể nói Số học lĩnh vực xuất sớm lịch sử Toán học Khi người bắt đầu làm việc với số ấy, Số học đời.Trải qua hàng nghìn năm phát triển, Số học giữ vẻ đẹp khiết Vẻ đẹp thể qua cách phát biểu đơn giản toán, học sinh lớp hiểu Thế nhưng, vẻ đẹp thường tiềm ẩn thử thách sâu thẳm bên để thách thức trí tuệ lồi người

Hãy nói Định lý Fermat lớn, định lý tiếng giới Toán học Trong tuyển tập văn học với tựa đề Thoả ước với Quỷ có truyện ngắn Con Quỷ Simon Flagg Arthur Poges Trong truyện ngắn Quỷ có đề nghị Simon Flagg đặt cho câu hỏi Nếu Quỷ trả lời vịng 24 giờ, lấy linh hồn Simon, cịn đầu hàng trả cho Simon 100.000 đơla Simon đặt cho Quỷ câu hỏi: “ Định lý cuối Fermat có khơng?” Nghe xong, Quỷ biến bay vút khắp vũ trụ để tiếp thu tất tri thức toán học sáng tạo Ngày hôm sau Quỷ quay trở lại thú nhận: “Simon, thắng”, Quỷ buồn rầu nói nhìn Si mon với mắt đầy thán phục “Ngay ta, ta khơng có đủ kiến thức toán học để thời gian ngắn giải đáp tốn khó Càng nghiên cứu sâu rắc rối Chà! Ngươi có biết”-Con Quỷ tâm sự- “ngay nhà toán học giỏi hành tinh khác, họ uyên bác nhà tốn học nhiều, khơng giải câu đố khơng? Thì đấy, gã Thổ nhìn giống nấm cà kheo, gã giải nhẩm phương trình vi phân đạo hàm riêng, mà phải đầu hàng thơi.”

Chính có cách phát biểu đơn giản cần suy luận sâu sắc tinh tế nên tốn Số học kì thi Olympic thường dùng để phân loại học sinh Tuy thị trường có nhiều sách viết Số học, nhu cầu sách lĩnh vực chưa vơi Đặc biệt, ngày nhiều phương pháp xuất hiện, cần đầu sách phát triển phương pháp cũ mà cịn giới thiệu phương pháp nhìn vấn đề cũ

(6)

Thặng dư phương, Phương pháp bước nhảy Viete Hi vọng tài liệu hữu ích cho bạn đọc gần xa việc ơn luyện cho kì thi Olympic

Thành phố Hồ Chí Minh, ngày Phụ nữ Việt Nam 20-10-2013 Tuy kiểm tra kĩ chuyên đề khơng tránh khỏi sai sót Mọi góp ý chuyên đề xin gửi lên Diễn đàn thuvientoan.net gửi hộp mail thuvientoan.net@gmail.com Xin chân thành cảm ơn

(7)

Bước nhảy Viete

Phạm Huy Hồng

Mở đầu

Trong kì thi học sinh giỏi, tốn phương trình Diophante bậc hai khơng cịn xa lạ Phương trình Pell ví dụ bật phương trình Diophante bậc hai, nhiên lượng tốn phương trình Pell nhiều, nên kì thi IMO 1988 xuất dạng phương trình Diophante bậc hai mẻ thời giờ:

Cho a, b hai số nguyên dương thỏa mãnab+ 1|a2+b2 Chứng minh

a2 +b2

ab+

là số phương

Bài tốn coi khó kì thi IMO trước năm 1988 toán khó kì thi Tác giả Authur Engel bình luận tốn (ngun văn):

" Nobody of the six members of the Australian problem committee could solve it Two of the members were George Szekeres and his wife, both famous problem solvers and problem creators Since it was a number theoretic problem it was sent to the four most renowned Australian number theorists They were asked to work on it for six hours None of them could solve it in this time The problem committee submitted it to the jury of the XXIX IMO marked with a double asterisk, which meant a superhard problem, possibly too hard to pose After a long discussion, the jury finally had the courage to choose it as the last problem of the competition Eleven students gave perfect solutions."

Dịch tiếng Việt nôm na là:

"Khơng có số sáu thành viên hội đồng giám khảo Úc giải toán Hai thành viên bật đó, người tiếng giải sáng tạo toán, George Szekeres vợ ơng Đây tốn số học nên gửi cho bốn nhà số học lớn Úc Họ u cầu giải tốn sáu khơng có giải sau Hội đồng thẩm định nộp cho

1Đại học Khoa học Tự Nhiên

(8)

ban giám khảo IMO XXIX toán với hai dấu hoa thị, để nói lên khó, khó để kì thi Sau hồi bàn bạc, hội đồng IMO XXIX định chọn toán làm cuối kì thi Có mười học sinh cho lời giải hồn chỉnh tốn."

Trong số 11 học sinh có Giáo sư Ngơ Bảo Châu

Lời giải nguyên thủy toán vấn đề liên quan

Chúng ta bắt đầu với toán gốc:

Bài toán Cho a, b số nguyên dương thỏa mãnab+ 1|a2 +b2 Chứng minh

a2 +b2

ab+

là số phương Lời giải Đặtk = a2+b2

ab+1 Cố địnhk xét tất cặp (a, b)nguyên dương thỏa mãn phương

trình

k = a

2+b2

ab+ 1,

có nghĩa xét tập

S =

(a, b)∈N∗×N∗|k = a

2+b2

ab+

Vì S tập cặp số nguyên dương nên tồn cặp(a0, b0)trong S mà a0+b0 đạt

giá trị nhỏ a0 >b0

Xét phương trình

x2+b20 xb0+

=k⇔x2−kx.b0+b20−k =

là phương trình bậc hai ẩnx Ta biết phương trình có nghiệm làa0 Như

vậy theo định lý Viete tồn nghiệm a1 thỏa mãn phương trình bậc hai với ẩnx

a1 =kb0−a0 =

b2 −k

a0

Từ ta có a1 số nguyên Ta chứng minh a1 không âm Thật vậy, nếua1 <0

a21−kb0a1+b20−k>a21+k+b20−k >0,

mâu thuẫn Do ta cóa1 >0 Từ ta có:

Nếu a1 >0thì (a1, b0) cặp thuộcS Theo định nghĩa (a0, b0) ta có:

a0+b0 6a1+b0 ⇒a0 6a1

Do theo Viete thì:

a20 6a0a1 =b02−k < b20 ⇒a0 < b0,

trái với giả thiết ban đầu

(9)

Từ tốn ta thấy bước giải toán dùng phương pháp sau: Nhận dạng tốn thuộc lớp phương trình Diophante bậc hai (trở lên)

2 Cố định giá trị nguyên mà đề cho, giả sử tồn cặp nghiệm thỏa mãn vài điều kiện mà khơng làm tính tổng qt tốn

3 Dựa vào định lý Viete để tìm mối quan hệ mâu thuẫn, từ tìm kết luận tốn

Điểm mấu chốt tốn làngun lí cực hạn: Trong tập hợp số ngun dương ln tồn số nguyên dương nhỏ Mệnh đề hữu dụng lớp toán mà cịn nhiều tốn tổ hợp, tổ hợp số học số học

Từ toán tiếp theo, tơi trình bày vắn tắt bước làm cách làm thay trình bày đầy đủ tốn trên, để bạn tự phát huy tính tự làm việc Phần gợi ý có cuối viết

Bài tốn Tìm tất số nguyên dương n cho phương trình sau có nghiệm ngun dương :

x2+y2 =n(x+ 1)(y+ 1)

Chứng minh Chúng ta làm theo bước toán trên:

1 Cố địnhn, giả sử tồn cặp (x0, y0) mà tổng x0+y0min x0 >y0

2 Xét phương trình bậc ẩnX sau:

X2−X.n(y+ 1) +y02−ny0−n=

Phương trình có nghiệm x0 nên có nghiệm x1

3 Áp dụng định lý Viete:

x0 +x1 =n(y0+ 1), x0x1 =y02−ny0−n

4 Tương tự trước, bạn chứng minh x1 > từ chứng minh x1 =

cách chứng minhx1 >0 dẫn đến mâu thuẫn

5 Từ đến kết luận tốn:x1 = vày20 =n(y0+ 1) suy y0+ 1|y20, điều không

thể xảy y0 nguyên dương

Do khơng tồn số ngun dương n thỏa mãn phương trình r Từ tốn này, ta dẫn đến toán thú vị sau:

Bài toán Giả sử a, b nguyên dương thỏa mãn:

b+ 1|a2+ 1, a+ 1|b2 +

(10)

Chứng minh Nhìn vào tốn trên, từ giả thiết ta khơng nhìn thấy mối tương quan giữaa, b

và tính chẵn lẻ hai số Vì vậy, nhờ kinh nghiệm, cách làm tốt để thêm kiện sử dụng phương pháp phản chứng

Giả sử a, bđều số chẵn Từ giả sử này, bạn đọc chứng minh hai mấu chốt sau: a+ b+ nguyên tố

2 a+ |a2+b2, b+ 1 |a2+b2.

Từ suy tồn n nguyên dương cho a2+b2 = n(a+ 1)(b+ 1) và theo tốn trên

ta có điều mâu thuẫn Vì a, blẻ r

Bài tốn bổ đề quan trọng tốn IMO Shortlist 2009 Bài tốn Tìm tất số nguyên dươngnsao cho tồn dãy số nguyên dươnga1, a2, , an thỏa mãn

ak+1 =

a2

k+

ak−1+

−1

với mọik thỏa mãn 26k6n−1 (Phần gợi ý có cuối viết)

Khi giải hai tốn đa số tốn với hai biếnx, y thành chuyện "đơn giản" Xin mời bạn đọc thử sức với toán sau, điều ẩn sau điều thú vị:

Bài tốn Tìm tất số n ngun dương cho phương trình sau có nghiệm ngun dương:

(x+y)2 =n(4xy+ 1)

Chứng minh Chúng ta theo bước Đáp án với n số phương

phương trình ln có nghiệm nguyên dương r

Đáng ý toán đơn giản mà lại bổ đề quan trọng cho tốn khó sau:

Bài tốn (Taiwan MO 1998) Cho m, nlà hai số lẻ với m > n >1thỏa mãn

m2−n2+ |n2−1

Chứng minh rằngm2−n2+ số phương

Chứng minh Nhìn vào tốn tốn trên, khơng thể thấy liên hệ Gợi ý cho toán làm để chuyển toán trước

Từ giả thiết ta có m2−n2+ 1|n2−1, tiện gọn hơn, ta cóm2−n2+ 1|m2 Từ đây

ta đặtm2 =k.(m2−n2+ 1)với k nguyên dương Đến khơng khó để nhìn ra

mối liên hệ: Từ giả thiết m, nlẻ, tồn hai số nguyên dương a, bsao chom=a+b, n=a−b Do phương trình trở thành:

(a+b)2 =k(4ab+ 1),

chúng ta quay toán Vậy k số phương, 4ab+ số

(11)

Nhìn tốn trên, người làm tốn, khơng khỏi thắc mắc là: liệu có tồn hai số m, nnhư không để thỏa mãn m2−n2+ 1|n2−1để suy ram2−n2+ 1 là số chính

phương? Bằng lối suy nghĩ nên tìm thử nghiệm tốn:

Bài tốn Tìm cặp nghiệm (m, n)lẻ nguyên dương thỏa mãn điều kiện toán Chứng minh Thử vài giá trị, tốn khơng dễ bạn tưởng: Chúng ta khơng thể "mị" nghiệm để suy Chúng ta nên bắt đầu với cách làm tự nhiên nhất: đặt

m2 =k(m2 −n2+ 1) Vì tốn u cầu nghiệm, ta bắt đầu với k = 1 là số chính

phương đầu tiên:

m2 =m2−n2+ ⇔n= 1,

khơng thỏa mãn m > n >1

Tiếp tục với k= số phương tiếp theo:

m2 = 4(m2−n2+ 1) ⇔4n2−3m2 =

Từ phương trình suy 2|m hay đặtm = 2t Từ phương trình tương đương với

n2 −3t2 = 1,

trở phương trình Pell quen thuộc phương trình chắn có nghiệm khơng phải số phương Tìm nghiệm phương trình khơng khó, bạn tự tìm cách sử dụng cơng thức nghiệm tổng quát phương trình Pell hệ phương

trình r

Vậy tất tốn tìm nghiệm thỏa mãn Bài tốn vừa xong nghiệm đơn giản Câu hỏi tìm nghiệm tổng qt khơng? Câu trả lời có Trong kì thi chọn học sinh giỏi Toán quốc gia năm 2012, toán sử dụng phương pháp bước nhảy Viete để giải tốn Xin trích dẫn đề bài, giải tài liệu "Nhận xét đánh giá đề thi VMO 2012" Thầy Trần Nam Dũng:

Bài toán Xét số tự nhiên lẻ a, b mà a ước số b2+ 2 và b là ước số của a2+ 2.

Chứng minh rằnga vàb số hạng dãy số tự nhiên (vn) xác định

v1 =v2 = 1;vn = 4vn−1−vn−2,∀n >2

Chứng minh Giả sử (a, b) cặp số tự nhiên lẻ mà a ước số b2 + b ước số

a2 + Trước hết ta chứng minh (a, b) = Thật vậy, đặt d = (a, b) d | a | b2+ nên

d|2 Màa, b lẻ nênd lẻ, suy d=

Xét sốN =a2+b2+ 2thì a2+ 2chia hết chob nên N chia hết chob Tương tự,N chia hết cho a Vì (a, b) = nên từ suy N chia hết cho ab Vậy tồn số nguyên dương k cho

a2+b2+ = kab (1)

Tiếp theo, ta chứng minh k= Thật vậy, đặtA ={a+b|(a, b)∈N∗×

N∗, a2+b2+ = kab}

(12)

a0, b0 cặp số thỏa mãn điều kiện (1) vớia0+b0 nhỏ

Khơng tính tổng qt, giả sử a0 > b0 Xét phương trình a2−kb0a+b20+ = có

nghiệma0 Theo định lý Viete phương trình cịn có nghiệm làa1 =kb0−a0 =

b2 0+2

2

Theo cơng thức nghiệm rõ rànga1 ngun dương Như vậy(a1, b0)cũng nghiệm

(1) Do tính nhỏ a0+b0, ta có a0+b0 6a1+b0, tức làa0 6kb0−a0 suy ab00 k2

Ta có a2

0+b20+ =ka0b0 suy

a0

b0

+ b0

a0

+

a0b0

=k (2)

Do a0 b0

k

2 nên từ (2) ta có k6

k

2 + + hay k66

Mặt khác, áp dụng bất đẳng thức AM-GM ta có a2

0+b20 >2a0b0 nên k >2

Nếu k 6= (a0, b0)6= (1,1), a0b0 > Dùng (2) để đánh giá ta có k k2 + + nên

k 64 Vậy giá trị k= 5,6bị loại Nếu k = 3thì doa2

0+b20+ = 3a0b0 nên suy raa20+b20+

chia hết cho 3, suy hai số a0, b0 chia hết cho 3, số cịn lại khơng chia hết cho

Nếu b0 = 1thì a0 chia hết cho 3, vế trái khơng chia hết cho vế phải chia hết cho

9, mâu thuẫn Vậy b0 >1 Từ suy raa0b0 >6 Lại sử dụng (2) để đánh giá, ta suy

k6 k

2 + +

2

6 ⇒k <

Mà k∈N nên k 62, mâu thuẫn

Như ta chứng minh a, blà số tự nhiên lẻ thỏa mãn điều kiện đề

a2+b2+ = 4ab (3)

Ta chứng minh trường hợp tồn số nguyên dương n cho (a, b) = (vn, vn+1) với dãy số định nghĩa đề

Trước hết, ta có nhận xét : Nếu a, b nghiệm (3) (4a−b, a) (4b−a, b) nghiệm (3) Từ đó, (v1, v2)là nghiệm (3) nên(4v2−v1, v2) nghiệm (3),

tức (v2, v3) nghiệm (3) Từ quy nạp suy (vn, vn+1) nghiệm

(3)

Giả sử tồn cặp số (a, b) thỏa mãn (3) không tồn n cho (a, b) = (vn, vn+1)

Trong cặp số thế, chọn (a, b) có tổng a+b nhỏ Khơng tính tổng qt, giả sử a > b (chú ý a b a = b suy a = b = 1, (a, b) = (v1, v2)

Theo nhận xét 4b−a, bcũng nghiệm (3) Nhưng do4b−a= b2a+2 < a(Vì a > b

nên ab−b2 = (a+b)(a−b) >3)), nên 4b−a+b < a+b Theo định nghĩa của (a, b) ở trên,

phải tồn n cho (4b−a, b) = (vn, vn+1) Sử dụng đẳng thức 4b−a = b 2+2

a b > 1, ta suy 4b−a 6 b Như 4b−a = vn, b = vn+1 Nhưng từ a = 4vn+1 −vn = vn+2, tức

là (a, b) = (vn+1, vn+2) mâu thuẫn Vậy điều giả sử sai, tức phải tồn số tự nhiên n

cho (a, b) = (vn, vn+1) a, b số hạng dãy (vn) Bài tốn giải hồn

tồn r

Nhận xét:

(13)

1 Chứng minh k số nguyên dương cho tồn a, b nguyên dương thỏa mãn điều kiện a2 +b2+ = kab thì k = 4 Phần quen thuộc với bạn

biết phương pháp phương trình Markov hay “bước nhảy Viete”

2 Mơ tả tất nghiệm phương trình a2+b2+ = 4ab thông qua cặp số hạng

liên tiếp dãyvn, gọi phương pháp gien

• Phương trình (3) cịn giải thơng qua phương trình Pell z2−3b2 =−2 Tuy nhiên cách giải cồng kềnh khơng phải phương trình Pell loại

Như qua nhận xét toán trên, thầy Trần Nam Dũng tổng kết lại bước làm chính: Đó sử dụng bước nhảy Viete để tìm k sử dụng phương pháp Gien, phương pháp lùi vơ hạn để tìm điểm đặc biệt nghiệm, nếu(a, b)là nghiệm thì(4b−a, b)

cũng nghiệm, dẫn đến tìm nghiệm tổng qt phương trình

Ngồi thầy đề cập tới phương trình Markov, phương trình tiếng, nói "thủy tổ" phương pháp bước nhảy Viete:

Bài tốn (Phương trình Markov) Tìm tất số ngun dương k cho phương trình sau có nghiệm nguyên dương:

x2+y2+z2 =kxyz

Chứng minh Chúng ta vừa giải xong lớp toán với hai ẩn, tốn ba ẩn lời giải có khác khơng? Và khác nào? Liệu ta tìm cách giải tổng qt cho tốn với ba ẩn với hai ẩn khơng?

Cách giải toán phụ thuộc vào cách giải toán hai số, trước hết ta cần bổ đề sau:

Bổ đề k = số nguyên dương để phương trình sau ln có nghiệm ngun dương (x, y):

x2+y2+ =kxy

Bổ đề toán sử dụng bước nhảy Viete quen thuộc, mời bạn đọc tự giải Trở lại toán:

1 Thấy k = phương trình có nghiệm (3,3,3) k = phương trình có nghiệm

(1,1,1)

2 Xét k 6= 1,3 Giả sử phương trình có nghiệm (x0, y0, z0) Khơng tính tổng qt, giả

sử x0 y0 z0 x0+y0 +z0 nhỏ tất tổng x+y+z với x, y, z

nghiệm phương trình

• Nếuy0 < z0, xét phương trình bậc hai:

Z2−k.x0y0.Z+x20+y =

Phương trình có nghiệm z0 Theo định lý Viete có nghiệm thứ hai

z1 thỏa mãn:

z1 =kx0y0−z0 =

x2 0+y02

z0

(14)

Từ suy raz1 nguyên dương và(x0, y0, z1)là nghiệm thứ hai Do theo giả thiết

cực hạn ta có:

x0+y0+z0 6x0+y0+z1 ⇒z0 6z1,

dẫn đến

x20+y02−kx0y0 =z1z0−z1−z0 = (z1−1)(z0−1)−1>y02−1,

và suy

1>x0(ky0−x0)>x0(kx0−x0)>x0,

mà x0 nguyên dương suy x0 = Ta trở tốn y2+z2+ = kyz,

bổ đề suy rak = 3, trái giả thiết

• Nếuy0 =z0 Ta có

2y20 −py02+x20 = ⇒x20 =y02(px0−2)>x20(px0 −2),

và từ dẫn đến 3>px0, mà px0 >2nên px0 = suy p∈ {1,3}, trái giả thiết

Vậy k ∈ {1,3} phương trình ln có nghiệm ngun dương r Trên số tốn mà tơi muốn giới thiệu với bạn đọc, tốn quen thuộc lặp lại nhiều kì thi Tơi hi vọng qua viết bạn đọc nắm bắt phương pháp giải lớp toán phương trình bậc hai Diophante nhiều ẩn

Sau gợi ý cho toán số toán thử sức

Gợi ý cho số toán

2 Chứng minhx1 >0:

x21−x1n(y0+ 1)−n(y0+ 1) +y02 = ⇔x1 =

x2 0+y02

n(y0+ 1)

−1>−1,

mà x1 nguyên nên x1 >0

3 Vớia, bchẵn: có b+ 1|b2−1, b+ 1|a2+ 1 nên b+ 1|a2+b2 Tương tự a+ 1|a2+b2.

Gọi d= (a+ 1, b+ 1, chứng minh d|2mà d lẻ doa+ 1, b+ 1lẻ nên d= 1, từ suy raa2+b2 =k(a+ 1)(b+ 1).

4 n= 1,2,3,4 Dễ dàng dãy vớin = 1,2,3 Dãy có độ dài 4: 4,33,27,1384

Phản chứng tồn dãy độ dài 5: a1, a2, a3, a4, a5 Từ chứng minh hai mệnh đề sau:

(a) a2, a3 chẵn (sử dụng phản chứng)

(b) a2+ 1|a23 + 1, a3+ |a22+

5 Chứng minhx1 >0: từ phương trình ta có

4x1y0 =

(x1 +y0)2

n −1>−1⇔x1 >−

1 4y0

(15)

Các toán thử sức

Bài toán 10 Chứng minh a, b số nguyên dương cho k = a2+abb2+6 số ngun k =

Bài tốn 11 Tìmn cho phương trình sau có nghệm ngun dương:

(x+y+z)2 =nxyz

2 Tìmn cho phương trình sau có nghệm nguyên dương:

(x+y+z+t)2 =nxyzt

Bài tốn 12 (Mở rộng phương trình Markov) Cho a, b, c ba số nguyên thỏa mãn

a2+b2+c2 =kabc

Chứng minh hoặc(a, b, c) = (a, b, c) = Bài toán 13 Chứng minh phương trình

x2+y2+z2 =n(xyz+ 1)

có nghiệm nguyên dương n biểu diễn dạng tổng hai số phương

Bài tốn 14 (Adapted from Vietnam TST 1992) Tìm tất cặp số nguyên dương (a, b)

thỏa mãn

a2+b2 =k(ab−1)

Bài toán 15 (Turkey TST 1994) Tìm tất cặp (a, b) mà ab|a2+b2+ 3.

Bài toán 16 Cho a, b, c ba số nguyên dương thỏa mãn

0< a2+b2−abc6c,

chứng minh a2+b2 −abc là số phương.

Bài tốn 17 Chứng minh tồn vơ hạn cặp (m, n) nguyên dương thỏa mãn

m+

n +

n+

m =

Bài tốn 18 (IMO Shortlist 2003) Tìm tất cặp a, b thỏa mãn

a2

2ab2−b3+ 1

Bài toán 19 Chứng minh tất nghiệm nguyên dương phương trìnhx2+y2+1 = 3xy

(16)

Bài toán 20 (IMO 2007, IMO shortlist) Cho a, b nguyên dương Chứng minh

4ab−1|(4a2−1)2, thì a=b.

Gợi ý Dùng phản chứng, giả sử a > b với a, bthỏa mãn Từ giả thiết, chứng minh: 4ab−1|(a−b)2

2 a0−b0 >(a0+b0)(4a0b0−1)với a0, b0 nghiệm nhỏ theo nghĩa a0+b0min, từ

suy mâu thuẫn

r Bài tốn 21 (**, Kiran Kedlaya) Chứng minh (xy + 1)(yz + 1)(zx + 1) số phương xy+ 1, yz+ 1, zx+ số phương

Gợi ý Nếu xy+ 1, yz + 1, zx+ số phương hiển nhiên ta có tích ba số phương

2 Nếu(xy+ 1)(yz+ 1)(zx+ 1)là số phương Hãy chứng minh tồn t thỏa mãn hệ sau:

(x+y−z−t)2 = 4(xy+ 1)(zt+ 1) (x+z−y−t)2 = 4(xz+ 1)(yt+ 1) (x+t−y−z)2 = 4(xt+ 1)(yz+ 1)

(t nghiệm phương trìnht2+x2+y2+z2−2(xy+yz+zt+tx+zx+ty)−

4xyzt−4 = 0) Xét nghiệm t nhỏ

Sử dụng phản chứng: giả sử xy+ khơng phương Từ đây, chứng minh: (a) t> max−{x,y,z1 } >−1nên t >0

(b) Xét hai trường hợp t= t >0, dẫn đến mâu thuẫn

r

Tài liệu tham khảo

1 Đặng Hùng Thắng, Nguyễn Văn Ngọc, Vũ Kim Thủy,Bài giảng số học NXB Giáo dục, 1996

2 Kiran S Kedlaya, When Is (xy+ 1)(yz + 1)(zx+ 1) a Square Mathematics Magazine, Vol 17, No.1, Feb., 1998

3 Authur Engel,Problem Solving Strategies Spinger Verlag, 1998

(17)

Vận dụng phương pháp LTE vào giải toán số học

Phạm Quang Toàn

Bổ đề số mũ (Lifting The Exponent Lemma) bổ đề hữu dụng việc giải toán số học biết đến lịch sử Olympiad Thực chất mở rộng từ bổ đề Hensel Ta thường viết tắt tên bổ đề LTE, tên Tiếng Việt gọi bổ đề số mũ Bài viết xin giới thiệu với bạn đọc bổ đề ứng dụng đặc sắc vào toán lý thuyết số

Bài viết chủ yếu dựa vào tài liệu thành viên Amir Hossein bên trang mathlinks.ro (về mặt lý thuyết giữ nguyên viết Amir Hossein sang viết này) có kèm theo số ví dụ lấy từ kì thi Olympic tốn giới

Một số khái niệm

Ở đây, thay kí hiệu a bnghĩa achia hết cho b, ta kí hiệub|a Vàa6 b thay

b-a

Định nghĩa Choplà số nguyên tố,a số nguyên vàα số tự nhiên Ta cópα là lũy thừa (exact power) a α số mũ (exact exponent) p khai triển α

nếu pα|a và pα+1

-a Khi ta viết pα ka hay vp(a) =α Ví dụ Ta có v5(5400) = hay 53 k5400 5400 = 53·32 ·22

Sau số tính chất Chứng minh tính chất khơng khó, xin dành cho bạn đọc Tính chất Cho a, b, c số nguyên Ta có

1 vp(ab) = vp(a) +vp(b) vp(an) = n·vp(a)

3 min{vp(a), vp(b)}6vp(a+b)

Dấu đẳng thức xảy khivp(a)6=vp(b) vp(gcd(|a|,|b|,|c|)) = min{vp(a), vp(b), vp(c)} vp(lcm(|a|,|b|,|c|)) = max{vp(a), vp(b), vp(c)} Chú ý vp(0) =∞ với số nguyên tố p

1Lớp 9C THCS Đặng Thai Mai, Tp Vinh

(18)

Hai bổ đề

Đầu tiên, xin giới thiệu với bạn đọc hai bổ đề Và hai bổ đề giúp ta tìm cách chứng minh định lí khác LTE

Bổ đề Cho x, y hai số nguyên cho n số nguyên dương Cho số nguyên tố p cho p|x−y p-x, p-y Ta có

vp(xn−yn) =vp(x−y) Chứng minh Ta có p|x−y nên

xn−1+xn−2y+· · ·+xyn−2+yn−1 ≡nxn−1 6≡0 (mod p)

Mà xn − yn = (x − y) (xn−1+xn−2y+· · ·+xyn−2+yn−1) nên ta suy điều phải chứng

minh r

Bổ đề Chox, y hai số nguyên n số nguyên dương lẻ Cho số nguyên tốpbất kì thỏa mãn p|x+y vàp-x, p-y Khi

vp(xn+yn) = xp(x+y)

Chứng minh Áp dụng bổ đề ta cóvp(xn−(−y)n) = vp(x−(−y))nên vp(xn+yn) =vp(x+y)

(vì n lẻ) Bổ đề chứng minh r

Lifting The Exponent Lemma (LTE)

Định lý Cho x y số nguyên (không thiết phải nguyên dương), n số nguyên dương p số nguyên tố lẻ thỏa mãnp|x−y p-x, p-y Ta có

vp(an−bn) = vp(a−b) +vp(n)

Chứng minh Ta chứng minh quy nạp theo vp(n) Trước hết, ta chứng minh khẳng định sau:

vp(xp−yp) =vp(x−y) + Để chứng minh điều ta cần

p|xp−1+xp−2y+· · ·+xyp−2+yp−1 (1)

p2 -xp−1+xp−2y+· · ·+xyp−2+yp−1 (2) Với (1), nhờ áp dụng x≡y (mod p) ta suy

(19)

Với (2), ta đặty =x+kpvới k∈N∗ Khi với 1

6i6p−1 (i∈N)thì

yixp−1−i ≡(x+kp)ixp−1−i

≡xp−1−i

xi+i(kp)xi−1+ i(i−1) (kp)

2xi−2+· · ·

≡xp−1−i xi+i(kp)xi−1

≡xp−1+ikpxp−2 (mod p2)

Do đó,

xp−1+xp−2y+· · ·+yp−1 ≡xp−1 + (xp−1+kpxp−2) + (xp−1 + 2kpxp−2) +· · ·+ (xp−1+ (p−1)kpxp−2)

≡pxp−1+ p−1

2 ·kp

2xp−2

≡pxp−1 6≡0 (mod p2)

Như vp(xp−yp) = vP(x−y) + Quay lại toán, đặt n=pk·h với b, k ∈

N, b >1 gcd(b, p) = Khi

vp(an−bn) = vp((ap k

)h−(bpk)h) =vp

apk−bpk

=vp((ap k−1

)p−(bpk−1)p) =vp(ap

k−1

−bpk−1) + =vp((ap k−2

)p −(bpk−2)p))

=vp(x−y) +k =vp(x−y) +vp(n)

Định lý chứng minh r

Định lý Cho hai số nguyên x, y,n số nguyên dương lẻ, p ước nguyên tố lẻ cho

p|x+y p-x, p-y Khi

vp(xn+yn) = vp(x+y) +vp(n) Chứng minh Áp dụng định lý ta có

vp(xn−(−y)n) =vp(x−(−y)) +vp(n) hay

vp(xn+yn) =vp(x+y) +vp(n)

r Định lý (cho trường hợp p= 2) Cho x, y hai số nguyên lẻ thỏa mãn 4|x−y Khi

v2(xn−yn) = v2(x−y) +v2(n)

Chứng minh Theo bổ đề p nguyên tố, gcd(p, n) = 1, p|x−y p-x, p-y

(20)

Do ta cần xét tới trường hợp n lũy thừa của2, tức cần chứng minh

v2(x2

n

−y2n) =v2(x−y) +n

Thật vậy, ta có

x2n−y2n = (x2n−1 +y2n−1)(x2n−2 +y2n−2)· · ·(x2+y2)(x+y)(x−y)

Vì x≡y≡ ±1 (mod 4) nên x2k ≡y2k ≡1 (mod 4) Do đó

v2(x2

n−1

+y2n−1) =v2(x2

n−2

+y2n−2) =· · ·=v2(x+y) =

Như v2(x2

n

+y2n

) =n+vp(x−y), ta có điều phải chứng minh r Định lý (cho trường hợp p = 2) Cho hai số nguyên lẻ x, y, n số nguyên dương chẵn

2|x−y Khi

v2(xn−yn) =v2(x−y) +v2(x+y) +v2(n)−1

Chứng minh Ta có 4|x2−y2 nên đặtn = 2k·h với k, h∈N∗, gcd(h,2) = 1 Khi ta có

v2(xn−yn) =v2(xh·2

k

−yh·2k) =v2((x2)2

k−1

−(y2)2k−1)

=v2(x2−y2) +k−1

=v2(x−y) +v2(x+y) +v2(n)−1

r Ta có hệ sau:

Hệ Cho a, nlà hai số nguyên dương:

i) p hai số nguyên tố lẻ cho vp(a−1) = α ∈ N∗, với số tự nhiên β ta có

v(an−1) =α+β ⇔vp(n) =β

ii) n chẵn cho v2(a2 −1) =α ∈ N∗, với số ngun dương β v2(an−1) =

α+β ⇔v2(n) =β+

Chú ý

a) Nếu tốn địi hỏi vận dụng phương pháp LTE, ta nên để ý tới điều kiện đặt n, x, y, lựa chọn định lý phù hợp đưa vào lời giải toán

b) Nếu liệu toán cho a|b với a, b∈ N với p ước nguyên tố b, ta ln có

vp(b)>vp(a) Ngược lại, vp(b)>vp(a) a|b Như

a|b ⇔vp(b)>vp(a)

(21)

Một số ví dụ

Sau xin đưa số ví dụ ứng dụng phương pháp Ví dụ Tìm số nguyên dương n nhỏ thỏa mãn 22013|1999n−1.

Lời giải Áp dụng Định lý ta có

v2(1999n−1) =v2(n) +v2(2000) +v2(1998) = v2(n) +

Để thỏa mãn 22013|1999n−1thì v

2(n) + >2013 hay v2(n)>2008

Vậy số nguyên dương n nhỏ thỏa mãn đề 22008.

Ví dụ (IMO Shortlist 1991) Tìm số nguyên dương k lớn thỏa mãn 1991k là ước của

199019911992+ 199219911990

Lời giải Đặt a= 1991thì a số nguyên tố lẻ Do theo Định lý

va

(a−1)aa+1+ (a+ 1)aa−1=va

(a−1)a2)aa−1 + (a+ 1)aa−1 =va

(a−1)a2 +a+

+va(aa−1)

=a−1 +va

(a−1)a2 +a+

Cũng theo Định lý thìva

(a−1)a2

+ 1=va(a)+va(a2) = 3nênva

(a−1)a2

+a+ 1= Vậy, va

(a−1)aa+1

+ (a+ 1)aa−1

=a Ta thu maxk =a= 1991

Ví dụ 1.(Italy TST 2003) Tìm số nguyên nguyên (a, b, p)sao cho a, blà số nguyên dương,

p số nguyên tố thỏa mãn 2a+pb = 19a.

Lời giải Vì a nguyên dương nên 17|19a−2a Vậy p= 17 Áp dụngĐịnh lý 1 ta có

v17(19a−2a) = v17(17) +v17(a)

⇔b = +v17(a)61 +a

1 Nếu b < +a hay 6 b 6 a Dễ dàng chứng minh quy nạp 19a−2a > 17a với

a>1 Mà17a>17b Vậy a=b= 1 ở trường hợp này.

2 Nếu b= +a dễ dàng chứng minh quy nạp 19a−2a<17a+1 = 17b, mâu thuẫn. Vậy (a, b, p) = (1,1,17) đáp án toán

Ví dụ (IMO 1990) Tìm số ngun dương n cho n2|2n+ 1. Lời giải Với n= thỏa mãn Với n>2, nhận thấy n lẻ

Gọiplà ước nguyên tố lẻ nhỏ củan Khi ta suy ra22n ≡1 (mod p) Gọiklà số nguyên dương nhỏ thỏa mãn2k≡1 (mod p) Khi đók|2n Theo định lý Fermat nhỏ thì 2p−1 ≡1

(mod p) nên k|p−1 Như ta suy gcd(n, k) = nên k|2 Với k = p|1, mâu thuẫn Vậy k = Do p= hay 3|n

Đặt v3(n) =k (k ∈N∗) Áp dụng Định lý ta có

(22)

Lại có n2|2n+ 1 nên v

3(2n+ 1) >v3(n2)⇔k+ 1>2k Vậy k= Đặtn = 3m với m∈N∗

và gcd(m,3) =

Gọi p1 ước nguyên tố nhỏ m Khi ta có 26m ≡1 (mod p1) Gọi k1 số nguyên

dương nhỏ thỏa mãn 2k ≡1 (mod p

1) Tương tự ta dễ dàng suy rak|6 Vì p1 >5 nên

k = k =

Với k = p1|7 nên p1 = Với k = p1|63 mà p1 > nên p1 = Tuy nhiên

2n+ = 23m+ = 8m+ 1 ≡2 (mod 7)mà 7|n2, mâu thuẫn.

Vậy ước nguyên tố n là3 mà 3kn nên n = Số nguyên dương n thỏa mãn đề n ∈ {1; 3}

Ví dụ 2.(European Mathematical Cup 2012, Senior Division) Tìm số nguyên dươnga, b, n số nguyên tố pthỏa mãn

a2013+b2013 =pn

Lời giải Đặt a=px·y, b=pz·t với x, y, z, t∈

N; t, y >1và gcd(y, p) = 1,gcd(t, p) = Khơng làm tính tổng quát, giả sử rằngx>z Dễ nhận thấy rằngn>2013x>2013z Khi phương trình ban đầu tương đương với

t2013+p2013(x−z)·y2013 =pn−2013z

Nếu x > z p-V T Do p-pn−2013z suy n= 2013z Vậy ta phương trình

t2013+p2013(x−z)·y2013 = 1,

mâu thuẫn V T >2 (do , ty>1) Vậy x=z Phương trình trở thành

t2013 +y2013 =pn−2013z =pk(k =n−2013z ∈N∗) (3) Nếup|2013thì theo định lý Fermat nhỏ ta suy rat2013+y2013 ≡2 (mod p), mâu thuẫn p|pk Vậy gcd(p,2013) =

Dễ thấy theo (3) p|t+y Do việc áp dụng Định lý ta có

vp t2013+y2013

=vp(t+y) Ta lại có t+y|t2013+y2013 và (3) nên ta suy ra

pk =t+y =t2013+y2013 t2012(t−1) +y2012(y−1) =

Vì t, y>1nên từ phương trình ta suy t =y= Do đóp= 2, từ suy raa=b= 2h, n=

2013h+ với h∈N

Nhận xét Ta tổng qt tốn lên thành: Giải phương trình nghiệm nguyên dương

an+bn=pk

với p nguyên tố

Ví dụ (Romanian IMO TST 2005) Giải phương trình nghiệm nguyên dương

3x = 2x·y+

(23)

1 Nếu x lẻ áp dụng Định lý ta có v2(3x−1) =v2(3−1) = hay v2(2x·y) = Do

đóx= Từ phương trình ta suy y= Nếu xchẵn áp dụng Định lý ta có

v2(3x−1) =v2(3−1) +v2(3 + 1) +v2(x)−1 = +v2(x)

⇔v2(2x·y) = +v2(x)⇔x+v2(y) = v2(x) + (1)

Đặt x= 2m·k với m, n∈N∗ Ta dễ dàng chứng minh quy nạp rằng 2m·k > m+ 2 với m ∈ N, m > Do x > v2(x) + với v2(x) > hay với x> 2v2(x) = Như

x > (1) không xảy Vậy x 6 8, x chẵn nên x ∈ {2; 4; 6} Từ ta tìm

(x, y) = (2; 2),(4; 5)

Vậy phương trình có nghiệm ngun dương(x, y) = (1; 1),(2; 2),(4; 5)

Nhận xét Qua toán trên, ta lưu ý số ý tưởng dùng phương pháp này: Với

p ước nguyên tố củaa=pm·k với m, k∈

N∗ thì: i) a>pvp(a)

ii) pm·k>m+α với m>β Từ suy raa>v

p(a) +α với vp(a)>β hay a>pβ

Các chủ yếu khơng khó để vận dụng bổ đề LTE ta xác định yếu tố p, a, b cách dễ dàng Tuy nhiên, có số tốn địi hỏi ta phải tìm yếu tố p, a, b

Ví dụ (IMO 1999) Tìm tất cặp (n, p) nguyên dương cho p số nguyên tố

(p−1)n+ 1 chia hết chonp−1.

Lời giải Dễ thấy với n = p số nguyên tố thỏa mãn đề Với n > 2, ta có trường hợp:

Trường hợp Nếup= n|2 Do n=

Trường hợp Nếu p lẻ Lấy q ước nguyên tố nhỏ n, (p− 1)n ≡ −1

(mod q)hay (p−1)2n≡1 (mod q)và gcd(p−1, q) = 1 Ta lấyo là số nguyên dương nhỏ nhất thỏa mãn (p−1)o ≡1 (mod q) Khi ta suy ra o|2n Áp dụng định lý Fermat nhỏ ta có

(p−1)q−1 ≡1 (mod q) Do đó o|q−1.

Như vậy,o|2n o|q−1 Nếu gcd(o, n)>1hay o, n chia hết cho số nguyên tốr, ta suy rar|n r6o Mà o|q−1nên o < q, đór < q Màr qđều ước nguyên tố n, mâu thuẫn với điều kiện nhỏ q Vậy gcd(n, o) = Do 2|o Vậy (p−1)2 ≡ (mod q)

hay q|p(p−2)

1 Nếu q|p−2 ta có (p−1)n+ 1≡1n+ 1≡2 (mod q) Vậy q = 2 Ta có (p−1)n+ 1 chia hết cho2 nên p= 2, mâu thuẫn p lẻ

2 Nếuq|p Dễ nhận thấyn phải lẻ (vì n chẵn thì(p−1)n+ 1≡0 (mod 4), mâu thuẫn p lẻ) Ta áp dụng Định lý ta có

(24)

Đặt p =qa·b với a, b∈

N∗ Dễ dàng chứng minh quy nạp qa·b > a+ (chú ý

q|p nên q >3), dấu xảy a =b = 1, q = Do p> vq(p) + Kết hợp với

(3) ta suy

p−2>vq(p)>vq(n)(p−2)

Vậy q =p = v3(n) = Đặt n = 3k với k ∈ N∗, gcd(k,3) = 1, gcd(k,2) = Như

vậy từ đề ta có 9k2|8k+ 1.

Hiển nhiên9|8k+ 1 Ta cần tìm k sao chok2|8k+ 1 Vớik = 1 thìn = 3, thỏa mãn. Với k >2, hoàn toàn tương tự, lấy r ước nguyên tố nhỏ k s số nguyên dương nhỏ cho 8s ≡ 1 (mod r) Ta suy ra s|2 nên s = 2 Khi đó r|82 −1 hay

r|7, điều mâu thuẫn 8k+ 1≡2 (mod 7). Vậy, cặp số (n, p) thỏa mãn đề (1, p),(2,2),(3,3)

Ví dụ (Brazil XII Olympic Revenge 2013) Tìm ba số (p, n, k) nguyên dương thỏa mãn p số nguyên tố Fermat

pn+n= (n+ 1)k (5)

Số nguyên tố Fermat số nguyên tố có dạng 22x

+ với x tự nhiên

Lời giải Đặt α= 2x Nếu n = 1 thì (5) ⇔p= 2k−1 = 2α+ 1 Do đó k = 2, α= 1 nên p= 3. Nếu n > Ta gọi r ước nguyên tố n Từ phương trình ta suy pn ≡1 (mod n) hay pn ≡1 (mod r) Do đó gcd(p, r) = 1 Đặtk là số nguyên dương nhỏ thỏa mãn pk≡1

(mod r) Ta có theo định lý Fermat nhỏ thìpr−1 ≡1 (mod r) Vậy ta suy ra k|r−1 và

k|n Vì gcd(r−1, n) = nên k= Ta có r|p−1 hay r|2α Vậy r= 2 hay 2|n Ta có

(5)⇔pn−1 = (n+ 1)(n+ 1)k−1−1

Từ phương trình dẫn đến v2(pn−1) =v2 (n+ 1)k−1−1

Nếu k−1 lẻ

v2 (n+ 1)k−1−1

=v2(n)< v2 p2−1

+v2(n)−1 = v2(pn−1),

mâu thuẫn Vậy k−1 chẵn Áp dụng Định lý ta có

v2(pn−1) =v2 (n+ 1)k−1−1

⇔v2(p2−1) +v2(n)−1 =v2(n) +v2(n+ 2) +v2(k−1)−1

⇔v2(p−1) +v2(p+ 1) =v2(n+ 2) +v2(k−1)

Nếu v2(k−1) > v2(p−1) p−1|k Do (n+ 1)k ≡ n+ (mod p) theo định lý Fermat

nhỏ Tuy nhiên theo (5) n ≡ (n+ 1)k (mod p) nên n ≡ n+ (modp), mâu thuẫn Vậy

v2(k−1)< v2(p−1) Khi theo phương trình ta có

16v2(p+ 1) =v2(2α+ 2) < v2(n+ 2)

Do đóv2(n+ 2) >2 Ta suy n ≡2 (mod 4)

1 Nếu p >5 22x

(25)

• Nếun ≡0 (mod 5)thì +n−(n+ 1)3 ≡3 (mod 5), mâu thuẫn.

• Nếun ≡1 (mod 5)thì +n−(n+ 1)3 ≡2 (mod 5), mâu thuẫn

• Nếun ≡2 (mod 5)thì +n−(n+ 1)3 ≡4 (mod 5), mâu thuẫn.

• Nếun ≡3 (mod 5)thì +n−(n+ 1)3 ≡3 (mod 5), mâu thuẫn.

• Nếun ≡4 (mod 5)thì +n−(n+ 1)3 ≡3 (mod 5), mâu thuẫn Vậy với mọin ∈N∗ n+ 46≡(n+ 1)3 (mod 5) Ta loại trường hợpp >5

2 Nếu p= α = Khi =v2(n+ 2) +v2(k−1) Vì v2(n+ 2) > nên ta suy

rav2(n+ 2) = 2, v2(k−1) = Ta có 5n+n= (n+ 1)k

• Với n= k =

• Với n >3 Gọi q ước nguyên tố lẻ n q|5(n,q−1)−1 = 52−1 = 24 Vậy q|3

nên q = Do n ≡ (mod 6) Kết hợp với n ≡ (mod 4) ta suy 5n ≡ −1 (mod 13)nên n−1≡(n+ 1)k (mod 13) Áp dụngĐịnh lý ta có

v3(5n−1) = v3 (n+ 1)k−1−1

⇔1 +v3

n

2

=v3(k−1) +v3(n)

Vậy 3|k−1 Ta có k ≡ (mod 4) nên k ≡7 (mod 12) Theo định lý Fermat nhỏ ta suy (n+ 1)k ≡ (n+ 1)7 ≡ ±(n+ 1) (mod 13) Như vậy n−1≡ −n−1

(mod 13) dẫn đến n ≡ (mod 13), vơ lý (vì với 13|n 5n ≡ 1 (mod 13), mâu thuẫn do5n≡5 (mod 13)).

Vậy (p, n, k) = (3,1,2),(5,2,3)

Ví dụ Tìm ba số nguyên dương (a, b, c) cho ab+ = (a+ 1)c.

Lời giải Gọi p ước nguyên tố lẻ a Khi theoĐịnh lý ta có

vp((a+ 1)c−1) = vp(a) +vp(c)>vp(a)·b

⇔vp(c)>vp(a)(b−1) (6)

1 Nếuclẻ ta cóv2((a+ 1)c−1) = v2(a) Do đób = Như ta cóa+ = (a+ 1)c

suy c=

2 Nếu cchẵn v2(c)>1 b>2 Theo Định lý

v2((a+ 1)c−1) =v2(a) +v2(a+ 2) +v2(c)−1 = v2(a)·b (7)

• Nếuv2(a) = 1thì ta ln cóv2(c)>v2(a) Kêt hợp với(6)ta suy rac>a(b−1)> b,

mâu thuẫn lúc (a+ 1)c> ab+ 1.

• Nếu v2(a) > (7) ⇔ v2(c) = v2(a) · (b − 1) Kết hợp với (6) ta dẫn đến

c>a(b−1)> b, mâu thuẫn

Vậy phương trình có nghiệm (a, b, c) = (k,1,1)với k số nguyên dương tùy ý Nhận xét Từ tốn trên, ta có thêm số mở rộng sau:

(26)

Mở rộng (IMO Shortlist 2000)Tìm ba số nguyên dương(a, m, n)thỏa mãnam+1|(a+1)n. Ngoài việc phương pháp LTE ứng dụng trực tiếp vào lời giải phương pháp cịn dùng để tìm dạng vơ hạn tốn chia hết

Ví dụ Chứng minh tồn vơ hạn số tự nhiên n thỏa mãn n|3n+ 1.

Phân tích định hướng lời giải Điều ta cần làm tìm dạng củan

thỏa mãn n|3n+ 1.

Trước hết, nhận thấy 5|32 + 12 Bây ta để ý đến điều kiện a, b, p trong Định lý 2, áp

dụng ta được5k+1|32·5k

+ 12·5k

Do đó2·5k|32·5k

+ Vậy ta cần chứng minhn= 2·5k với k ∈N n|3n+ 1.

Lời giải Trước hết, ta chứng minh34·5k−1

≡1 (mod 5k) Áp dụng Định lý 1 ta có

v5

34·5k−1 −1=v5(34−1) +v5(5k−1) = k

Vậy 34·5k−1 ≡ (mod 5k) hay 5k|32·5k −1 32·5k +

Do 5k|32·5k + Lại có 2|32·5k +

nên 2·5k|32·5k +

Vì k ∈N∗ nên tồn vô hạn số tự nhiên n = 2·5k sao cho n|3n+ 1.

Ví dụ (Romanian Master of Mathematics Competition 2012) Chứng minh tồn vô hạn số nguyên dương n thỏa mãn 22n+1+ chia hết cho n

Phân tích định hướng lời giải Ta tìm số n thỏa mãn điều kiện Dễ thấy n =

thỏa mãn Ta mạnh dạn thử với n= 9,27· · · thỏa mãn Từ ta dễ dàng tìm dạng nn n= 3k Ở xin giới thiệu hai lời giải:

Lời giải Ta chứng minh số nguyên dương an = 3n thỏa mãn yêu cầu toán Thật vậy, theo Định lý ta có

v3(2an+ 1) =v3(3) +v3(an) =k+ Và

v3(22

an+1

+ 1) =v3(3) +v2(2an + 1) =k+

Vậy an|22 an+1

+

Lời giải Ta chứng minh số nguyên dương an= 3n+1

9 thỏa mãn yêu cầu đề

Áp dụng Định lý ta có

v3(an) = v3(3) +v3(3n)−2 =n−1

Đặt an = 3n−1m với m∈N∗, gcd(3, m) = Ta có

v3(22

an+1

+ 1)> v3(2an + 1)> v3(an) =n−1 Vậy 3n−1|22an+1+ Mặt khác, tiếp tục áp dụng Định lý

v3(2an + 1) =v3(3) +v3(an) = n Do đó3n|2an + 1 Vậy ta suy ra 23n

+ 1|22an+1

+ Mà m|2an+ 1 nên m|22an+1

+ Vì gcd(m,3) = nên an|22

an+1

(27)

Bài tập vận dụng

1 Chứng minh phương trình x7+y7 = 1998z khơng có nghiệm ngun dương Tìm tất số nguyên dươngn thỏa mãn 72013|5n+ 1.

3 Tìm số nguyên dương n lớn cho 2n|201120132016−1

−1

4 Chứng minh tồn vô hạn số nguyên dươngn ∈Nthỏa mãn n2|2n+ 3n+ 6n+ 1. (Japan MO Finals 2012) Cho p số nguyên tố Tìm số nguyênn thỏa mãn với

số nguyên x, p|xn−1 thì p2|xn−1.

6 Cho a > b > 1, b số lẻ, n số nguyên dương Nếu bn|an−1 Chứng minh

ab > 3nn

7 Tìm số nguyên dương n thỏa mãn 9n−1 chia hết cho7n

8 (IMO Shortlist 2007) Tìm hàm số tồn ánh f :N→N cho với m, n∈N với pnguyên thố, f(m+n)chia hết cho pkhi f(m) +f(n) chia hết chop (IMO 2000) Tồn hay không số nguyên n thỏa mãn n có 2000 ước nguyên tố

2n+ 1 chia hết cho n ?

10 Với số tự nhiên n, cho a số tự nhiên lớn thỏa mãn 5n−3n chia hết cho 2a. Lấyb số tự nhiên lớn thỏa mãn2b 6n Chứng minh rằng a6b+ 3.

11 Chứng minh n>2 cho n|7n−3n thì n chẵn. 12 Tìm số nguyên dương n thỏa mãn

i) n|5n+ ii) n2|5n+ 1. iii) n3|5n+ 1.

13 Tìm số nguyên dương k cho k số nguyên tố lẻ p1, p2· · · , pk tồn hai số nguyên dương a, nthỏa mãn

p1·p2· · ·pk−1 = an

14 (MOSP 2001) Tìm số nguyên dương (x, r, p, n) thỏa mãn xr−1 = pn

15 Tìm tất số(m, p, q)vớip, qnguyên tố vàmnguyên dương cho2mp2+1 =q5 16 (Iran TST 2009) Cho n số nguyên dương Chứng minh

352

n

−1

2n+2 ≡(−5) 32n−1

(28)

17 (IMO Shortlist 2010) Tìm cặp số ngun khơng âm (m, n) thỏa mãn

m2+ 2·3n =m 2n+1−1

18 (Iran Third Round 2011) Cho số tự nhiênk > Có cặp nguyên dương (x, y)

thỏa mãn

7373x ≡99y (mod 2k)?

19 Giải phương trình nghiệm ngun dương đóp số nguyên tố:

pa−1 = 2n(p−1)

Tài liệu tham khảo

[1] Amir Hossein Parvardi, Lifting The Exponent Lemma: (tài liệu pdf) [2] Các diễn đàn toán:

(29)

Các tốn số học hốn vị vịng quanh

Nguyễn Anh Huy, Nguyễn Việt Tâm

Sự bình đẳng hốn vị ẩn số nép đẹp Toán học, thường gặp tốn hệ phương trình bất đẳng thức Tương tự, Số học có tốn hốn vị vịng quanh mà ẩn số ngun, số nguyên dương, nhiên lời giải đa dạng phức tạp nhiều Bài viết đề cập đến hai hướng cụ thể để giải dạng trên, đối xứng hóa bất đẳng thức

Trong viết có sử dụng số kiến thức bước nhảy Viete hàm vp(n), bạn đọc tham khảo hai chuyên đề trước

Phương pháp đối xứng hóa

Phương pháp thường dùng toán chia hết hoán vị vịng:a|f(b); b|f(a)

Nếu có (a;b) = ta xây dựng hàm g thỏa g(x) x ∀x∈Z h thỏa

h(a;b) =f(a) +g(b) = f(b) +g(a) đối xứng theo a, b Khi

a | h(a;b); b |h(a;b)⇒ab| h(a;b)⇒h(a;b) =kab

Đây phương trình nghiệm nguyên với a, b đối xứng Ta dùng bất đẳng thức

degh= bước nhảy Viete degh=

Nếu khơng có(a;b) = từ giả thiết ta suy ab| f(a)f(b), sau khai triển vế phải bỏ hạng tử chia hết cho ab để phương trình nghiệm ngun có dạng tương tự

Bài tập 3.1 Tìm số nguyên tố p>q thỏa

 

q−1 | 3p−1

p−1 | 3q−1

Lời giải Đặt a=p−1, b=q−1 (a>b>1), ta có

 

a | 3(b+ 1)−1

b | 3(a+ 1)−1

⇒ab| (3a+ 2)(3b+ 2) (∗)⇒6a+ 6b+ ab

1Lớp 12CT THPT chuyên Lê Hồng Phong

(30)

Từ ta có

6a+ 6b+ >ab⇒

a +

6

b +

4

ab >1

Lại có

12

b +

4

b2 >

6

a +

6

b +

4

ab

Suy

12

b +

4

b2 >1⇒b

2 −12b−4

60⇒16b 612

Do b+ 1nguyên tố nên b ∈ {1; 2; 4; 6; 10; 12} Xét trường hợp sau, với lưu ýa+ 1nguyên tố:

* Nếu b = : (∗)⇒6a+ + a⇒10 a⇒a∈ {1; 2; 10} * Nếu b = : (∗)⇒6a+ 12 + 2a ⇒8 a⇒a∈ {2; 4}

* Nếu b = : (∗)⇒6a+ 24 + 4a ⇒a+ 14 2a⇒a ∈ {6; 10} * Nếu b = : (∗)⇒6a+ 36 + 6a ⇒40 6a (loại)

* Nếu b = 10 : (∗)⇒6a+ 60 + 10a⇒3a+ 32 5a⇒a= 16 * Nếu b = 12 : (∗)⇒6a+ 72 + 12a (loại)

Từ ta tìm cặp (p, q) nguyên tố thỏa toán

(2,2),(3,3),(5,3),(7,5),(17,11)

Bài tập 3.2 Tìm số số nguyên dương(a;b;c)đôi nguyên tố thoảa < b < c

a | bc−31; b |ca−31; c | ab−31

Lời giải

Do a | bc−31và a | a(b+c) nên ta có

a |ab+bc+ca−31

Tương tự với b c Lại (a;b) = (b;c) = (c;a) = nên

abc | ab+bc+ca−31 (∗)

Xét trường hợp sau:

~ Trường hợp 1:a >3

Suy b >4; c>5, ab+bc+ca >31 Ta có

abc >3bc > ab+bc+ca > ab+bc+ca−31

Điều mâu thuẫn với (*)

~ Trường hợp 2:a =

Suy b | 2c−31và c| 2b−31,

(31)

Chứng minh tương tự ta có b= 3; c=

~ Trường hợp 3:a =

Suy b |c−31 vàc| b−31 Nếub+c= 31 hai điều kiện hiển nhiên thoả Khi ta cóa= < b < cvà b+c= 31 Do a < b < c nên 26b 615 Dễ thấy có 14 số (a;b;c)thoả Nếu b+c6= 31 ta chứng minh 1< b < c <31 Nếu ngược lại:

* Nếu b >31⇒c > b > b−31>0 Do ckhơng ước củab−31 * Nếu b = 31 31| c−31⇒31 |c, loại (b;c) = (31;c) =

* Nếu b <31< c |c|>|31−b|>0 đóc không ước củab−31

Như 1< b < c <31 Ngoài ra, từ b |c−31và c |b−31với (b;c) = ta có

bc | 31−b−c

* Nếu b >5⇒c>6⇒bc>30>11>31−(b+c), vô lý

* Nếu b = | c−31 c| 27⇒c= 27 (loại dob+c6= 31) * Nếu b = | c−31 c| 28, suy c∈ {4; 7}

Vậy trường hợp cho 16 bộ(a;b;c) thoả tốn Kết luận: Có 17 số (a;b;c) thoả tốn

Bài tập 3.3 Tìm số ngun tố p, q thoả p < q <1000

q | p3−1; p | q3−1

Lời giải Ta có q |p3−1 = (p−1)(q2+q+ 1) mà q > p−1 nên

q | p2+p+

Và p | q3−1 = (q−1)(q2+q+ 1) Do ta xét trường hợp:

~ Trường hợp 1:p | q−1

Đặt p2+p+ =nq (n∈

N∗) Do p2+p+ ≡q ≡1 (mod p)nên n ≡1 (mod p) Lại có

n= p

2+p+ 1

q

p2+p+

p+

=

p+

p+

=p (do q>p+ 1)

Suy n = 1, vậyq =p2+p+ 1 Do đó p631, vì372+ 37 + 1>1000.

Thử với tất số nguyên tố p <31ta tìm số thoả tốn

(p;q) = (2; 7),(3; 13),(5; 31),(17; 307)

~ Trường hợp 2:p | q2+q+ 1

Do p | q2+q+ 1 và q |p2+p+ 1 nên ta có

p | q2+q+ +p2+p; q | p2 +p+ +q2 +q

hay

(32)

Như ta xét phương trình nghiệm nguyên dương

a2+b2+a+b+ =mab (∗) (m∈N∗)

Viết lại (*) dạng

a2+ (1−mb)a+b2+b+ = (∗∗)

Gọi S tập số nguyên dương (a;b) thoả (**) Theo nguyên lý cực hạn S tồn cặp số (a0;b0) thoả a0+b0 nhỏ Không giảm tổng quát giả sử a0 >b0

Theo định lý Viete, (**) có nghiệm (a0;b0)trong a0 thoả

 

a0+a0 =mb0−1

a0a0 =b20+b0+

Từ phương trình đầu suy raa0 ∈Z Từ phương trình sau suy a0 >0 Vậy (a0;b0)∈S Do

a0+b0 6a0 +b0 ⇔a0 6a0 =

b2

0+b0 +

a0

⇔b20+b0+ >a20

Nếu a0 > b0 a0 >b0+ 1⇒a20 >(b0+ 1)2 > b20+b0+ 1, mâu thuẫn Vậy a0 =b0, suy

a20 + (1−a0).a0+a20+a0+ =

Suy a0 ⇒a0 =b0 = Vậy ta cóm =

Khi (*) trở thành

a2+b2+a+b+ = 5ab (∗)

Theo chứng minh trên, (a0;b0) nghiệm (*) (5a0−b0−1;a0)cũng nghiệm

(*) Do từ nghiệm(1; 1) ta có nghiệm (3; 1), sau (13; 3), Tóm lại nghiệm (*) cho công thức

(ai;bi) = (xi+1;xi) với (xn) :

 

x0 =x1 =

xn+1 = 5xn−xn−1−1 ∀n>1

Việc chứng minh điều hoàn toàn tương tự số học VMO 2012, xin dành cho bạn đọc

Tiếp theo ta tìm nghiệm số nguyên tố nhỏ 1000 Dễ thấy cần xét i 6 4,

x6 >1000 Từ ta tìm được(p;q) = (3; 13),(13; 61)

Kết luận: Các số (p;q)thoả toán (2; 7),(3; 13),(5; 31),(13; 61),(17; 307)

Phương pháp dùng bất đẳng thức

(33)

Bài tập 3.4 Cho số nguyên dương a, b, c Chứng minh

(a;b).(b;c).(c;a) (a;b;c)2 =

[a;b].[b;c].[c;a] [a;b;c]2

Trong (a;b) [a;b] là ước chung lớn bội chung nhỏ củaa, b Lời giải

Gọi plà số ngun tố Nếupkhơng ước của[a;b;c]thì dễ thấy vp(V T) =vp(V P) = Nếu p | [a;b;c] p ước a, b c Đặt a = px.a1;b = py.b1;c =pz.c1 không giảm

tổng quát giả sử x>y>z Khi ta có

vp(V T) = y+z+z−2z =y

vp(V P) = x+y+x−2x=y Suy vp(V T) = vp(V P) ∀p∈P Do V T =V P

Nhận xét lời giải ta giả sử vp(a) > vp(b) > vp(c), sử dụng định lý số học: Mọi số nguyên lớn có cách phân tích thừa số nguyên tố

Bài tập 3.5 Tìm số nguyên tố p, q thỏa

(5p−2p)(5q−2q) pq (∗)

Lời giải Giả sử p6q Nhận xét p=q = thỏa

> Nếup= 3, q >3thì ta có

117(5q−2q) 3q⇔39(5q−2q) q

Do 5q−2q ≡5−2≡3 [q] theo Fermat nên q | 39, đóq = 13

> Nếu5< p < q (để ý V T(∗)không chia hết p, q 6= 5) tương tự ta có

5p−2p ≡5−2≡3 [p], từ giả thiết suy

5q−2q p

Lại có 5p−1 ≡2p−1 ≡1 [p] nên 5p−1−2p−1 . p.

Do q > p−1nên (q, p−1) = Theo định lý Bezout, tồn m, n∈N∗ thỏa

mq−(p−1)n= m(p−1)−nq = (∗∗)

Xét đồng dư mod p ta có

 

5p−1 ≡2p−1

5q ≡2q

 

5n(p−1) ≡2n(p−1)

(34)

Suy

5n(p−1).2mq ≡2n(p−1).5mq

Kết hợp với (**), sau rút gọn hai vế ta 5≡2 [p] hay p= (loại xét p >5) Kết luận: (p;q) = (3; 3),(3; 13),(13; 3)

Bài tập 3.6 Tìm cặp số nguyên tố (p, q)thỏa

2p+ 2q pq

Lời giải

Nhận xét (x−1, x+ 1) = (2, x+ 1)62 ∀x∈Z * Nếu p=q 2p+1 . p2 ⇒p=q = 2.

* Nếu p > q :

Xét q= ta có 22+ 2p .2p⇒2 + 2p−1 . p⇒p∈ {2; 3}.

Xét q >2thì 2p+ 2q = 2q(2p−1+ 1) q nên

2p−1+ q (1)⇒22(p−q) ≡1 [q]

Gọi a số nhỏ thỏa 2a ≡1 [q] Theo tính chất ord suy q−1 a 2(p−q) a Đặt p−q= 2k.m; q−1 = 2l.n; a = 2r.s (k+ 1 >r;l >r;m . s;n . s;m, n, s lẻ).

> Trường hợp 1:r =k+ Ta suy ral >k+

 

2(p−q) = 2k+1.m q−1 = 2l.n ⇒

 

2(p−q)s =am

(q−1)m = (p−q).n.2l−k−1

⇒ n.2

l−k−1

2s =

(q−1).m

2s(p−q) =

q−1

a =

2l.n

2r.s

⇒ 2l−k−2 = 2l−r ⇒r=k+ (mâu thuẫn) > Trường hợp 2:r 6k

Suy p−q a⇒2p−q−1 2a−1 q (2)

Từ (1) (2) suy q | (2p−q−1,2p−q+ 1)

Mà(2p−q−1,2p−q+ 1) 62theo chứng minh trên, q= (mâu thuẫn xétq >2) Kết luận: (p, q) = (2; 2),(2; 3),(3; 2)

Bài tập 3.7 Cho số nguyên dươngx, y, zthoả(xy+ 1)(yz+ 1)(zx+ 1) số phương Chứng minhxy+ 1, yz+ 1, zx+ số phương

(35)

Trong số (x;y;z)thoả toán, xét (x;y;z)có x+y+z nhỏ (1) Khơng giảm tổng qt giả sử z = max{x;y;z}

Gọi t số thỏa phương trình bậc hai

t2+x2+y2+z2 −2(xy+yz+zt+tx+zx+ty)−4xyzt−4 = (∗)

⇔t2−2t(x+y+z+ 2xyz) +x2 +y2+z2−2(xy+yz +zx)−4 =

Nhận xét (*) tương đương với phương trình sau:

(x+y−z−t)2 = 4(xy+ 1)(zt+ 1) (x+z−y−t)2 = 4(xz+ 1)(yt+ 1) (x+t−y−z)2 = 4(xt+ 1)(yz + 1)

Và t nguyên (*) có nghiệm nguyên

t1,2 =x+y+z+ 2xyz±2

p

(xy+ 1)(yz+ 1)(zx+ 1)

Nên nhân phương trình vế theo vế, ta suy (xt+ 1)(yt+ 1)(zt + 1) số phương

Ngồi ta có

xt+ 1>0; yt+ >0; zt+ >0

Suy

t > −1

max{x;y;z} >−1 (do x=y =z = không thoả)

* Nếu t = từ (*) ta suy

(x+y+z)2 = 4(xy+yz+zx+ 1) ⇔(x+y−z)2 = 4(xy+ 1)

Suy xy+ số phương Chứng minh tương tự ta có yz+ 1, zx+ số phương * Nếu t >0 từ (1) suy t >z với t thoả (*)

Nhưng t t1 t2, ta lại có

t1t2 =x2+y2+z2 −2(xy+yz+zx)−46z2−x(2z−x)−y(2z−y)< z2

Suy điều mâu thuẫn Vậy ta có đpcm

Bài tập 3.8 Tìm số nguyên (a;b;c) thỏa

    

   

a2−bc= 91

b2−ca= 91

c2−ab= 91

(36)

Không giảm tổng quát giả sử a6b6c Do 91 không số phương nên a, b, c6= Nếu a>0thì b, c>0 a2−bc60<91, vậya <0.

Nếu (a;b;c) thỏa hệ (−a;−b;−c) thỏa hệ Do ta xét số có c >0 Nếu b=cthì ta có

a2−b2 = 91 =b2−ab⇔

"

a−b =

a=−2b

Dễ thấy a=b=cvô lý, a=−2b dẫn đến 5b2 = 91, vô lý.

Do đób 6=c Tương tự ta có a6=b

~ Trường hợp 1:b >

Từ hệ ta có

91b =b(b2−ac); 91c=c(c2−ab)

⇒ 91b−91c=b3−c3

⇒ 91 =b2+bc+c2 >3b2

⇒ b∈ {1; 2; 3; 4; 5}

Thay vào hệ ta tìm nghiệm (a;b;c) = (−10; 1; 9),(−11; 5; 6)

~ Trường hợp 2:b <

Từ hệ ta có

91b=b(b2−ac); 91a=a(a2−bc)

⇒ 91(b−a) = b3−a3

⇒ 91 =a2+ab+b2 >3b2

⇒ b ∈ {−1;−2;−3;−4;−5}

Thay vào hệ ta tìm nghiệm (a;b;c) = (−9;−1; 10),(−6;−5; 11)

Tóm lại (a;b;c)thỏa giả thiết

(−10; 1; 9),(10;−1;−9),(−11; 5; 6),(11;−5;−6),(−9;−1; 10),(9; 1;−10),(−6;−5; 11),(6; 5;−11)

và hoán vị

Bài tập tự luyện

Bài 1: (APMO 2002) Tìm số nguyên dương a, bthoả

b2−a | a2+b; a2−b | b2+a

Bài 2: Chứng minh có vơ hạn số nguyên dương(a;b;c) thoả ab+ 1, bc+ 1, ca+ số phương

Bài 3: Tìm số nguyên dương x, y, z đôi nguyên tố thoả

x y +

y z +

z x ∈N

Bài 4: Tìm số nguyên (x;y;z) thoả 26x6y6z

(37)

Bài 5: Tìm số nguyên dương a, b, c >1đôi khác thỏa

(a−1)(b−1)(c−1)| abc−1

Bài 6: Chứng minh với số nguyên dương a, b, nthì

(36a+b)(36b+a)6= 2n

Bài 7*: (VMO 2013) Tìm số (a;b;c;a0;b0;c0) với a, b, c, a0, b0, c0 ∈ {0; 1; 2; ; 14} thoả

ab+a0b0 ≡bc+b0c0 ≡ca+c0a0 ≡1( mod15)

Bài 8: (VMO 2012) Tìm số nguyên dương lẻ a, bthoả

a | b2+ 2; b |a2+

Bài 9: Cho a, b, c∈Z thỏa

a b +

b c +

c a =

Chứng minhabc lập phương số nguyên

Tài liệu tham khảo

1 Trang web brilliant.org

2 Các chuyên đề số học - Phan Huy Khải

3 Các phương pháp giải toán qua kỳ thi Olympic - Trần Nam Dũng (Chủ biên), Võ Quốc Bá Cẩn, Lê Phúc Lữ

(38)(39)

DÃY SỐ SỐ HỌC

Ninh Văn Tú1

Dãy số vấn đề thiết yếu giải tích ứng dụng vào nhiều lĩnh vực khác phương trình hàm, tổ hợp, số học Những toán giới hạn dãy số dường trở thành vấn đề quen thuộc xuất nhiều kì thi học sinh giỏi cấp trường, cấp tỉnh, cấp thành phố kì thi Olympic, VMO Nhưng mảng đặc biệt dãy số việc ứng dụng số học toán số học để giải toán dãy số vấn để thú vị mảng dãy số Hi vọng chuyên đề giúp ích cho bạn việc tiếp thêm kinh nghiệm mảng thú vị

Dãy số nguyên tính chất số học

Ta xét tổng quan toán với dạng tổng qt để có nhìn bao quát hết giới dãy nguyên phong phú, đa màu Từ ta có ý tưởng gặp toán chứng minh dãy nguyên toán liên quan đến dạng

Bài tập 4.1 Cho a, b, c∈Z thỏa mãn a2 =b+ Dãy số (un) xác định

(

u0 =

un+1 =aun+

p

bun2+c2,∀n ∈N Chứng minh số hạng dãy số nguyên

Lời giải

Cách giải tốn tương tự cách tìm cơng thức truy hồi dạng công thức truy hồi thức để tìm cơng thức tổng qt dạng dãy với điều kiện ngặt Ta có:

un+1−aun =

p

bun2+c2

⇒un+12−2aunun+1+a2un2 =bun2+c2

⇒un+12 = 2aunun+1−un2+c2 (1)

1Học sinh THPT chuyên Trần Đại Nghĩa.

(40)

Giảm n xuống đơn vị ta có:

un2 = 2aunun−1−un−12+c2 (2)

Lấy(1)−(2), ta có:

un+12−un−12 = 2aun(un+1−un−1)⇒

"

un+1 =un−1

un+1 = 2aun−un−1

*Với un+1 =un−1, ∀n>1 dãy(un)là dãy tuần hồn theo chu kì Như , ta suy

(

u0 =u2 =u4 = =u2k =

u1 =u3 =u5 = =u2k+1 =|c|

Nên (un) có số hạng số nguyên *Với (un) thỏa

(

u0 = 1; u1 =|c|

un+1 = 2aun−un−1, ∀n>1

,

Ta có hệ số hệ thức truy hồi số nguyên số hạng dãy số nguyên nên từ ta suy (un) có số hạng số nguyên

Tóm lại, số hạng dãy số nguyên

Bài tập 4.2 Cho dãy (an)thỏa mãn điều kiện

(

a0 = 2; a1 =

an+1an−1−an2 = 6n−1, ∀n>1

Chứng minh tồn dãy số nguyên dương thỏa mãn điều kiện Lời giải

Ý tưởng toán tìm cơng thức truy hồi cho dãy Như vậy, ta dùng phương pháp quen thuộc để đánh giá tốn, phương pháp sai phân

Ta có

(

an+1an−1−an2 = 6n−1

anan+2−an+12 = 6n

⇒an(an+2+ 6an) =an+1(an+1+ 6an−1)

⇒(an+2+ 6an)

an+1

= (an+1+ 6an−1)

an Đặt =

(an+1+6an−1)

an , ta

vn+1 =vn =vn−1 = =v1 = ⇒an+1 = 5an−6an−1, ∀n= 1,2,3

Do số hạng đầu dãy số nguyên: a0 = 2, a1 = 5và công thức truy hồi dãy

(41)

Để có thêm điều kiện dãy có số hạng số nguyên dương, ta cần phải có thêm yếu tố đơn điệu tăng Việc chứng minh đơn điệu sử dụng công thức truy hồi mà ta vừa tìm ta chưa chứng minh cơng thức Chính mà ta chứng minh trực tiếp quy nạp thông qua cách diễn đạt đề

Ta thấy điều với n = : a1 > a0 (5>2)

Giả sử điều đến n =k : ak > ak−1

Xét n=k+ 1, ta có:

ak+1ak−1−ak2 = 6k−1 ⇒ak+1 =

6k−1+ak2

ak−1

>

k−1

ak

+ak> ak (ak> ak−1)

Vậy điều với n=k+ nên theo nguyên lý quy nạp ta suy (an) dãy đơn điệu tăng với mọin ∈N

Bây ta chứng minh dãy dãy thỏa mãn điều kiện đề bài:

Cách thông thường mà nghĩ đến dùng phản chứng để chứng minh tồn

Giả sử tồn dãy an0 cho với n>2 tồn tạian+20 :an+2 > an+20 thỏa:

(

anan+2−an+12 = 6n

anan+20−an+12 = 6n

Suy

an(an+2−an+20) = (vô lý an+2 > an+20)

Như ta suy dãy dãy thỏa mãn điều kiện đề

Những tốn tương tự dạng tồn nhiều dạng khác Đôi lúc cách diễn đạt truy hồi khiến ta phương hướng sai phân Nhưng ta cần phải hiểu mục đích việc chứng minh dãy số nguyên tìm cơng thức truy hồi

Bài tập 4.3 (GER 2003, Ngày 2) Cho dãy số(an) xác định

 

a1 = 1; a2 = 1; a3 =

an+3 =

an+2an+1+

an

,∀n ∈N

Chứng minh số hạng dãy số nguyên dương Lời giải

Ta có:

an+3an=an+1an+2+

(42)

nhoáng ý nghĩ Hạ n xuống bậc, ta có:

an+2an−1 =anan+1+

Trừ biểu thức cho nhau, ta

an+3an−an+2an−1 =an+1an+2−anan+1

⇒an(an+3+an+1) =an+2(an+1+an−1)

⇒an+3+an+1

an+2

= an+1+an−1

an Đặt = an+1an+an−1, ta cóvn+2=vn

Vậy dãy (vn) tuần hồn theo chu kì

Như ta cần xét thêm tính chẵn lẻ dãy, điều dẫn đến dãy gồm công thức truy hồi song song

*Với n chẵn ta suy ra:

v2k+2 =v2k = =v2 = 3⇒an+1 = 3an−an−1

*Với n lẻ ta suy ra:

v2k+3 =v2k+1 = =v3 = 5⇒an+1 = 5an−an−1

Từ ta xác định cơng thức truy hồi dãy là:

   

  

a1 =a2 = 1, a3 =

an+1 = 3an−an−1, n = 2k

an+1 = 5an−an−1, n = 2k+

(k ∈N)

Do số hạng đầu dãy số nguyên hệ số hệ thức truy hồi dãy số nguyên nên ta suy số hạng (an)đều số nguyên

Để dãy dãy số nguyên dương, ta cần chứng minh dãy dãy tăng ngặt với

n >2

Dễ thấy: a3 > a2 nên điều vớin =

Giả sử điều với n =k : ak > ak−1

Xét n=k+ 1, ta có:

ak+1 =

akak−1+

ak−2

> akak−1+ ak−1

> ak+

7

ak−1

> ak

Vậy điều với n =k+ nên theo nguyên lý quy nạp ta suy (an) dãy tăng ngặt

Từ đó, ta chứng minh dãy (an) có số hạng số nguyên dương

Bài tập 4.4 (Croatia TST 2011) Với a, b số nguyên tố phân biệt, cho dãy (xn) thỏa mãn

  

 

x1 =a, x2 =b

xn+2 =

xn+12+xn2

xn+1+xn

, ∀n ∈N

(43)

Lời giải

Ta dễ dàng chứng minh xn>0,∀x∈N theo quy nạp Ta chứng minh bổ đề dãy này:

Bổ đề: Nếu kể từ phần tử n0 cho xn0 không số nguyên dãy mà có dạng phân thức, khơng tồn n để xn ∈N, ∀n >n0

Giả sử x3 số hữu tỉ

Ta có:

    

   

x3 = a 2+b2 a+b =

c

d (c, d∈N; (c, d) = 1)

x4 =

(c d)

2 +b2 c d+b

= cd2(+c+b2bdd2)

Do (c, d) = nên x4 ∈/ N, đặt

x4 =

e f

e, f ∈N; (e, f) = 1; f d

Ta có: f d⇒f >d Ta chia làm trường hợp:

> Trường hợp 1:f > d ⇒1> df, ta có:

x5 =

c d

2

+fe

2

c d +

e f

= (cf)

2

+ (ed)2 (cf +de)df

Giả sử x5 ∈N, suy

(cf)2+ (de)2 df ⇒(de)2 df ⇒de2 f (vô lý)

Mặt khác tử số mẫu số f < (cf)2+ (de)2 nên f chia hết cho (cf)2+ (de)2

Điều vô lý (e, d) = d < f Vậy x5 số vô tỉ

> Trường hợp 2:f =d, ta có:

x5 =

c d

2

+e

f

2

c d +

e f

= (cf)

2

+ (ed)2 (cf +de)df =

c2 +e2

f(c+e)

Giả sử x5 ∈N, suy

c2+e2 c+e⇒2ce c+e

Điều hiển nhiên vơ lý

Vậy tóm lại, ta chứng minh rằng, xuất phát với x3 số hữu tỉ ta thu

một dãy số hữu tỉ khơng tồn số nguyên dãy với n >3

(44)

thức nên x3 có dạng phân số tối giản

Giả sử x3 ∈N, suy

a2+b2 a+b⇒2ab a+b

Xét hai số số nguyên tố chẵn Giả sử làa, suy

a= ⇒4b b+ 2⇒4 b+ (vô lý b >2)

Suy 4b=b+ (vô lý dob số nguyên tố lẻ nên tính chẵn lẻ vế khơng đồng nhất) Kí hiệu{a, b} ước củaa b

Vậy a+b | {a, b,2} do(a, b,2) =

Ta có nhận xét a+b | {a, b,2} do: * a+b | {a, b}

(

a>a+b b>a+b ⇒

(

0>b

0>a (sai a, b >0)

* a+b | {2}: Do a, b số nguyên tố nên a+b >2 Suy

⇒a+b= 2ab⇒

 

a+b a a+b b

 

cb a a b

⇒a=b (sai a6=b)

Như vậy, ta kết luận điều giả sử sai nên x3 số nguyên

Vậy theo bổ đề ta suy xn số nguyên với n>3

Bài tập 4.5 Cho dãy số nguyên dương cho ta chọn 1998 phần tử dãy tạo thành hệ thăng dư không đầy đủ riêng biệt xét modulo 1999 Liệu có tồn cách chia 1998 phần tử thành tập A, B cho tích phần tử tập không?

Lời giải

Một suy luận đơn giản cho này: tích chúng phải có số dư xét modulo với số nguyên dương

Sử dụng bổ đề quen thuộc: Nếu x2+y2 .p (p nguyên tố )thì p= 4k+ 3 hoặc p= 4k+ 1 Khi

p= 4k+ x, y p

Gọi 1998 phần tử dãy thỏa điều kiện đề x1;x2;x3; ;x1998

Ta có: A∪B ={x1;x2; ;x1998}

Giả sử∃xi ≡0 (mod 1999) Do 1998 phần tử lập thành hệ thăng dư không đầy đủ riêng biệt nên

xj ≡r6= (mod 1999)∀j 6=i

Vì nếuxi ∈Athì A chia hết cho 1999 B không chia hết cho 1999 Tương tự xi ∈B Vậy khơng có phần tử dãy chia hết cho 1999 Vậy hệ thăng dư dãy:{1; 2; ; 1998} Ta có:

Y

xk∈A

xk2

!

= Y

xi∈A

xi

Y

xj∈B

(45)

Suy

Y

xk∈A

xk

!2

≡1998! (mod 1999)

Mặt khác theo định lý Wilson ta có: 1998!≡ −1 (mod 1999),

Y

xk∈A

xk

!2

+ 1≡0 (mod 1999)

Áp dụng bổ đề ta suy điều vô lý 1999 số nguyên tố dạng 4k+ nên chia hết cho 1999

Vậy khơng thể chia 1998 phần tử dãy ngun dương thành tập có tích phần tử tập

Bài tập 4.6 Cho dãy số (un) thỏa

  

 

u1 = 1, u2 =

un+1 =

unun−1(n+ 1)

Pn−1

a) Tìm số nguyên tố pđểS = up−2−1

pp−1 số nguyên b) Chứng minh un−1

n

≡0 (modn−1)

Lời giải

Ý tưởng đơn giản tìm plà số nguyên tố để up−2 −1≡0 (modpp−1) Qua đó, ta

dự đốn cơng thức tổng qt un để đưa toán số học đơn

> Câu a:

Ta chứng minh theo qui nạp un=n! ∀n ∈1,2,3 Thật điều với n= 1; :

(

u1 = = 1!

u2 = 2.u1 = 2!

Giả sử điều đến n =ktức uk =k! Xét uk+1, ta có:

uk+1 =

(k+ 1)ukuk−1

Pk−1

= (k+ 1)!(k−1)!

(k−1)! = (k+ 1)!

Vậy điều đến n =k+ 1nên theo nguyên lý quy nạp ta suy raun =n!∀n= 1; 2; Vậy S = (p−pp2)!−1−1.Giả sử S số nguyên, ta suy S.pp

−1+ = (p−2)!.

Xét p >5 ta có:

(p−2)! ≡0 (modp−1)⇒S.pp−1+ 1≡0 (modp−1)⇒S+ ≡0 (modp−1)

Mặt khác: S+ 2< p−1⇔(p−2)!<1 +pp−2(p−3)(đúng).

(46)

> Câu b:

+Trường hợp 1: n số nguyên tố Theo định lý Wilson ta có:

(n−1)! ≡ −1 (modn)⇒ (n−1)! +

n ∈N∗

Lại có

hun−1

n

i

=

(n−1)!

n

=

(n−1)! +

n −

1

n

= (n−1)! +

n −1 (do 0<

1

n <1)

Suy

hun−1

n

i

= (n−1)!−(n−1)

n =

(n−1) [(n−2)!−1]

n

Mặt khác, (n;n−1) = 1⇒ (n−2)!n −1 ∈N∗ nên ta suy

hun−1

n

i

≡0 (modn−1)

+ Trường hợp 2: n khơng bình phương số ngun tố⇒n =rs với 1< r < s6n−1 Do r < s 6 n −1 nên ta suy r < s 6 n − nên r, slà ước số số

{1; 2; ;n−2} Vậy

(n−2)!

n =

(n−2)!

rs ∈N∗ ⇒

hun−1

n

i

= (n−1).(n−2)!

n = (n−1).k ≡0 (modn−1)

+Trường hợp 3: n bình phương số nguyên tố Đặt n = p2 với 6 p < n−1

p= hiển nhiên ta có điều phải chứng minh

Lập luận tương tự trường hợp 2, p ước số {1; 2; ;n−2} Bây ta xét xem liệu p cịn ước số khơng

Thật vậy, ta có: n−1 = p2−1 = (p−1)(p+ 1)>2p > p

Vậy 2p ước số số {1; 2; ;n−2} Chính mà

(n−1)!≡0 (mod 2p.p.(n−1))⇒(n−1)!≡0 (mod 2n(n−1))

⇒(n−1)!

n ≡0 (modn−1)⇒

hun−1

n

i

≡0 (modn−1)

Từ điều ta có đpcm

Bài tập 4.7 Cho dãy số xác định bởi:

an+1 ={an}[an], n>0 Chứng minh rằngan=an+2 khin đủ lớn

(47)

> Xét0< a0 <1, ta có

[a0] = 0⇒an = [an]{an}= 0, ∀n= 1,2,3 > Xéta0 >1, ta có:

an+1 ={an}[an]<[an]< an⇒0<[an+1]<[an]< <[a1]⇒[an0] =c1 n0 đủ lớn > Xéta0 <0⇒an <0, ∀n= 1,2,3 ta có:

an+1 ={an}[an]> an ⇒0>[an+1]>[an]> >[a1]⇒[an0] =c2 n0 đủ lớn Như [an0] =k với n0 đủ lớn

Ta có:

an+1 ={an}[an] =an[an]−([an])

2

=kan−k2,∀n >n0

Sử dụng phương pháp sai phân với hạng tử dãy với hệ số tương ứng ta có:

                  

kn−1an+1=knan−kn+1

knan+2 =kn−1an+1−kn

kai+n−1 =k2ai+n−2 −k3

ai+n=kai+n−1−k2

⇒ai+n =knan−

k2(1−kn)

1−k =k

n

an+

k2

1−k

− k

2

1−k

Nếu |k|>1

an+i =kn

an+

k2

1−k

− k

2

1−k → ∞ (vô lý dãy bị chặn)

Suy −16k61⇒k ∈ {−1; 0; 1} * Nếu k = : an = 0, ∀n > n0

* Nếuk = : an+1 =an−1, ∀n >n0 (vô lý do[an0] =k ∀n >n0 mà khoảng cách hạng tử liên tiếp dãy 1)

* Nếu k =−1suy

an+1 = 1−an, ∀n >n0 ⇒

(

an+1 = 1−an

an+2 = 1−an+1

Trừ ta cóan=an+2, ∀n >n0 (đpcm)

Dãy số nguyên tính phương

Bài tập 4.8

a) Chứng minh +

5

!n

+ 3−

5

!n

là số nguyên

b) Chứng minh số hạng lẻ dãy +

5

!n

+ 3−

5

!n

(48)

Lời giải > Câu a:

Xét dãy số sau:

(

x1 = 3; x2 =

xn+2 = 3xn+1−xn∀n= 1; 2; Ta chứng minh theo quy nạp

xn=

3 +√5

!n

+ 3−

5

!n

∀n= 1; 2;

Thật điều với n= 1; :

            

x1 =

3 +√5

!

+ 3−

5

!

=

x2 =

3 +√5

!2

+ 3−

5

!2

=

Giả sử điều đến n =ktức xk =

3 +√5

!k

+ 3−

5

!k

Xét xk+1, ta có:

xk+1 = 3xk−xk−1 =

3 +√5

!k

+ 3−

5

!k −

3 +√5

!k−1

+ 3−

5

!k−1 

⇔xk+1 =

3 +√5

!k−1

7 + 3√5

!

+ 3−

5

!k−1

7−3√5

!

= +

5

!k+1

+ 3−

5

!k+1

Vậy điều đến n =k+ nên theo nguyên lý quy nạp ta suy

xn=

3 +√5

!n

+ 3−

5

!n

∀n= 1; 2;

Do xn=

3+√5

n

+3−

5

n

∀n= 1; 2; có công thức truy hồi với hệ số nguyên x1;x2

nguyên nên từ ta suy xn nguyên Vậy

3 +√5

!n

+ 3−

5

!n

nguyên∀n = 1; 2;

> Câu b:

Tiếp tục với ý tưởng biện luận dãy nguyên theo công thức truy hồi chút biến đổi khéo léo để chứng minh số phương

Ta có:

3 +√5

!n

+ 3−

√ !n −2 =   √

5 +

!2n

+

5−1

!2n

−2

=

" √

5 +

!n

− √

5−1

(49)

Vậy toán trở thành toán thứ ta việc chứng minh xn =

5 +

!n

− √

5−1

!n

nguyên với n lẻ

Chứng minh theo quy nạp xn=

5 +

!n

− √

5−1

!n

có cơng thức truy hồi

(

x1 = 1; x2 =

5

xn+2 =−

5xn+1+ 2xn∀n= 1; 2;

và x2n =α

5;x2n+1 =γ

Thật điều với n= 1; :

            

x1 =

5 +

!

− √

5−1

!

=

x2 =

5 +

!2

− √

5−1 !2 = √ = √

Giả sử điều đến n = 2k tức x2k+2 =

5x2k+1 −x2kvà x2k+1 nguyên x1;x3;

nguyên vàx2k=α

5 Vậy x2k+2 =β

5 (β =x2k+1−α)

Xét x2k+3;x2k+4 Ta có:

x2k+3 =

5x2k+2−x2k+1

=√5

5 +

!2k+2

− √

5−1

!2k+2 −

5 +

!2k+1

− √

5−1

!2k+1 

⇒x2k+3 =

5 +

!2k+1

5 +√5

2 −1

!

− √

5−1

!2k+1

5−√5

2 −1

!

⇒x2k+3 =

5 +

!2k+1

3 +√5

!

− √

5−1

!2k+1

3−√5

!

=

5 +

!2k+3

− √

5−1

!2k+3

(1)

x2k+4 =

5x2k+3−x2k+2

=√5

5 +

!2k+3

− √

5−1

!2k+3 −

5 +

!2k+2

− √

5−1

!2k+2 

⇒x2k+4 =

5 +

!2k+2

3 +√5

!

− √

5−1

!2k+2

3−√5

!

=

5 +

!2k+4

− √

5−1

!2k+4

(2)

Lại có x2k+3 =

5x2k+2−x2k+1 = 5β−x2k+1 số nguyên (3)

và x2k+4 =

5x2k+3−x2k+2 =

(50)

Từ (1);(2);(3);(4) ta suy điều vớin =k+ nên theo nguyên lý quy nạp ta chứng minh

(

x1 = 1; x2 =

5

xn+2 =−

5xn+1+ 2xn∀n= 1; 2;

và x2n =α

5;x2n+1 =γ

Do x2n+1 =γ nên ta suy số hạng lẻ dãy số phương

Dựa vào ta xét thêm thú vị sau:

Bài tập 4.9 Cho dãy số (xn) thỏa mãn:

(

x0 = 1;x1 =

xn+2 = 6xn+1−xn

Chứng minh với n>1 (xn) khơng số phương Lời giải

Tương tự ý tưởng 2, ta biến đổi khéo léo để đưa y dạng Nhưng điều đặc biệt ta cần dãy chứng minh song song quy nạp để củng cố cho lời giải toán thêm chặt chẽ (Lưu ý: Lời giải để bổ sung cho ta thấy ứng dụng mạnh 2)

Dễ dàng chứng minh theo quy nạp:

xn=

3 + 2√2n+ 3−2√2n

Ta biến đổi xn theo hình thức sau:

            

xn=

2 + 12n+ √2−12n

2 =

h √

2 + 1n+ √2−1n

i2

2 −1

xn=

2 + 12n+ √2−12n

2 =

h √

2 + 1n− √2−1ni

2

2 +

Và ta dung phương pháp truy hồi để tìm cơng thức truy hồi √2 + 1n± √2−1n

lần lượt có cơng thức truy hồi là:

(

v0 = 2;v1 =

2

vn+2 =

2vn+1−vn

(

z1 = 2;z2 =

2

zn+2 =

2zn+1−zn Theo ta chứng minh chúng có tính chất

(

v2n=a

v2n+1 =b

2(a;b ∈Z)

(

z2n=c

2

z2n+1 =d

(c;d ∈Z)

(51)

Một vấn đề thú vị ta lại phải cần đến dãy song song để chứng minh Chính dãy song song tạo nên thú vị tốn, khiến trở nên chặt chẽ lời giải Vì xét n chẵn lẻ ta diễn đạt xn theo hình thức hình thức có dạng 2x22 ±1 nên hiển nhiên khơng thể số phương với n>1

Một lời giải khác chung ý tưởng lời giải ngắn gọn hơn: Xét dãy số nguyên

(

u0 = 0; u1 =

un+2 = 2un+1+un Ta dễ dàng chứng minh theo quy nạp

un=

(√2 + 1)n− 1−√2n

2

Mặt khác, dãy

(

x0 = 1;x1 =

xn+2 = 6xn+1−xn

có cơng thức tổng quát

xn=

3 + 2√2n+ 3−2√2n

2 =

2 + 12n+ √2−12n

Vây xn + (−1)n+1 = un2 Từ ta kết luận xnkhơng thể số phương với

n >1

Bài tập 4.10 (CIS 1992) Cho dãy số an xác định

(

a1 =

an+1 =a12+a22+a32+ +an2+n∀n >1 Chứng minh rằngan khơng số phương với mọin >2

Lời giải

Với tốn có cơng thức truy hồi phức tạp trên, việc ta cần nghĩ đến sử dụng sai phân để đưa biểu thức ngắn gọn đơn giản để biện luận Qua đó, ta vận dụng tính chất số phương để chứng minh Cụ thể số phương có tận là0, 1, 4,5, 6,

Ta có: a2 =

Với n >2; ta có:

     

    

an+1 =

n

X

k=1

ak2+n

an = n−1

X

k=1

ak2+n−1

(52)

Ta suy dãy dãy tăng ngặt Bây ta có dãy

(

a2 =

an+1 =an2+an+ 1∀n >2

Vận dụng tính chất trên, ta chứng minhan khơng thể số phương theo phương pháp quy nạp đơn thuần, an số phương phải có tận số

0, 1, 4, 5, 6,

Ta thấy rõ số hạng dãy có tận 7xuất phát từ n = 3: a3 = 7;a4 = 57

Giả sử điều đến n =k; tức akcó tận Xét n=k+ 1, ta có:ak+1 =ak2+ak+

Do ta xét chữ số tận ak+1 nên ta xét chữ số tận số hạng

biểu thức

Ta có ak có tận 7nên ak2có tận Như chữ số tận

ak+1 =.b1b2 +c1c2 + =d1d2 + + = + =

Điều vớin =k+ 1, theo nguyên lý quy nạp ta suy raan có tận là7∀n>3 Áp dụng tính chất ta suy an số phương với mọin >3

Mặt khác a2 = khơng số phương

Vậy ta suy an khơng số phương với n>2

Bài tập 4.11 (Balkan 2002) Cho dãy số an thỏa mãn

(

a1 = 20; a2 = 30

an+2 = 3an+1−an∀n >1 Tìm n để A= 5an+1an+ số phương

Lời giải

Tương tự, ta tìm cơng thức tổng qt dãy số biện luận Đối với tìm giá trị n để biểu thức số phương dãy số Ta cần lưu ý đến tính chất dãy dãy (nếu có) đơn điệu, đồng dư

Dễ dàng chứng minh theo quy nạp công thức tổng quát an

an= 10

3 +√5

!n−1

+ 3−

5

(53)

Suy

A= 5an+1an+

⇒A= 5.102

"

3 +√5

!n

+ 3−

5

!n#

3 +√5

!n−1

+ 3−

5

!n−1 +

⇒A= 500

3 +√5

!2n−1

+ 3−

5

!2n−1

+

+

⇒A= 500

5 +

!4n−2

+

5−1

!4n−2

+

+

⇒A= 500

5 +

!2n−1

+

5−1

!2n−1 

2

+ 501

Xét dãy phụ sau:

(

x1 =

5; x2 =

xn+2 =

5xn+1−xn∀n>1 Dãy có cơng thức tổng qt

xn =

5 +

!n

+

5−1

!n

∀n ∈N

Ta có:

xn−xn−1 =

5 +

!n

+

5−1

!n

− √

5 +

!n−1

− √

5−1

!n−1

=

5 +

!n−2

+

5−1

!n−2

>0

Suy xn > xn−1∀n∈N Vậy xn dãy tăng ngặt

Tương tự 2, dãy có tính chất chứng minh quy nạp theo dãy song song là:

(

x2n=c

x2n−1 =d

5 (c;d∈N)

Chính mà ta suy ra: A= 500 d√52+ 501

Giả sử ∃n0 ∈N:A= 5an0+1an0 + số phương, ta có:

(50d)2 + 501 =k2 (k ∈N)⇒501 = (k−50d) (k+ 50d)

Mặt khác ta phân tích được501 = 3.167, mà

(

(54)

nên ta suy

(

k+ 50d = 501

k−50d= ⇒d=

Vậy ∃n0 ∈ N: xn0 =

5 Ta tính tốn x3 =

5 xn dãy tăng ngặt nên ta suy

n =

Vậy vớin = A = 5an+1an+ số phương

Bài tập 4.12 Cho dãy số tn thỏa mãn:

(

t1 = 9;t2 = 25; t3 = 81

tn+3 = 7tn+2−14tn+1+ 8tn∀n >1 a) Chứng minh rằng: =tn+1−2n+1

tn+1+ 22n∀n>1 số phương

b) Chứng minh T = + 2√12tn+ 1∈N số phương

c) Giả sử tồn dãy số xn cho xn.tn có lẻ số ước Chứng minh dãy xn gồm toàn số phương

Lời giải

> Câu a:

Ta thấy biểu thức cần chứng minh có dạng thức Điều gợi cho ta việc chứng minh tn dãy số phương

Bằng việc thử số hạng dãy, ta rút nhận xét dãy tn gồm toàn số phương Bây việc ta cần làm chứng minh dãytn gồm tồn số phương Thơng thường tốn chứng minh dãy số phương khơng số phương ta thường phải tìm công thức tổng quát dãy để đưa tốn số học thơng thường Nhưng dãy số ta chủ động biến đổi linh hoạt chúng để tạo nên lời giải hợp lý Điển hình với toán ta thấy việc chứng minhtn= 22

n

+ 2n+1+ 1 gồm tồn

số phương rõ ràng công việc không khả thi

Song ta dựa vào phương pháp dãy số chứng minh thẳng để dự đoán chứng minh tn bình phương đẳng thức

Nhờ phương pháp sai phân ta có đánh sau:

3 17 33

21 22 23 24

Hay

t1

t2

t3

t4

t5

21 22 23 24

Do đó, ta chứng minh theo quy nạp tn+1 = (2n+

tn)

2

(55)

Giả sử điều với n =k; tức tk = 2k−1 +

tk−1

2

Ta xét n=k+ 1; ta có:

tk+1 = 7tk−14tk−1+ 8tk−2 = 7tk−14tk−1+

p

tk−1−2k−2

2

⇒tk+1 = 7tk−6tk−1 −8.2k−1

p

tk−1+ 8.22(k−2) = 7tk−6

tk−2k−1

2

−8.2k−1 √tk−2k−1

+ 8.22(k−2)

⇒tk+1 =tk+ 2.2k

tk+ 22k =

tk+ 2k

2

Vậy theo nguyên lý quy nạp ta suy

tn+1 = 2n+

tn

2

∀n∈N

Mặt khác dot1 = số phương nên từ ta suy số hạng dãy số

phương

Vậy vn=tn+1−2n+1

tn+1+ 22n= (

tn+1−2n)

là số phương

> Câu b:

Việc chứng minh số phương giúp ta đưa toán dạng quen thuộc hơn: “NếuA = + 2√12a2+ 1 ∈

Nthì số phương"

Thật vậy, giả sử A∈N⇒√12tn+ =k(k∈N)⇒12tn= (k−1)(k+ 1) Do k lẻ nên ta suy rak = 2l+ (l ∈N) Từ ta suy ra: 3tn =l(l+ 1) Do (l;l+ 1) = vàtn số phương nên ta suy

(

l= 3a2

l+ =b2

(

l =b2

l+ = 3a2 (a;b ∈N)

* Trường hợp 1:

(

l=b2

l+ = 3a2 ⇒3a

2 = +b2

Mặt khác3 = 4.0 + 3nên từ ta suy b2+ 1 có ước dạng4k+ 3nên ta suy ra 1 . 3(vô lý).

Vậy trường hợp xảy

* Trường hợp 2:

(

l= 3a2 l+ =b2

Ta có:

A= + 2√12tn+ = + 2k = (k+ 1) = 4(t+ 1) = 4b2 Từ ta suy raA số phương

Vậy A= + 2√12n2+ 1 ∈

Nthì số phương > Câu c:

Một tính chất số phương: Nếu a số phương ln có lẻ số ước ngược lại (tính chất chứng minh nhờ việc sử dụng phép đếm)

Chứng minh: Giả sử a số phương, ta viết dạng tổng quát a

(56)

Số ước a số tích số số nguyên tố với lần thay đổi khác số mũ αi,∀i= 1;n Chính mà theo quy tắc nhân ta đếm a có

(2α1+ 1) (2α2+ 1) (2α3+ 1) (2αn+ 1) số ước hiển nhiên tổng số ước số lẻ

Ngược lại giả sử số a có lẻ số ước, ta có dạng tổng quát a

a=p1r1p2r2p3r3 pnrn Theo quy tắc nhân ta đếm acó

(r1+ 1) (r2+ 1) (r3+ 1) (rn+ 1) Do a có lẻ số ước nên ri+ 1, ∀i= 1;n phải số lẻ, suy

ri = 2k (k ∈N), ∀i= 1;n Vây a số phương

Tính chất chứng minh hồn tất Quay lại tốn sử dụng tính chất để tìm lời giải thích hợp cho tốn

Ta có: xn.tn có lẻ số ước Áp dụng tính chất ta suy raxn.tn số phương Mặt khác

tn dãy số phương ⇒xn dãy số phương

Bài tập 4.13 Cho dãy (un)thỏa mãn

(

u0 = 2; u1 =

un+1= 5un−6un−1, ∀n>1

Xét hàm số

f(x) = x

3

3 −

5x2u

n

2 + 6un

2+ 6n

x+C (C∈R)

có đạo hàm x0 ∈(a;b) với a;b ∈N

Chứng minh rằngf(x)có điểm cực đại cực tiểu có hồnh độ ln số ngun Lời giải

Đầu tiên, ta có nhận xét sau: điểm cực trị hàm số thuộc phương trình

f0(x) =x2−5xun+ 6un2+ 6n=

Như điều kiện để nghiệm phương trình số nguyên thì∆ =un2−4.6nphải số phương với n>1 Như ta đưa toán việc chứng minhvn=un2−4.6n số phương với n >1

Trong toán này, ta chứng minh phương pháp dùng tam thức bậc hai để giải toán nhanh gọn (nghĩa ta đưa toán dạng X2+αXu

n+β+ 6n = số 6n xuất cách không tự nhiên số hạng u

(57)

Như dãy số có dạng un+12+αunun+2+ 6n+β = (do dạng ta tách un+2

theo cơng thức truy hồi đưa phương trình dạng nêu trên) Ngồi ta chứng minh biểu thức thông thường theo quy nạp biểu thức đoán

Như diễn đạt trên, ta có:

un+12−5αun+1un+ 6αun2+ 6n+β =

Xem biểu thức phương trình bậc theo tham số un ẩn un+1, ta

∆ = 25α2un2−24αun2−4.6n−4β =α2un2−4.6n−4β Đồng hệ số với biểu thức cần chứng minh ta α= 1; β = Như ta chứng minh theo quy nạp

unun+2−un+12 = 6n,∀n∈N

Thật điều với n= do2.13−52 = 60 = n= do5.35−132 = Giả sử điều đến n =k, tức ukuk+2−uk+12 = 6k

Xét n=k+ 1, ta có:

uk+1uk+3−uk+22 =uk+1(5uk+2−6uk+1)−uk+22 = 5uk+1uk+2−6uk+12−uk+22

⇒uk+1uk+3−uk+22 = 5uk+1uk+2+ 6k+1−6ukuk+2−uk+22

=uk+2(5uk+1−6uk) + 6k+1−uk+22 = 6k+1 (do uk+2 = 5uk+1−6uk) Vậy điều với n=k+ 1, theo nguyên lý quy nạp ta chứng minh

unun+2−un+12 = 6n,∀n∈N

Do đó, ta có:

u2n+1−unun+2+ 6n= ⇒u2n+1−5unun+1+ 6u2n+

n = (∗) Xem (*) phương trình bậc theo tham sốun ẩnun+1, ta có:

X2−5unX+ 6un2+ 6n =

Do dãy số dãy số nguyên nên ta suy X phải số nguyên Như

∆ = 25un2−24u2n−4.6 n=u

n2−4.6n=vn phải số phương

Xét phương trình tổng qt

x2 −5unx+ 6un2+ 6n = (x∈R) Phương trình có nghiệm ∆ =vn =un2−4.6n >0

Màvn số phương (chứng minh trên) nên phương trình ln có nghiệm

∆∈N Như nghiệm phương trình

x= 5un±

un2−4.6n

(58)

+Với un lẻ un chẵn, 5un±

un2−4.6n số chẵn nên hiển nhiên chia hết cho Cho nên x số nguyên

Mặt khác

Z

x2−5xun+ 6n+ 6un2

= x

3

3 −

5x2u

n

2 + 6un

2+ 6n

x+C (C ∈R) = f(x)

Mặt khác hàm số đạt cực đại cực tiểu tạia, b, x0 nên từ ta suy điều phải

chứng minh

Bài tập 4.14 Xét hàm số f(t) =t+√t Dãy số (an) xác định

(

a0 =m >2, (m∈N)

an+1 =f(an), ∀n= 0,1,2 dãy (tb) :tb =

b√2 ∀b ∈N Chứng minh tồn tạin0; b0 :

an0.tb0 ∈Z Lời giải

Với điều kiện đề bài, ta nhận tích số phải số phương Như

an0;tb0 số phương, an0 =tb0, chỉan0.tb0 số phương Việc tồn hoặcan0 =tb0, chỉan0.tb0 số phương cơng đoạn tính toán vất vả hàm phần nguyên Như ta chứng minh tồn vô số số hạng số phương để khẳng định tồn biến độc lập mà không cần cụ thể điều kiện biến việc tính tốn Ta chứng minh tốn theo tính chất hàm phần ngun áp dụng khai triển Newton dãy số độ lệch hạng tử dãy

Xét dãy (an), ta có: m < f(m) < f (f(m)) < nên dãy dãy tăng ngặt Vậy ta chia làm hai trường hợp sau:

> Trường hợp 1: Nếu m khơng phải số phương

Gọi t2 số phương lớn khơng vượt q m=a0 Theo cách xác định ta suy

đượcd = [√m]

Đặt m=d2+k Ta có d2 < m6(d+ 1)2 ⇒0< k62d+ * a) Xét 0< k < d+ 1.Theo cách xác định dãy ta có:

d2 < a1 =f(a0) =m+

m

=d2+k+d <(d+ 1)2

⇒d <pf(a0)<(d+ 1)⇒d=

hp

f(a0)

i

= [√a1]

Ta có:

a2 =f(a1) =a1+ [

a1] =m+

m+ [√a1] =m+ 2d= (d+ 1)2+k−1

Như ta thấy số phương lớn khơng vượt a2 là(d+ 1)2 độ lệch chúng

(59)

Áp dụng tương tự với phần tử bắt đầu a1 Quá trình lặp lại độ lệch giảm dần

về nên sau hữu hạn bước ta gặp số phương * b) Xét d+ 16k 62d+ ta có:

a1 =f(m) = m+

m=d2+k+d= (d+ 1)2+k−d−1

Do 06k−d−1< d+ nên ta có:

a1 =

p

f(m) =

q

(d+ 1)2+k−d−1⇒[√a1] =

hp

f(m)i=d+

Mặt khác, ta lại có:

a2 =f(a1) =f(m) +

hp

f(m)i =d2+k+ 2d+ = (d+ 1)2+k

Vậy (d+ 1)2 số phương lớn khơng vượt qa2 k độ lệch Do

06k < d+ nên ta quay lại trường hợp a) với phần tử bắt đầu dãy a2

Như vậy, ta chứng minh sau hữu hạn bước dãy (an) gặp phần tử số phương

> Trường hợp 2: Nếu m số phương dãy (an) có vơ hạn số phương tồn tạin0 ∈N:an0khơng số phương Khi ta quay lại trường hợp sau hữu hạn bước ta gặp lại phần tử dãy số phương

Do q trình lặp lại vơ hạn lần nên ta suy dãy có vơ hạn số phương Xét dãy (tb) Theo khai triển Newton, ta có:

  

 

2 + 1l=zl

2 +yl

2−1l =zl

2−yl

khi l lẻ

⇒(2−1)l = 2zl2−yl2 ⇒1 +yl2 = 2zl2

⇒yl4 +yl2 =

ylzl

22

⇒pyl4+yl2 =ylzl

2

Mặt khác:

yl2 <

p

yl4+yl2 < yl2+

⇒hpyl4 +yl2

i

=yl2 Lại có

hp

yl4+yl2

i

=hylzl

2i ⇒hylzl

2i =yl2 Do ylzl

2 dãy dãy tb nên ta suy dãy có vơ hạn số phương Như

∃b0, n0 ∈N:

p

an0.tb0 ∈Z⇒an0+1 =f(an0) =d

2+d=d(d+ 1)

(60)(61)

Một số hàm số học ứng dụng

Lê Phúc Lữ

Hàm số học f hàm nhận đối số giá trị tập rời rạc tập số nguyên, thông thường ta xét f : N → N Các hàm số học quen thuộc gặp nhiều ứng dụng như: hàm phần nguyên, hàm phần lẻ, hàm tổng chữ số, hàm Euler Còn hàm tổng ước hàm số ước rõ ràng giới thiệu nhiều phần ứng dụng Bài viết lược dịch từ chương Elementary Number Theory and Its Application tác giả Kenneth Rosen Mong với số lượng tập phong phú, tiếp cận tốt hai hàm số học “quen mà lạ”

Hàm tổng ước số số ước số

Kiến thức cần nhớ

Định nghĩa tính chất Với số nguyên dương n, kí hiệu tổng ước dương n (kể cản) τ(n) số ước dương n (kể n)

Ta xét biểu diễn n

n=pa1 p

a2 p

a3 p

ak k

với p1, p2, p3, , pk số nguyên tố a1, a2, a3, , ak số nguyên dương Từ ta quy ước biểu diễn n dạng

Khi ta có:

σ(n) = p

a1+1 −1

p1−1

pa2+1 −1

p2−1

pa3+1 −1

p3−1

p

ak+1

k −1

pk−1

τ(n) = (a1+ 1)(a2+ 1)(a3+ 1) (ak+ 1) Chú ý ta kí hiệu σ(n) =P

d|n

d τ(n) =P

d|n

1 Chứng minh

Ta thấy sốd=pb1 p

b2 p

b3 p

bk

k ước củankhi khi06bi 6aivớii= 1,2,3, , k Như giá trị bi có ai+ cách chọn thế, theo nguyên lí nhân, số ước

1Sinh viên Đại học FPT

(62)

n Tiếp theo, ta xét biểu diễn k Y i=1 X j=1

pji

!

= +p1+p21+ +p

a1

1 +p2+p22 + +p

a2

+pk+p2k+ +p ak k

Dễ thấy khai triển có tất (a1+ 1)(a2+ 1)(a3+ 1) (ak+ 1) số hạng ước phân biệt n Do đó, đại lượng tổng ước n thu gọn thành

σ(n) = p

a1+1 −1

p1−1

pa2+1 −1

p2−1

pa3+1 −1

p3−1

p

ak+1

k −1

pk−1

Nhận xét Nếu f hàm nhân tính, nghĩa f(m)f(n) = f(mn) với (m, n) = ta có

F(n) = P

d|n

f(d) hàm nhân tính

Thật vậy, giả sử m, n số nguyên dương nguyên tố F(mn) = P

d|(mn)

f(d) Rõ ràng ta viết d=d1d2 cách cho d1 ước m d2 ước củan

với (d1, d2) = Khi , ta có

F(mn) = X

d|(mn)

f(d) = X

d1|m, d2|n

f(d1d2) =

X

d1|m, d2|n

f(d1)f(d2) =

X

d1|m

f(d1)

X

d2|n

f(d2) =F(m)F(n)

Nhận xét chứng minh Từ đây, thay f(x) =x f(x) = 1, ta suy hàm σ(n) τ(n)đều hàm nhân tính

Ví dụ áp dụng

Ví dụ

1 Tìm tất số nguyên dương n cho σ(n)lần lượt 12,18,24,48,52,84?

2 Tìm số nguyên dương n nhỏ cho τ(n)lần lượt 1,2,3,6,14,100?

Lời giải

1 Để làm này, ta cần chọn số nguyên tố số mũ thích hợp cho

1 +p1+p21 + +p

a1

1 +p2+p22+ +p

a2

+pk+p2k+ +p ak k

=σ(n)

Ta liệt kê số có dạng +pk+p2k+ +p ak

k từ nhỏ đến lớn:

1 + 2,1 + 3,1 + 5,1 + + 22,1 + 7,1 + + 32,1 + + 22+ 23,

Với σ(n) = 12, ta thấy viết thành 12 = (1 + 2)(1 + 3) số cần tìm n =

Các số lại thực tương tự

2 Ở này, ta chọn số mũ thích hợp trước tiếp đến chọn số nguyên tố nhỏ giá trị n nhỏ tốt Chẳng hạn vớiτ(n) = 14, ta chọn số mũ 13 ứng với ước nguyên tố 2, tức n = 213 Tuy nhiên, ta làm cho giá trị nhỏ hơn

bằng cách viết14 = 2·7, ứng với số mũ 6, ta chọn sốn= 3·26 Đây số nhỏ cần

tìm

(63)

Ví dụ

1 Chứng minh τ(n) số lẻ khin số phương Với giá trị n σ(n)là số lẻ?

Lời giải

1 Ta thấy τ(n) = (a1+ 1)(a2+ 1)(a3+ 1) (ak+ 1) nên τ(n) lẻ tất số mũ phải số chẵn, nghĩa số phương

2 Ta cần chọn ước nguyên tố số mũ thích hợp để σ(n) số lẻ Ta có

σ(n) = +p1+p21+ +p

a1

1 +p2+p22+ +p

a2

+pk+p2k+ +p ak k

nên có ước nguyên tố thỏa mãn, n có ước ngun tố lẻ tổng trên, chúng phải xuất chẵn lần Do đó, số cần tìm có dạng n = 2km2 với m số phương lẻ

Ví dụ Một mật hợp lệ có độ dài n, n >6 gồm phần: kí tự đầu chữ số 1, n−4 kí tự sau 26 kí tự bảng chữ tiếng Anh khơng chứa tồn chữ ‘A’ Chẳng hạn: 0100XYZ mật có kí tự, hợp lệ 0111AAA hay 0123ABC mật có kí tự không hợp lệ

a Hỏi tổng số mật hợp lệ bao nhiêu? Đặt số lượng sn

b Gọi số tất cách biểu diễnsn thành tổng số nguyên dương (có lượng số hạng tùy ý gồm số hạng) tn Chứng minh tn chia hết cho 10

Lời giải Câu a:

Xét mật có dạng X1X2X3 Xn với

 

Xi ∈ {0; 1} ∀i= 1,4

Xk ∈ {A, B, C, , Z} ∀k >

Mỗi số X1, X2, X3, X4 có cách chọn nên có tất 2×2×2×2 = 16 cách chọn kí tự đầu

Mỗi số Xk,4 < k n có 26 cách chọn nên có tất 26n−4 cách chọn trừ trường hợp tồn A nên có tổng cộng26n−4−1cách chọn n−4kí tự sau

Do đó, tổng số mật hợp lệ có sn= 16(26n−4−1) Câu b:

Dễ dàng thấy tn số ước sn Ta có (16,26n−4−1) = nên viếtsn dạng sn = 24×p với p số nguyên dương lẻ

Do n>6 nên n−4>2và 26n−4 chia hết cho 4, tức 26n−4−1 có dạng 4k+ 3, khơng thể số phương, tức có số ước chẵn

Thêm vào đó, + = nên theo cơng thức tính số ước số nguyên dương tn vừa chia hết cho vừa chia hết phải chia hết cho 10

(64)

Ví dụ

a) Cho n số tự nhiên thỏa mãn n+ chia hết cho 24 Chứng minh tổng ước dương n (kể n) chia hết cho 24

b) Xét số nguyên

A= 20102011−20112010−20521994

Hãy chứng minh A hợp số dương tổng ước số dương A chia hết cho 24 Lời giải

a) Gọi d(n) tổng ước dương củan Trước hết, ta chứng minh d(n) chia hết cho

Thật vậy, gọi a ước n na ước n xét 0< a < √n a;na

đôi khác (do n+ chia hết cho 24 nên n chia dư khơng thể số phương)

Vì n chia dư nên hai sốa,na có số chia dư số chia dư 2; suy tổng chúng phải chia hết cho Do

d(n) = X

0<a<√n

a+n

a

Tương tự, ta thấy n+ chia hết n chia dư Don số lẻ nên chia có số dư 1,3,5,7; đó, dễ dàng thấy hai số a,na có số chia dư 1, số chia dư số chia dư 3, số chia dư

Dễ thấy tổng hai ước chia hết cho Từ suy d(n) chia hết cho

Kết hợp hai điều lại, ta thấy tổng ước dương n chia hết cho 24

b) Trước hết, ta chứng minh n hợp số dương Thật vậy, ta chứng minh

x > y >3thì

yx > xy (∗)

Bất đẳng thức tương đương với

xlny > ylnx⇔ lny

y >

lnx x

Hàm số f(t) = lntt, t > cóf0(t) = 1−t2lnt <0 nên hàm nghịch biến, suy f(x)< f(y) hay (*)

Cũng cách dùng hàm số, ta chứng minh với n đủ lớn < a < n có đánh giá

nn+a

(n+a)n >2

Do đó20102011 >2.20112010 >20112010+ 20521994 nên A >0 Xét modun thì

(65)

24 ≡1(mod5)⇒21994 = 24498 ≡4(mod5)

Suy A chia hết cho hay A hợp số

Tiếp theo, ta chứng minh A+ 1chia hết cho 24 Thật vậy, 2010,2052 chia hết

A+ ≡ − (−1)2010

+ = 0(mod3)

Hơn 2052 chia hết cho

A+ 1≡ −32010+ ≡ − 321005+ 1≡ −11005+ = 0(mod8)

Suy A+ chia hết cho chia hết A+ chia hết cho 24

Theo kết câu a, ta có tổng ước dương A chia hết cho 24, suy đpcm

Bài tập có hướng dẫn, gợi ý

Bài tập 5.1 Chứng minh phương trình τ(n) = k có vơ số nghiệm ngun dương n với k, cịn phương trình σ(n) = k có hữu hạn nghiệm nguyên dương n

Gợi ý

Ta thấy giá trị σ(n) có tham gia số nguyên tố, giá trị τ(n) khơng (điều có nghĩa ta chọn giá trị ước nguyên tố số n lớn tùy ý)

Bài tập 5.2 Tích tất ước n bao nhiêu? Chứng minh hàm tích ước n đơn ánh tập hợp số nguyên dương

Gợi ý

Chú ý ước củancó thể chia thành cặp có dạng d;ndvà nhân chúng lại, ta giá trị n Kết nτ(n)/2.

Cịn việc chứng minh tính đơn ánh sử dụng phân tích thành thừa số nguyên tố

Bài tập 5.3 Tìm tất số nguyên dương n cho σ(n) +φ(n) = 2n, kí hiệu

φ(n)là số số nguyên dương không vượt n nguyên tố với n

Gợi ý

Có thể sử dụng trực tiếp công thức hàm số dùng bất đẳng thức chứng minh σ(n) +φ(n)>2n dùng lập luận trực tiếp dựa theo ý nghĩa hàm số

Bài tập 5.4

a Chứng minh số cặp có thứ tự số có BCNN n τ(n2)

b Chứng minh với số nguyên dương n

τ(2n−1)>τ(n)

(66)

Gợi ý

a Ta tính số cách chọn (a, b) mà [a, b] = n (2a1 + 1)(2a2 + 1) (2ak+ 1) việc lập luận số mũ tương tự chứng minh cơng thức tính τ(n)

b Chú ý (2ab−1) .(2a−1) với a, b∈

Z+ nên dễ thấy số ước 2n−1 nhiều n c Giả sử n =ab với 1< a6 b < n σ(n) > +a+b+n > +√n+n > n+√n Chiều ngược lại chứng minh dễ dàng việc kiểm tra số nguyên tố

Bài tập 5.5

1 Chứng minh

n

X

i=1

τ(i) =

[√n]

X

i=1

hn

i

i

−√

n2

Từ tính tổng

100

P

i=1

τ(i)

2 Cho a, blà số nguyên dương Chứng minh

max

σ(a)

a , σ(b)

b

6 σ(ab)

ab

σ(a)σ(b)

ab

3 Chứng minh a, b số nguyên dương thìσ(a)σ(b) = X

d|(a,b)

d·σ

ab d2

Gợi ý

a Ta sử dụng phương pháp quy nạp Chú ý bước quy nạp, ta kiểm tra trường hợp n số phương không Tổng

100

P

i=1

τ(i) = 482

b Ta chia thành bất đẳng thứcbσ(a)6σ(ab)và σ(ab)6σ(a)σ(b)rồi sử dụng trực tiếp công thức hàm σ(n)

c Đặt a = Q

paii , b = Q

pbii ci = min(ai, bi) với i = 1,2,3, , k Trước hết, ta chứng minh

Y

pi ci

X

j=0

pjiσ(paii +bi−2j) = X

d|(a,b)

ab d2

Sau xem xét tổng có dạng c

P

j=0

(pa+b−j+pa+b−j−1+ +pj)với c= min(a, b) ý số lần xuất thừa số nguyên tố trongσ(a)σ(b)

Bài tập tự giải

Bài tập 5.6

(67)

Bài tập 5.7

a Hỏi số nguyên dương có ước nguyên dương? b Hỏi số nguyên dương có ước nguyên dương? c Hỏi số nguyên dương có ước nguyên dương?

Bài tập 5.8 Cho số nguyên dương a Xét dãy số (un) xác định

(

u1 =a,

un+1 =τ(un), n= 1,2,3, Chứng minh với n đủ lớn uN = 2,∀N >n

Bài tập 5.9 Sốn gọi “highly composite” τ(m)< τ(n) với mọi16m < n Chứng minh với k ∈ Z+, tồn số HC n thỏa mãn k 6n < 2k Từ suy có

vô hạn số HC đánh giá chặn cho số HC thứ k

2 Chứng minh n số HC tồn số nguyên dương k cho ta có biểu diễn n = 2a13a25a3 pak

k với pk số nguyên tố thứ k a1 >a2 > >ak >1 Tìm tất số HC có dạng n = 2a3b với a, blà số nguyên dương.

Biết số square-free số có dạng n = p1p2p3 pk với p1, p2, p3, , pk số nguyên tố Hãy trả lời câu hỏi sau

Bài tập 5.10

a Hỏi số sau đây, số số square-free?

A.44 B.50 C.10000000 D.95

b Một số square-free có số ước làA HỏiA nhận giá trị nào?

A.25 B.30 C.1024 D.99

c Cho tập hợp B = {2,3,5,7} Hỏi có tất số square-free có ước nguyên tố thuộc tập B?

A.15 B.16 C.17 D.14

Bài tập 5.11 Cho n số nguyên dương Chứng minh đẳng thức sau: a

 X

d|n

τ(n)

2

=X

d|n

(τ(d))3 b τ(n2) =X

d|n

2ω(d) với ω(n) số ước nguyên tố n

c X d|n

nσ(d)

d =

X

d|n

dτ(d)

(68)

Một số hàm số khác

Hàm phần nguyên

Các kiến thức cần nhớ

Các tính chất phần nguyên:

• [x] =x x số nguyên

• [x+n] = [x] +n với n số nguyên

• [x+y]6[x] + [y] với số thực x, y

• [x] + [y] =x+y−1với x, y số không nguyên x+y nguyên

• Số số nguyên dương không vượt n chia hết chok lànk

• Khai triển Legendre: số mũ số nguyên tố p khai triển số nguyên dương n!

thành thừa số nguyên tố

+∞ X

i=1

n pi

=

n p

+

n p2

+

n p3

+

Các tập áp dụng

Bài tập 5.13 Hỏi có số nguyên không vượt 2013 chia hết cho hoặc 5?

Bài tập 5.14 Chứng minh số số nguyên không vượt n không chia hết cho

n+hn

i

−hn

2

i

+hn

i

Bài tập 5.15 Chứng minh với số nguyên dương x, y (xx+!yy!)! số nguyên

Bài tập 5.16 Phần lẻ{x}chính đại lượng tính bằngx−[x] Chứng minh rằng06{x}<1

và với số thực x, y {x+y}6{x}+{y}

Bài tập 5.17 Chox, y, z số thực thỏa mãn{x}+{y}+{z}= Tính giá trị biểu thức

P = [x+y+z]−([x] + [y] + [z])

Bài tập 5.18 Rút gọn biểu thức sau:

A =

21

3

+

22

3

+

23

3

+ +

2100

3

(69)

Hàm tổng chữ số

Các kiến thức cần nhớ

Kí hiệuS(n)là tổng chữ số n Ta có tính chất:

• n−S(n) chia hết cho

• S(m+n)6S(m) +S(n) với m, n số nguyên dương

• S(mn)6S(m)S(n) với m, n số nguyên dương

• S(n) = n−9

+∞ X

k=1

h n

10k

i

Bài tập áp dụng

Bài tập 5.19 Tìm số nguyên dương n nhỏ cho S(n), S(n+ 1) chia hết cho Bài tập 5.20 Khi viết44444444 hệ thập phân tổng chữ số làA GọiB tổng chữ số A Tính tổng chữ số củaB

Bài tập 5.21 Chứng minh với số ngun dương n ta có bất đẳng thức: a)S(2n)6S(n)65S(2n)

b)S(5n)6S(n)62S(5n)

Bài tập 5.22 Người ta gọi số biểu diễn thành tổng a+b với S(a) =S(b) số đẹp

a Chứng minh số 999,2999 số đẹp tồn vô số số không số đẹp b Chứng minh tất số thỏa mãn999 < k <2999 số đẹp

Hàm Euler

Kiến thức cần nhớ

Với số ngun dươngn,kí hiệuφ(n)là số số ngun dương khơng vượt quánvà nguyên tố với n Đây gọi hàm Euler

Ta có kết quen thuộc:

• φ(p) =p−1 với p số nguyên tố

• φ(pk) =pk−1(p−1) với p là số nguyên tố.

• φ(mn) =φ(m)φ(n)với (m, n) = Trong trường hợp tổng quát, ta có

φ(pa1 p

a2 p

a3 p

ak k ) =n

1−

p1

1−

p2

1−

p3

1−

pk

(70)

Bài tập tổng hợp

Bài tập 5.23

1 Cho n số nguyên dương lớn Chứng minh rằngφ(n) chẵn Với giá trị n φ(n) = n

2? φ(n)|n? φ(3n) = 3φ(n)? φ(n) chia hết cho 4?

3 Chứng minh n có k ước nguyên tố lẻ phân biệt φ(n)chia hết cho 2k. Chứng minh m, n∈Z+ và m|n thì φ(m)|φ(n).

5 Chứng minh hàm f(n) = φ(nn) hàm nhân tính đầy đủ Bài tập 5.24

1 Giả sử m, n∈Z+ và (m, n) =p với p nguyên tố Chứng minh φ(mn) = pφ(m)φ(n)

p−1

2 Chứng minh

φ(mk) =mk−1φ(m)

với mọim, k ∈Z+.

3 Cho a, b∈Z+, chứng minh rằng

φ(ab) = (a, b)φ(a)φ(b)

φ((a, b)),

từ suy φ(ab) =φ(a)φ(b) với (a, b)>1 Bài tập 5.25

1 Chứng minh phương trình φ(n) = k với k ∈ Z+ có nghiệm nhất n thì n chia

hết cho 36

2 Chứng minh phương trình φ(n) =k với k ∈Z+ có hữu hạn nghiệm n.

3 Chứng minh p số nguyên tố, 2ap+ 1 là hợp số với a = 1,2,3, , r và p không phải số nguyên tố Fermat

φ(n) = 2rp

vơ nghiệm

4 Chứng minh tồn vô hạn số nguyên dương k cho phương trình

φ(n) = k

có nghiệm nguyên dương n

Gợi ý: xét số nguyên dương có dạng k= 2·36j+1, j = 1,2,3,

Bài tập 5.26

1 Chứng minh n 6= 2, n6= φ(n)>√n

2 Chứng minh n hợp số φ(n)6n−√n Tìm số nguyên dương n nhỏ thỏa mãn φ(n)>105

(71)

Bài tập 5.28 Cho số nguyên dương a Xét dãy số (un) xác định

(

u1 =a,

un+1 =φ(un), n = 1,2,3, Chứng minh với n đủ lớn uN = 1,∀N >n

Bài tập 5.29

1 Chứng minh X d|n

φ(d) = n với số nguyên dương n

2 Chứng minh X

16a6n,(a,n)=1

a= nφ(n)

2

3 Giả sử p > số nguyên dương p−1, p+ hai số nguyên tố sinh đôi Chứng minh 3φ(p)6p

(72)(73)

Thặng dư bình phương

Nguyễn Huy Hồng, Trần Hy Đơng

Thặng dư bình phương cơng cụ mạnh để giải tốn số học Thặng dư bình phương đơn giản, dễ hiểu lại hữu dụng Tuy vậy, độc giả cần có tảng kiến thức tốt (như cấp số, định lý Fermat, ) để sử dụng thành thục

Tính chất thặng dư bình phương kí hiệu Leg-endre

Định nghĩa 6.0.1

Giả sử p số nguyên tố lẻ Khi số nguyên a gọi số phương mod p

(a, p) = phương trình đồng dưx2 ≡a(modp)có nghiệm.

Ví dụ:-1 số phương mod ( 72 ≡ −1(mod5) )

Định lý 6.0.1

Nếu p số nguyên tố lẻ số1,2,3, , p−1có p−21 số phương mod p

Phần chứng minh hai định lý xin dành cho bạn đọc

Định nghĩa 6.0.2

Cho a∈Z , p số nguyên tố lẻ Ký hiệu Legendre

a p

được định nghĩa sau: Nếu (a, p) = a số bình phương mod p

a p

=

  

 

1

−1

Nếu (a, p) = a khơng số bình phương mod p

(a, p)6=

1Lớp chun Tốn trường Phổ Thông Năng Khiếu trường THPT Chuyên Lê Hồng Phong

(74)

Ví dụ:Ta thấy rằng: 11 = 11 = 11 = 11 = 11 = 11 = 11 = 11 = 11 = 10 11

=−1

Định lý 6.0.2

Giả sử p số nguyên tố lẻ , (a, p) = Khi :

a p

≡ap−21 (modp)

Chứng minh: > Trường hợp 1:

a p

=

Khi phương trình đồng dư x2 ≡a(modp)có nghiệm x

0 Theo định lý Fermat nhỏ, ta có :

ap−21 ≡(x2 0)

p−1

2 =xp−1

0 ≡a(modp)(đpcm)

> Trường hợp 2:

a p

=−1

Khi phương trình đồng dưx2 ≡a( modp)vơ nghiệm Với mỗii từ đếnp−1, tồn duy

nhất j,16j 6p−1sao cho i.j ≡a(modp)

Rõ ràng i6=j nên nhóm số từ đến p−1thành p−21 cặp , cho tích cặp đồng dư a mod p Nhân tất số 1,2,3, , p−1, ta :

(p−1)! ≡ap−21(modp)

Theo định lý Wilson , (p−1)! ≡ −1(modp), định lý chứng minh Định lý 6.0.3

Giả sử p số nguyên tố lẻ , a b số ngun khơng chia hết cho p Khi đó: 1/a ≡b(modp)⇒

a p = b p 2/ a p b p = ab p / a2 p

=

Bạn đọc dễ chứng minh định lý định lý Định lý 6.0.4

Với số nguyênp > 2, ta có

1

p

≡(−1)p−21

Từ suy phương trình đồng dư x2 ≡ −1(modp)có nghiệm p=

p≡1(mod4)

(75)

Định lý 6.0.5

Giả sử(x;y) = 1,a, b, c số nguyênplà ước ngun tố củaax2+bxy+cy2,p khơng ước củaabc D=b2−4aclà thặng dư bậc mod p

Đặc biệt p ước củax2−Dy2và(x, y) = 1thì D thặng dư bậc mod p Chứng minh:

Dễ biến đổi

p(2ax+by)

2

−Dy2

Giả sử p|y , p|2ax+by ⇒p|2ax

Mà (a, p) = nên p|x Vậy (x, y)>1 ( Vô lý )

Do đó(p, y) = nên tồn y0sao cho yy0 ≡1(modp) Suy

(2axy0+byy0)2 ≡Dyy0 ≡D(modp)

Vậy D thặng dư phương mod p

Định lý 6.0.6 (Bổ đề Gauss)

Giả sử p số nguyên tố lẻ , a số nguyên không chia hết cho p Nếu số thặng dư bé số nguyên a,2a,3a, ,p−21a có s thặng dư lớn p2 thì:

a p

= (−1)shay

a p

= (−1)

p−1

P

k=1[ 2ka

p ]

Chứng minh:

Cho a số nguyên , p số nguyên tố cho (a, p) = Với k∈

1; 2; 3; ;p−21 tồn rk ∈

±1;±2;±3; ;±p−21 cho

ka≡rk(modp) Dễ thấy không tồn hai rk có trị tuyệt đối ,

|r1|,|r2|,|r3|, ,

rp−21

là hoán vị

1; 2; 3; ;p−21 Cho k chạy từ 1→ p−1

2 nhân tất vế lại với ta :

ap−21 ≡

r1r2r3 rp−1

1.2.3 (p−21) =

r1r2r3 rp−1

|r1| |r2| |r3|

rp−21

(modp)

Đặt bk =

rk

|rk|

;bk =±1.Ta có

ap−21 ≡b1 bk(modp)

bk=−1khi phần dư lấy ka chia cho p lớn hơnp0 tức ka=pq+r

r > p−21

2ka

p = 2p+

2r p ⇔ 2ka p

= 2p+ =

ka p

+ 1⇔bk = (−1)[ 2ka

(76)

Vậy ap−21 ≡

a p

= (−1)

p−1

P

k=1[ 2ka

p ]

(đpcm)

Từ định lý ta thu hệ sau Định lý 6.0.7

a/

2

p

= (−1)[p+14 ]

b/-2 thặng dư bậc mod p p≡1; 3(mod8)

c/ -3 thặng dư bậc mod p p≡1(mod6)

d/ thặng dư bậc mod p p≡ ±1(mod12)

e/ thặng dư bậc mod p p≡ ±1(mod10)

Định lý 6.0.8 Luật tương hỗ

Nếu p,q số nguyên tố lẻ p khác q

p q q p

= (−1)(p−21)( q−1

2 )

Bài tập ví dụ

Bài tập 6.1 Chứng minh với số nguyên tố p= 4k+ 3, nếux2 +y2 p

 

x p y p

Lời giải Giả sử x, y p⇒

x p , y p

= 0, ta có

y2

p

= Nhưng theo định lý 4b, 4c 5, ta có

−y2

p = −1 p y2 p = −1 p

=−1(vơ lí!)

Do ta có

 

x p y p

Bài tập 6.2 Chứng minh với số nguyên tốp= 3k+2thì m2+mn+n2 p⇔

 

m p n p

Lời giải Giả sử m, n6 p⇒

m p , n p

= Ta có

m2+mn+n2 p⇒4 m2+mn+n2 p⇒(2m+n)2+3n2 p⇒

3n2

p

= ⇒

3

p

=

Theo định lý 8c, ta suy điều vơ lí, m2+mn+n2 p⇔

 

m p n p

(77)

một ứng dụng thặng dư bình phương

Các định nghĩa thặng dư bình phương đơi sử dụng để chứng minh số toán thú vị Chúng ta cần áp dụng tính chất để lập luận thể sức mạnh thặng dư bình phương

Bài tập 6.3 Chứng minh với số nguyên tố lẻ p:

∃a∈Z+:

     a p

=−1

a <1 +√p

Lời giải

Gọi a số tự nhiên nhỏ không thặng dư bậc hai mod p Đặt b=p

a

+ ⇒0< ab−p < ado hiệu ab−p thặng dư bậc mod p Vậy

1 =

ab−p p = ab p = a p b p =− b p

Suy b không thặng dư bậc mod p Do

a6b < p

a + ⇒a <

p+

Bài tập 6.4 (Korea Final 2000) Cho số nguyên tốp= 4k+ 1, tính p−1

X

x=1

2x2

p −2 x2 p Lời giải

Đầu tiên, ta có nhận xét:

[2x]−2 [x]61∀x∈R

Đẳng thức xảy {x}>

2

Thế nên, nhiệm vụ tìm số đồng dư số phương chia cho số nguyên tố p= 4k+ cho lớn p−1

2

Theo tính chất số thặng dư bình phương, ta có p−1

2 số phương mod p

Bởi p= 4k+ 1⇒

1

p

= 1⇒

a2

p

= ⇒

a p = − a p =

p−a p

Như ,ta có p−41 số phương modpkhơng lớn hơn p−1

2

p−1

4 số

chính phương mod plớn p−21 Do

p−1

X

x=1

2x2 p −2 x2 p

= p−1

(78)

   

  

∀i, j ∈N

16i, j 6p−1

i+j =p

thì i2 ≡j2(modp).

Bài tập 6.5 (Iran TST 2004)Cho trước số nguyên tốp số nguyên dươngk, chứng minh tồn số nguyên dương n cho

n p

=

n+k p

Lời giải

Bài toán tương đương với việc chứng minh tồn n cho

n(n+k)

p

= Ta giả sử điều ngược lại, tức giả sử tồn 16 k 6 p−1 cho

n(n+k)

p

= −1∀1 6

n 6p−1

Vì với số ngun tố p bất kì, có p−21 số khơng phương mod p Do

f(n, k, p) = n(n+k) modpnhận nhiều p−21 giá trị phân biệt tồn số phương mod p

Vậy tập giá trị f(n, k, p) nhận không p−21phần tử, theo ngun lý Dirichlet, ta có ba số x, y, z nguyên phân biệt cho

(

16x, y, z 6p−1

f(x, k, p) =f(y, k, p) = f(z, k, p)

Tức

x(x+k)≡y(y+k)≡z(z+k) (modp)⇔

   

  

p|x+y+k p|y+z+k p|z+x+k

⇔x≡y≡z(modp)

Điều vơ lí x, y, z phân biệt 16x, y, z 6p−1 Do ta có điều phải chứng minh Nhận xét: Rõ ràng hai toán này, ta chủ yếu dùng định lý số định lý số làm sở lập luận

Thặng dư bình phương hữu dụng giải phương trình nghiệm nguyên, ta xét đến ví dụ sau:

Bài tập 6.6 (Serbia-2008) Tìm tất nghiệm ngun khơng âm phương trình :

12x+y4 = 2008z

Lời giải

Nếu z >0 y >0

(79)

Dễ thấy 2008 có ước ngun tố 251 , a,b khơng chia hết cho 251 Từ ta có -1 -3 số chình phương mod 251 Tức

−1 251

= ∨

−3 251 = Mà    −1 251

= (−1)2512−1 =−1

−3 251

=−

251

=−(−1)2512−1 251

= 23=−1

Suy x=y=z=

Kí hiệu Jakobil

Khơng dừng lại tập số ngun tố, tính chất thặng dư bình phương cịn mở rộng cho hợp số Do không sợ nhầm lẫn nên tương tự kí hiệu Legendre, kí hiệu Jakobil viết dạng na

Một số tính chất kí hiệu Jakobil:

Với n số tự nhiên lẻ vàa nguyên, (a, n) = ta có định lý sau: Định lý 6.0.9

Với a, b: a≡b(modn)⇒ a n

= nb

Định lý 6.0.10

Đặt n=Ypiai, với pi ước nguyên tố n, ta có:

a n =Y a piai

=Y a pi

Hay nói cách khác

a mn = a n a m

∀m, n∈Z+

Nhưng lưu ý: a số phương modn khia phải số phương mod pi với pi ước nguyên tố n Ta tiến hành chứng minh hai bước:

1

a piai

=a

pi

ai

2 pqa=ap aq gcd (p, q) =

Định lý 6.0.11 Ta có kí hiệu Jakobil hàm nhân tính hồn tồn, vì:

a n b n = ab n

∀a, b∈Z

Hệ quả: an2=

Định lý 6.0.12

−1

n

(80)

Định lý 6.0.13

2

n

= (−1)

(n−1)(n+1)

Định lý 6.0.14

m

n n m

= (−1)(n−21)( m−1

2 )

Bài tập ví dụ

Bài tập 6.7 Cho m, n∈Z+ sao cho n+ 1 . 4m, chứng minh −m n

=−1 Lời giải

Ta giả sử điều ngược lại −nm=

Vì tính chẵn lẻ m khơng quan trọng ta coi m số lẻ Đầu tiên, ta có nhận xét: −k1= ⇔k−1

Vì n+ 4m ⇒ −1

n

=−1 theo nhận xét Do mn = −nm −n1=−1

Vì n+ 4m ⇒gcd (m, n) = nên ta áp dụng luật nghịch đảo bình phương, ta có:

m

n n m

= (−1)

(m−1)

(n−1)

2 ⇔

n

m

= (−1)

(m−1)

−1

−1

n

=−1

Nếu m= 4t+ 1⇒ −1

m

= ⇒ n

m

=−1⇒ −n

m

=−1 Nhưng n+ = 12+n . 4m⇒ −n

m

= 1, mâu thuẫn

Với trường hợp lại, ta dễ dàng suy điều vơ lí, −nm= khơng thể, suy đpcm

Bài tập 6.8 (Korea 1999) Tìm số nguyên dương n cho 2n −1 . 3 và tồn số nguyên m cho

2n−1 | 4m2+

Lời giải

Ta có nhận xét x2+ 1 .p⇔p= 4k+ 1 vì 2n−1 .3⇒n . 2.

• Điều kiện cần:

Ta chứng minh số nguyên dương cần tìm luỹ thừa Giả sử n q

q >1, q

, ta có: 2n−1 . 2q−1.

Nhưng

 

2q−1 2q+

, 2q−1 có ước nguyên tố dạng 4k+ khác 3, vô lý

Vậy ta có tồn số nguyên m cho 2n−1|4m2+ 1 thì n là luỹ thừa 2.

• Điều kiện đủ:

Theo chứng minh trên, ta có22k

(81)

22k −1 6 . 9, ta phân tích thành thừa số nguyên tố 2n−1

3 =

Q

paii

Áp dụng kí hiệu Jakobil, ta có:

−1

2n−1

3 = −1 Q

paii

=Y

−1

paii

=Y −1 pi =

Do đó, tồn số nguyên t cho

t2+ n−1

3

⇔22kt2+ + 22k−1

2k

−1

⇔22kt2+

2k

−1

⇔4u2+

2k

−1

Kết thúc chứng minh

Bình luận: Trong sách số học Titu Andreescu có lời giải liên quan đến số nguyên tố Fermat định lý thặng dư Trung Hoa, sử dụng kí hiệu Jakobil, toán trở nên đơn giản

Khai thác bổ đề

Bây xin giới thiệu độc giả bổ đề thú vị sau:

Chứng minh với số nguyên dương a số phương, tồn vơ số số ngun tố p cho

a p

=−1

Chứng minh bổ đề Phân tích a thành thừa số nguyên tố, ta cóa= 2t

n

Q

i=1

paii , đơn giản, ta giả sử số số square-free (tức số khơng có ước bình phương số nguyên tố) ⇒ai = 1, t∈ {0; 1} Chọn s số khơng phương modulo pn

Sử dụng định lý thặng dư Trung Hoa, ta dễ dàng tìm số nguyênv cho

      

v ≡1 (mod8)

v ≡1 (modpi) (16i6n−1)

v ≡s(modpn) Đặt v =

m

Q

j=1

qj, với qj ước nguyên tố ta có: m Y i=1 qi = v

= (−1)v

(82)

Và theo định lý nghịch đảo bình phương, ta có: m Y j=1 pi qj = m Y j=1

(−1)

(pi−1)(qj−1) qj pi = v pi

∀16i6n

Do m Y j=1 a qj = " m Y j=1 qj

#2 m Y j=1 n Y i=1 pi qj = n Y i=1 v pi = v pn = s pn

=−1

Suy phải có qj cho

a qj

= −1, mà có vơ số số nên ta có dpcm Với trường hợp a= toán hiển nhiên

Bổ đề lúc đầu nhìn vào ta tưởng khơng có ứng dụng nhiều, thực tế có ứng dụng bất ngờ, điển hình tốn sau:

Bài tập 6.9 (CWMO 2011)Tìm cặp số nguyên (a, b)sao cho với số nguyên dương

n, ta có

n | an+bn+1

Lời giải

Chọn n=p với p số nguyên tố lẻ đủ lớn, ta có

ap+bp+1 ≡a+b2 ≡0 (modp)⇒

a p

= ∀p>p0

Với p0 số nguyên tố lẻ đủ lớn thoả mãn gcd (a, p0) =

Mà theo bổ đề quen thuộc xkhông số phương tồn vơ số số ngun tố lẻ

p cho

x p

=−1, ta phải có −a số phương Tương tự, chọn n = 2p với p số nguyên tố lẻ đủ lớn ta có

−b2p+1 p

= ∀p>p0 ⇒

−b p

= ∀p>p0

Từ suy −blà số phương Tiếp tục chọn n = p với p số nguyên tố lẻ, đặt

−a=k2,−b =l2 ta có

−k2p+ −l2p+1 p⇔l4−k2 ≡0 (modp) (theo định lý Fermat nhỏ)

Do ta lại có l2±k ≡ 0 (modp) với vơ số số nguyên tố p, ta lại suy tiếp (±k)2

là luỹ thừa bậc

Tương tự, chọn n = 2p với p số nguyên tố lẻ ta lại có tiếp (±l)2 luỹ thừa bậc Lập lại q trình vơ hạn lần, ta có−a,−b luỹ thừa bậc2ttuỳ ý, đó

"

−a=−b =

−a=−b =

(83)

Bài tập 6.10 (Mathlink contest 2004) Cho 2004 số nguyên không âma1, , a2004 cho 2004

X

i=1

ani số phương với n Tìm số số hạng nhỏ không

Lời giải

Chọn số nguyên tố lẻp cho p không ước số số cho, áp dụng định lý Fermat nhỏ, ta có:

2004

X

i=1

api−1 ≡k(modp)⇒

k p

=

Do

2004

P

i=1

ani số phương với n

Vì có hữu hạnp cho kp=−1nên theo bổ đề trên, ta có k số phương, mà

k 62004<2025 = 452 ⇒k 6442

Do đó, số số hạng khơng nhỏ 2004−1936 = 68

Ta a1 =a2 = =a1936 =t2, a1937 = =a2004 = thoả mãn đề

Bài tập đề nghị

1 (Bulgaria NMO 1998)Cho m, n số tự nhiên cho A = (m+3)3mn+1 số nguyên Chứng minh rằngA lẻ

2 (Poland 2013) Cho số nguyên a,b cho +a+b2 chia hết cho 6a Chứng minh

rằng a âm

3 Chứng minh rằng3n+ 1khơng có ước ngun tố có dạng3k+ 2nếun là số ngun dương lẻ

4 Chứng minh rằng2n−1ln có ước nguyên tố có dạng 8k+ n số nguyên dương lẻ

5 (Vietnam TST 2003 P6) Cho số nguyên dương n Chứng minh 2n+ 1 không có ước nguyên tố dạng8k−1

6 Chứng minh với số nguyên tốp, tồn số nguyênx, y chox2+y2+ 1 .p.

7 (Korea Final 2000 P1) Chứng minh với số nguyên tố , tồn số nguyên

x, y, z cho x2+y2+z2 =wpvà 0< w < p

8 (IMO Shortlist 1998 N5)Tìm số nguyên dươngn cho tồn số nguyênm để

(84)

9 (Czech-Polish-Slovak MO 2008)Chứng minh tồn số nguyên dươngn cho với số nguyên dương k k2+k+n khơng có ước ngun tố nhỏ 2008.

10 (AMM) Tìm số nguyên dương n cho 2n−1|3n−1. 11 Nếu p= 2n+ (n >2)là số nguyên tố chứng minh rằng

3p−21 + p

12 (Sierpinski) Chứng minh ta khơng thể tìm số nguyên dương n > cho

2n−1+ n

13 (AoPS) Cho số nguyên a, b, c, chứng minh a, b, c, abc khơng số phương tồn vô số số nguyên tố p cho

a p

=

b p

=

c p

14 (Iran TST 2013) Có tồn số nguyên a,b,c cho

a2+b2+c2 2013(ab+bc+ca)

(85)

Phạm Tiến Kha

Cấp nguyên thủy công cụ hữu dụng việc giải toán số học sơ cấp Khơng thế, chúng cịn cầu nối để ta bước lên khái niệm, định lý cao cấp toán học Bài viết xin trình bày số ứng dụng cấp nguyên thủy toán Olympiad

Cấp số nguyên dương

ĐỊNH NGHĨA: Cho (a, n) = Số h >1 gọi cấp a modulo n , kí hiệu h =on(a), h số nguyên dương nhỏ thoả mãn

ah ≡1 mod n

Từ định nghĩa cấp, ta rút tính chất quan trọng sau: Nếun|ak−1thì o

n(a)|k

Chứng minh tính chất đơn giản Đặt on(a) =h, giả sửk =lh+r Chú ý rằng:

ak ≡ahl.ar ≡ar ≡1 mod n

Do r < h nên theo định nghĩa cấp, suy r= 0, hayh|k

Ta bắt đầu với toán quen thuộc cấp:

Bài Chứng minh ước nguyên tố lẻ p số a2n + 1, a > số tự nhiên bất kì, thoả mãn

p≡1 mod 2n+1

Giải.Từ giả thiết suy p|a2n+1−1 Đặth=o

p(a) h|2n+1, theo định lý Fermat nhỏ

h|p−1

Giả sử h <2n+1, tức là h|2n Suy ra p|a2n−1 Mặt khác, theo giả thiết thì p|a2n + 1 Suy ra

p|2(!) Do h= 2n+1, hay

p≡1 mod 2n+1.

Bài toán quen thuộc trở thành bổ đề cho nhiều toán khác cấp Ta xét toán

1Lớp 12CT THPT chuyên Lê Hồng Phong

(86)

Bài Chứng minh tồn vơ hạn số ngun tố có dạng 2nk+ 1 với n cố định.

Giải Rõ ràng tốn có liên quan mật thiết, hay nói cách khác hệ trực tiếp Bài Việc cịn lại ta cần tìm a để dãy xn = a2

n

+ có số hạng đơi ngun tố

Xét dãy Fermat Fn= 22 n

+ Ta chứng minh (Fn, Fm) = với mọim6=n Thật vậy, ý đẳng thức sau:

Fn−2 = Fn−1F2· · ·F0

Từ dễ dàng suy với mọim 6=n (Fn, Fm) = 1, toán giải Nhận xét: Dãy Fermat Fn = 22

n

+ đại lượng 22n

rất phổ biến toán cấp, ta tiếp tục gặp dãy lời giải toán sau

Bài Cho n >1, a số nguyên dương thoả mãn n|an+ 1 Chứng minh rằng

(a+ 1, n)>1

Giải Gọi p ước nguyên tố bé n, đặt on(a) = h Từ giả thiết suy h|2n h|p−1 Do tính bé củap nên (n, p−1) = Do

h|(2n, p−1) = (2, p−1) = 1,2

• Nếuh = p|a−1, mà p|an+ 1 nên p= 2 Khi đó 2|(a+ 1, n).

• Nếuh = p|a2−1, lại theo định nghĩa của h nên suy rap|a+ 1 Khi đó p|(a+ 1, n).

-Nhận xét: Nếup số nguyên tố n|(p−1)n+ 1 thì p|n. Bài Tìm tất cặp số nguyên dương (m, n) thoả mãn

 

n|2m−1+

m|2n−1+ 1

Giải Với m= n= 1,2 ngược lại Ta xét trường hợp m, n >1

Đặt m−1 = 2a.x, n−1 = 2b.y, a, b, x, y nguyên dương xy lẻ Gọi p ước nguyên tố n

p|2m−1 + = (2x)2a +

Theo Bài p≡1 mod 2a+1 Suy ra n cũng có dạng 2a+1.k+ 1, hay n−1 = 2a+1k Suy ra

(87)

Tương tự, ta suy a>b+ 1, điều mâu thuẫn vớib >a+ Tóm lại, (m, n) = (1,1),(1,2),(2,1).

Bài Cho (a, b) = hai số nguyên dương không đồng thời 1, p số nguyên tố lẻ thoả mãn p|a2n

+b2n

p≡1 mod 2n+1

Giải Lại tốn với hình thức gần tương tự với Bài Ở ta cần bước chuyển nhỏ: ý (b, p) = (nếu khơng p|a p|(a, b), trái giả thiết) nên tồn b0

thoả mãn

bb0 ≡1 mod p

Do từ giả thiết suy

p|(ab0)2n +

Và ta bắt gặp lại Bài Bài toán giải quyết.

Một khởi đầu nhẹ nhàng cho viết, đến lúc đẩy độ khó tốn lên bậc Những toán tiếp sau cần suy luận dài tinh tế

Bài (Hàn Quốc 1999) Tìm tất số n nguyên dương cho tồn m thoả mãn

2n−1

3 |4m

2+ 1

Giải.Rõ ràng với n lẻ 2n−1≡ −2 mod nên 2n−1 khơng chia hết cho3 Do đón chẵn Ta có bổ đề quen thuộc sau:

Các số có dạng x2+ 1 thì khơng có ước ngun tố dạng4k+ 3

Don chẵn nên 2n−1≡3 mod 4 Nếun có ước nguyên tố lẻ thì2n−1có ước ngun tố p6= 3 dạng 4k+ Khi p|(2m)2+ 1, mâu thuẫn bổ đề.

Do đón = 2k Ta chứng minh với mọi n như tồn tại m thoả mãn đề bài. Chú ý kí hiệu số FermatFn= 22

n

+ Như

2n−1

3 =Fk−1Fk−2· · ·F1

Lại tính đơi ngun tố số Fermat nên theo định lý Thặng dư Trung Hoa, tồn cchẵn thoả

c2 ≡22i−1 mod 22i, i= 1, k−1

Chọn m= c2, ta có đpcm.

(88)

Bài Cho n số thoả mãn tồn m cho 2n3−1|4m2+ 1.

Chứng minh với ước nguyên dương d 2n3−1, tồn q nguyên dương cho

d≡1 mod 2q

Giải.Rõ ràng với bổ đề Bài 6, toán trở nên đơn giản nhiều Theo kết Bài thì:

2n−1

3 =Fk−1Fk−2· · ·F1

Theo Bài với i, ước nguyên tố pi số Fi thoả mãn pi ≡1 mod 2i+1 Do với ước ngun dương d=pa1

i1p a2 i2 · · ·p

al

il, chọn q = min{i1, i2,· · ·, il}, ta được:

d≡1 mod 2q.

Ta tiếp tục với kết nhà toán học người Pháp Francois Édouard Anatole Lucas: Bài Cho n >1 Nếu p số nguyên tố thoả p|Fn = 22

n

+

p≡1 mod 2n+2

Giải Đây mở rộng Bài Để giải toán này, trước hết ta cần kiến thức thặng dư phương:

Nếu p số nguyên tố thoả p≡1 mod tồn x đểx2 ≡2 mod p

Đây kết bản, khuôn khổ viết xin không nêu chứng minh Trở lại toán Don >1 p≡1 mod 2n+1 nên p≡1 mod 8 Do tồn tại

x để

x2 ≡2 mod p

Luỹ thừa 2n hai vế, ta được

x2n+1 ≡22n ≡ −1 modp

Suy

p|x2n+1+

Sử dụng kết Bài lần nữa, ta đpcm.

Nhận xét: Mở rộng với a thoả mãn tồn x mà a ≡ x2 modp (hay a

thuộc hệ thặng dư phương p), plà ước a2n

+

(89)

Ta tiếp tục đến với kết thú vị kì thi chọn đội tuyển Trung Quốc: Bài (Trung Quốc TST 2005) Chứng minh với n > 2, ước nguyên tố lớn số 22n

+ không bé (n+ 1)2n+2+ 1.

Giải Đặt22n

+ =pk1 p

k2

2 · · ·pkrr với p1 < p2 <· · ·pr Chú ý pi ≡1 mod 2n+1 nên tồn qi cho

pi = + 2n+1qi Viết lại hệ thức 22n

+ =pk1 p

k2

2 · · ·pkrr theo modulo 22n+2, ta có:

1≡1 + 2n+1

r

X

i=1

kiqi mod 22n+2

Suy

2n+1 6

r

X

i=1

kiqi 6qr r

X

i=1

ki Mặt khác

22n+ > + 2n+1k1+k2+···+kr >2(n+1)(k1+k2+···kr) Suy

r

X

i=1

ki <

2n

n+

Do

qr >

2n+1

r

X

i=1

ki

>2(n+ 1)

Vậy

pr >(n+ 1)2n+2+

Nhận xét: Với kết Bài 8, ta phát biểu mạnh hơn: Với n >3 ước nguyên tố lớn 22n

+ không bé (n+ 2)2n+4+ 1.

Một câu hỏi thú vị đặt ra: giá trị bé ước nguyên tố bé số dạng

22n

+ bao nhiêu?

Tiếp theo, ta đến định lý đẹp đẽ số nguyên tố - Định lý Dirichlet:

(90)

Chứng minh định lý hồn tồn khơng dễ, phải sử dụng đến kiến thức toán cao cấp Trong khuôn khổ viết này, ta xét đến trường hợp nhỏ định lý Dirichlet, b = a số nguyên tố

Bài 10 (Hàn Quốc TST 2003) Cho số nguyên tố p bất kì, đặt

fp(x) = xp−1+xp−2+· · ·+x+

a) Chứng minh p|m ước nguyên tố fp(m) nguyên tố với

m(m−1)

b) Chứng minh tồn vô hạn n nguyên dương để pn+ số nguyên tố

Giải a) Gọi q ước nguyên tố fp(m) Rõ ràng (q, m) = Nếu (q, m−1) 6= q|m−1 Khi 0≡fp(m)≡m modq, suy q|m (vơ lí!)

b) Ta chứng minh tồn vô hạn số nguyên tố q mà p|q−1

Gọi q ước nguyên tố fp(m) Suy q|mp −1 Do p số nguyên tố nên

oq(m) = 1∨p

Nếu oq(m) = q|m−1, mâu thuẫn với kết câu a Do oq(m) =p, suy p|q−1 Việc lại xây dựng dãy{mk}k>1 đểfp(mk)chia hết cho vô hạn số nguyên tố Xét dãy m1 =p,mk=pfp(m1)fp(m2)· · ·fp(mk−1) Khi

fp(m1)fp(m2)· · ·fp(mk−1)|fp(mk)−fp(0) =fp(mk)−1 Suy fp(mk)nguyên tố với fp(m1), fp(m2),· · · , fp(mk−1)

Ta công nhận mở rộng toán trên:

Với a nguyên dương dãy a+ 1,2a+ 1, chứa vô hạn số nguyên tố Sau ta xét đến vài ứng dụng định lý Dirichlet toán Số học Bài 11 Cho r số nguyên tố Đặt

fr(x) = xr−1+xr−2+· · ·+x+

Chứng minh rằngfr(x)chia hết cho vô hạn số nguyên tố với x số nguyên tố Giải Giả sửfr(p) chia hết cho hữu hạn số nguyên tố q1 < q2· · ·< qk

Theo định lý Dirichlet, tồn l cho p=l.q1q2· · ·qk+ 1là số nguyên tố Suy

fr(p)≡r≡0 modqi với i thuộc {1,2,· · ·k}

Suy r = qi Hơn nữa, fr(p) chứa ước nguyên tố qi, tồn qj 6= qi mà

qj|fr(p) r =qj =qi, mâu thuẫn với phân biệt qi qj Đặt fr(p) = qis =rs Chú ý

r|p−1 Xét

(91)

Suy fr(p) = r Tuy nhiên, chọn p đủ lớn fr(p)> r, mâu thuẫn Ta có đpcm

Nhận xét: Kết toán tổng quát so với phần chứng minh Bài 11 Và ta kết luận hàm sốfr(x)chia hết cho vô hạn số nguyên tố với x∈Z

Bài 12 (A.Makowski) Cho k > số nguyên dương Chứng minh tồn vô hạn hợp số n thoả mãn n|an−k−1 với mọi a nguyên tố với n

Giải Chọn n = kp với p số nguyên tố Ta cần p|an−k −1 và k|an−k − 1 Chọn n sao cho p−1|n−k, điều kiện thoả mãn Lại chọn n cho ϕ(k)|n−k, điều kiện thứ hai thoả mãn Tóm lại, ta cần chọnp choϕ(k)|p−1, chứng minh tồn vô hạn p Tuy nhiên lại kết trực tiếp từ định lý Dirichlet, tốn chứng minh hồn tồn.

Bài 13 Cho k > số nguyên Chứng minh tồn số nguyên tố p dãy tăng nghiêm ngặt a1, a2, thoả mãn p+ka1, p+ka2, số nguyên tố

Giải Theo định lý Dirichlet tồn vơ hạn số nguyên tố p thoả p ≡ mod k Trước hết chọn số nguyên tố p vậy, đặtp=lk+

Khi đóp+kai =k(l+ai) + Việc tồn dãy tăng nghiêm ngặt a1, a2, hiển nhiên

theo định lý Dirichlet.

Bài 14 (Định lý Hardy-Wright) Cho p số nguyên tố lẻ Chứng minh tồn x, y nguyên dương cho

p|x2+y2+

Giải.• Nếup= 4k+ 1thì −1là số phương mod p, tức tồn xsao chop|x2+ Chọn

y=p, ta cóp|x2+y2+

• Nếup= 4k+ 3:

Chú ý (4p,2p−1) = Xét dãy

1.4p+ 2p−1,2.4p+ 2p−1,3.4p+ 2p−1,

Theo định lý Dirichlet, tồn l cho l.4p+ 2p−1 số nguyên tố Khi

l.4p+ 2p−1 = (4l+ 2)p−1≡1 mod

Theo kết quen thuộc: số nguyên tố dạng 4k+ biểu diễn dạng tổng hai số phương, suy tồn tạix, y để

l.4p+ 2p−1 = x2+y2

(92)

Bài 15 Chứng minh với số nguyên tố p a ngun dương, số ap −1 ln có ít ước nguyên tố q thoả mãn

q≡1 modp

Giải.Ta xét trường hợpa > p Đặtap−1 =qα1 q

α2

2 qkαk Ta cóoqi(a) = p∨1vàoqi(a)|qi−1 Giả sử khơng tồn isao cho oqi(a) = p Khi với j = 1, k qj|a−1 Mặt khác:

ap−1+ap−2+· · ·+a+ 1≡p mod qi, i= 1, k Suy (ap−1+ap−2+· · ·+a+ 1, q

i) = 1,∀qi 6=p Do vqi(ap−1) =vqi(a−1) = αi,∀qi 6=p Giả sử p =qj Suy ap −1|(a−1)q

αj

j , hay ap

−1+ap−2 +· · ·+a+ 16 qαj

j = pαj 6pp

−1 (vơ

lí!) Trường hợp cịn lại a6p (p, qi) = 1, i= 1, k Từ tiếp tục giải trên, ta có đpcm

Do đó, ln tồn ước nguyên tốq ap−1 thoả q≡1 mod p.

Nhận xét: Với cách lí luận trên, ta thay p luỹ thừa p Có thể thấy tốn gần giống với Bài 1, nhiên thay "với ước nguyên tố" Bài ta khẳng định "tồn ước nguyên tố" Dạng tổng quát yếu hơn, lại bắt nguồn để ta đến định lý mạnh nhiều-Định lý Zsigmondy: Dạng 1:

Với mọia > b>1 (a, b) = 1thì an−bn ln có ước nguyên tố p thoả mãn

p|an−bn, p6 |ak−bk với 16k < n

(trừ trường hợp 26−16 a2−b2 với a+b luỹ thừa 2) Dạng 2:

Với a > b > an +bn ln có ước ngun tố p thoả mãn p|an +bn p6 |ak+bk với mọi16k < n

(trừ trường hợp 23+ 13)

Ta không đề cập đến chứng minh dài định lý đây, mà xét số ứng dụng

Bài 16 (Romanian TST) Chứng minh dãy an = 3n −2n khơng có ba số hạng lập thành cấp số nhân

Giải Giả sử tồn x < y < z cho

(3y −2y)2 = (3x−2x)(3z−2z)

Từ đẳng thức suy ước nguyên tốp 3x−2x đều chia hết3y−2y Tuy nhiên, do

(93)

Bài 17 (IMO SL 2000) Tìm tất cảa, m, n nguyên dương thoả mãn

am+ 1|(a+ 1)n

Giải.Nếu a6= theo định lý Zsigmondy, am+ 1 tồn ước nguyên tốp mà p6 |a+ 1, mâu thuẫn với yêu cầu đề

Nếu m= 3, a= Khi mọin >2 thoả mãn

23+ 1|(2 + 1)n

Nếu a= chọn m, nnguyên dương

Vậy (a, m, n) = (2,3, x+ 2) với x∈Nhay (1, m, n).

Bài 18 (Nhật Bản 2011)Tìm a, n, p, q, r nguyên dương thoả mãn

an−1 = (ap−1)(aq−1)(ar−1)

Giải.Giả sửp>q>r, Nếua, nkhông rơi vào trường hợp ngoại lệ định lý Zsigmondy, ta dễ dàng suy điều vơ lí

Nếu a= 2, n = 6, suy p= 6, q=r= Nếu n= không tồn a, p, q, r

Nếu a= chọn n, p, q, r nguyên dương

Tóm lại, (a, n, p, q, r) = (1, n, p, q, r),(2,6,6,1,1),(2,6,1,6,1),(2,6,1,1,6).

Bài 19 (Iran) Cho A tập hữu hạn số nguyên tố a > số nguyên dương Chứng minh tồn hữu hạn n cho tất ước nguyên tố củaan−1đều thuộc A. Giải Giả sửA ={p1, p2,· · · , pk} Ta xét N = [p1 −1, p2−1,· · ·, pk−1] Rõ ràng

pi|aN −1,∀i= 1, k

Khi đó, theo định lý Zsigmondy, với mọin0 > N, sốan0−1ln có ước ngun tốq không

phải ước củaaN −1, hay q 6∈A.

Suy n số thoả điều kiện đề thìn 6N, tức có hữu hạnn vậy.

Bài 20 (Ba Lan) Cho < q < p hai số nguyên tố lẻ Chứng minh 2pq −1 có ít ba ước ngun tố phân biệt

Giải Theo định lý Zsigmondy,2pq−1 có ước nguyên tố r mà r 6 |2p −1 và r6 |2q−1. Lại theo định lý Zsigmondy, 2p−1 có ước nguyên tố s mà s6 |2q−1.

(94)

tố Do đó, 2q−1 cũng có ước nguyên tố t không chia hết 2p−1. Ba ước nguyên tố r, s, tchính ba ước cần tìm.

Bài 21 Cho a ∈ N∗ Gọi A là tập hợp số tự nhiên n sao cho n|an + 1, và B là tập tất ước nguyên tố phần tử A Chứng minh B hữu hạn

Giải •Trường hợp n lẻ:

Gọi B1 tập hợp ước nguyên tố phần tử lẻ trongA

Từ giả thiết suy n|a2n−1 Gọi p là ước nguyên tố bé củan và đặt h =o

p(a) Khi

h|2n h|p−1 Suy h|(2n, p−1) = (2, p−1) = 1∨2 Suy p|a2−1, hay B

1 hữu hạn

• Trường hợp n chẵn:

Gọi B2 tập hợp ước nguyên tố phần tử chẵn trongA

Đặt n= 2n1 n1 lẻ (nếu khơng 4|n| a

n

22+ 1, vơ lí) Từ lí luận tương tự trường hợp

n lẻ, ta suy B2 hữu hạn

Do B1, B2 hữu hạn nênB =B1∪B2 hữu hạn

Nhận xét: Với cách lí luận tương tự, ta có toán quen thuộc (và bản) sau đây:

Nếu n|2n−1thì n= 1

Tuy vậy, thật bất ngờ tốn đơn giản có ứng dụng thú vị: Bài 22 (Ba Lan) Tìm tất đa thức với f(n) có hệ số hữu tỉ thoả mãn

f(n)|2n−1,∀n∈N

Giải Chú ý rằngf(n)|f(n+f(n)) với mọin Suy

f(n)|f(n+f(n))|2n+f(n)−1

Suy

f(n)|2n+f(n)−2n= 2n 2f(n)−1

Do (2n, f(n)) = với mọin nên

f(n)|2(f(n)−1

Suy f(n) = 1,∀n ∈N.

Nhận xét: Một tốn "có vẻ tương tự" dành cho bạn đọc:

Tìm tất đa thức hệ số nguyên f(n)thoả mãn f(n)|2n+ 1,∀n ∈ N Bài 23 Tìm số nguyên tố p nhỏ cho tồn a thoả mãn p|a221

−a p |a2k

−a,

(95)

Giải Rõ ràng(a, p) = nên p|a221−1

−1 Đặth=op(a) Khi h thoả hệ điều kiện sau:

    

   

h|221−1

h6 |2k−1,∀16k <21

h|p−1

Với 16k <21, ta có bổ đề quen thuộc

(221−1,2k−1) = 2(21,k)−1627−1 = 127

Do với p >127 điều kiện h6 |2k−1,∀16k < 21được thoả mãn Chọn h= 49, giá trị nhỏ p 197

Tóm lại, giá trị nhỏ plà 197.

Nhận xét: Bạn đọc giải tốn tương tự sau: Tìm p ngun tố nhỏ cho tồn tạia thoả

 

p|a215+1

−1

p6 |a2k+1

−1,∀16k < 15

Bài 26 (Fermat) Cho p >3 số nguyên tố Chứng minh ước dương 2p3+1 có dạng 2kp+

Giải Trước hết nhận thấy

v3(2p+ 1) =v3(2 + 1) +v3(p) =

nên ước nguyên tố số 2p3+1 lớn Gọi q ước ngun tố 2p3+1 Khi q|2p+ 1, suy ra q|22p −1 Gọi h =o

q(2) Ta có h|2p, suy rah = 1∨2∨p∨2p Nếu h = q|1 Nếuh = q|22−1 = 3 Nếu h=p thì q|2p−1, suy raq|h Tất trường hợp vơ lí Do h= 2p Suy rah= 2p|q−1 hay q≡1 mod 2p Từ dễ dàng kết luận ước dương 2p3+1 có dạng 2k+ 1.

Nhận xét: Bài tốn tổng quát sau:

• Cho p, q > số nguyên tố phân biệt Khi tất ước dương số

(q−1)p+1

q có dạng 2kp+

• Cho p, q > số nguyên tố phân biệt Khi tất ước dương số

(q+1)p−1

q có dạng 2kp+

Một số toán thú vị phát triển từ toán này:

(96)

sau:

i) a bội số 4; ii)a lẻ và(a, p) =

Chứng minh ước dương (a−2)ap+2p (a−1)ap+1 có dạng2kp+

FCho số nguyên tố p >3 Chứng minh ước dương 2p3+1 có dạng2kp+ 1,

k ≡p mod ước ước nguyên tố

Bài 27 (VN TST 1997) Chứng minh tồn hàm số f nhận giá trị nguyên thoả mãn

2n|19f(n)−97

với n nguyên dương Giải Dễ thấy

v2(192

n−2

−1) =v2(19−1) +v2(2n−2) = n

nên o2n(19) = 2n−2.ngSuy ra19t chứa2n−2 thặng dư phân biệt hệ thặng dư của2n Mặt khác, để ý 19t chỉ chứa thặng dư lẻ, cụ thể 19t ≡ 1,3 mod 8, mà 2n−2 là một

nửa số thặng dư lẻ 2n, suy ra 19t chứa tất thặng dư đồng dư với 1,3 mod 8 của 2n. Do 97≡1 mod 8, suy tồn tạif để19f(n)≡97 mod 2n.

Bài 28 (Italian TST 2006) Cho số nguyên dương n Gọi An tập tất a ∈Z,16a6n thoả mãn n|an+ 1

a) Tìm tất n để An6=∅

b) Tìm tất n để |An| chẵn khác khơng c) Liệu có tồn n để |An|= 130 không? Giải a) Vớin lẻ, ta chọn a=n−1

Xét n chẵn, đặt n = 2n1 Khi đó2n1|(an1)2+ Suy n1 lẻ n1 có ước nguyên tố dạng

4k+ Tóm lại, An 6= ∅ n lẻ n = 2n1, n1 lẻ gồm ước nguyên tố

dạng 4k+

b) Xét n > chẵn Khi n|an + 1 thì n|(n − a)n + 1 (chú ý rằng a 6= n

1) Do

nếu n chẵn thì|An| chẵn

Xét n lẻ Chú ý ϕ(n) chẵn với mọin > 2, suy ϕ(n)−2cũng chẵn

Rõ ràng(a, n) = Xéta6= 1, n−1thoản|an+ 1 Do (a, n) = 1nên tồn tại26b6n−2thoả

ab ≡1 mod n Lại cón|an+ 1, suy ran|bn+ 1 Điều có nghĩa là |A

n\ {n−1} | chẵn, hay

|An|lẻ

Tóm lại, |An| chẵn n chẵn vàn >2

c) Giả sử tồn n thoả |An|= 130 Theo câu b thìn chẵn Đặt n= 2pα11p

α2 · · ·p

αk

k ,

pi ≡1 mod với mọii= 1, k Ta chứng minh bổ đề sau:

(97)

đúng nghiệm modulo p

Xét x thoả x2 ≡ −1 mod p Giả sử tồn tại y 6= x thoả mãn y2 ≡ 1 mod p Khi đó x2 ≡ y2

mod p, suy x≡ −y mod p, hay y=p−x.

Trở lại toán Do pi có dạng 4l+ nên −1 số phương mod pαii Theo bổ đề số nghiệm theo modulo n phương trình x2 ≡ −1 mod n là 2k Suy không tồn n để|An|= 130

Bài 29 (IMO 2003) Chứng minh với số nguyên tố p, tồn số nguyên tố

q ước np−p với mọi n>1. Giải Với p= q = Xétp lẻ Ta có

pp−1 = (p−1)(1 +p+p2+· · ·+pp−1)

Chú ý +p+p2 +· · ·+pp−1 khơng đồng dư mod p2, nên có ước ngun tố

q khơng có dạng kp2 + Ta chứng minh q thoả điều kiện đề bài.„ suy q|p−1 Suy

0≡1 +p+p2+· · ·+pp−1 ≡p modq, hayp=q, vơ lí.

Bài 30 Tìm số nguyên k, m, n >0 cho km|mn−1 kn|nm−1 Giải Gọi p>3là ước nguyên tố k Giả sử m > n Đặt d=op(m) Suy

vp(mn−1) =vp(md−1) +vp

n

d

=op(md−1) Suy pm|md−1 Suy pm 6md−16mp−1−1< mp (1)

Mặt khác, km 6mn−1< mm−1< mm Suy rap < k < m

Nếu p> m >3 Khi hàm số f(x) = logxx đồng biến [3,+∞), suy logmm > logpp, hay pm > mp, mâu thuẫn (1)

Suy p= Suy m, n lẻ

Nếu n= dễ dàng suy m= Khi k

Xét m > n >1 Ta có 4|km|mn−1, lẻ nên m≡1 mod Suy

v2(mn−1) =v2(md−1)

Suy 2m|md−1, suy ra 2m 6md−1 = m−1, vơ lí. Tóm lại, (m, n, k) = (1,1, k),(m, n,1).

Căn nguyên thủy

(98)

đây ta xét số tập nguyên thủy

Bài Chứng minh nguyên thủy 3n với mọi n>1.

Giải Ta chứng minh quy nạp Dễ thấy khin = 1, kết luận toán Giả sử kết luận vớin =k, tức là2ϕ(3k)

≡22.3k−1

≡1 mod 3k Gọi dlà bậc của2 mod 3k+1.

Do 2d ≡ 1 mod 3k+1 nên 2d ≡ 1 mod 3k, suy ra 2.3k−1|d Mặt khác, d|ϕ3k+1 = 2.3k Suy ra

d= 2.3k−1 hay d= 2.3k Ta chứng minh bổ đề sau quy nạp:

22.3n−1 ≡1 + 3n mod 3n+1

Bổ đề với n= Giả sử bổ đề với n=k, suy

22.3k−1 = + 3k+ 3k+1.m

Lập phương hai vế

23k = + 3k+1+ 3k+2.M

Suy

22.3k ≡1 + 3k+1 mod 3k+2

Theo nguyên lý quy nạp, bổ đề chứng minh Suy d= 2.3k.

Nhận xét: Ta chứng minh toán sau: nguyên thủy 5n với mọi

n >1

Bài Chứng minh n = 3k−1 thì 2n≡ −1 mod 3k.

Giải Theo Bài nguyên thủy 3k, suy bậc của 2 mod 3k là 2n Suy ra

22n−1≡ (2n−1)(2n+ 1) ≡ 1 mod 3k Mặt khác, 2n−1≡(−1)3k−1

−1≡ mod 3, suy

2n+ 1 ≡1 mod 3k.

Bài Cho n > p = 2n + 1 Chứng minh nếu 3p−21 + 1 ≡ 0 mod p thì p là số nguyên tố

Giải Do p−21 = 2n−1 nên 32n−1

≡ −1 mod p, suy 32n

≡1 mod p, suy bậc mod p

là2n, hayp−1 Suy rap−1|ϕ(p), hayp−16ϕ(p) Suy raϕ(p) = p−1, hayplà số nguyên tố. Bài (AMM) Xét f(n) ước chung lớn 2n−2,3n−3,4n−4, Xác định f(n) chứng minh f(2n) =

(99)

f(n) =p1p2· · ·pk với pi−1|n−1, i= 1, k

Xétf(2n) Theo cách xác định nếuf(2n)có ước ngun tố q >2thì q−1là ước chẵn 2n−1, vơ lí Do f(2n) = 2.

Bài Cho p>2 số nguyên tố Tìm tất số nguyên dương k cho

p|1k+ 2k+· · ·+ (p−1)k

Giải ĐặtSk = 1k+ 2k+· · ·+ (p−1)k Với k bội củap−1, ta có

Sk≡p−16≡0 mod p

Ta chứng minh với k bội p−1 p|Sk

Thật vậy, gọi a nguyên thủy p Dễ dàng chứng minh {0, a1, a2, , ap−1} là hệ

thặng dư đầy đủ modp Suy (a1, a2, , ap−1) là hoán vị của (1,2, , p−1).

Suy

1k+ 2k+· · ·+ (p−1)k≡ a1k+ a2k+· · · ap−1k

≡ak +ak+· · ·a(p−2)k

≡ak.a

(p−1)k−1

a−1

≡0 mod p

Vậy với k khơng phải bội p−1 p|Sk

Nhận xét: Bài tốn có ứng dụng đặc sắc sau:

Choa, b, c số nguyên vàplà số nguyên tố Chứng minh tồn số nguyên x, y, z

không đồng thời chia hết cho psao cho p|ax2+by2+cz2.

TÀI LIỆU THAM KHẢO

1 Titu Andreescu, Gabriel Dospinescu, Prolems from the book Naoki Sato, Number theory

(100)(101)

Lưu Giang Nam

Phần nguyên lĩnh vực hay độc đáo toán sơ cấp, cao cấp ứng dụng.Có nhiều tốn hay phần nguyên sử dụng làm đề thi học sinh giỏi cấp, có nhiều đề thi học sinh giỏi quốc gia Olympic quốc tế Mặt khác, hàm phần nguyên có ứng dụng quan trọng khơng tốn học phổ thơng, mà cịn nhiều vấn đề tốn ứng dụng cơng nghệ thơng tin (làm trịn số, tính gần đúng, ) Phần nguyên thể kết nối tính liên tục tính rời rạc, tốn giải tích tốn rời rạc nên thú vị Tuy nhiên giới hạn kiến thức nên viết tác giả xin trình bày tính chất, ứng dụng phần nguyên phạm vi THPT

Định nghĩa, tính chất tập bản

Định nghĩa

Phần nguyên số thực x, kí hiệu [x], số nguyên lớn không vượt x

Phần lẻ số thực x, kí hiệu {x}, phần cịn lại x bỏ phần nguyên: {x}=x−[x] Giá trị nhỏ hai sốx−z x+ 1−z gọi khoảng cách từ x đến số ngun gần kí hiệu ]x[

Số nguyên gần số thực x kí hiệu (x)và gọi số làm trịn x

Các tính chất quen thuộc

1 x= [x] x∈Z

2 x={x} 06x <1

3 x−1<[x]6x

4 Nếu k∈Z [x+k] = [x] +k,{x+k}={x}+k

5 [x+y]−[x]−[y]∈ {0; 1} suy [x+y]>[x] + [y],{x+y}6{x}+{y}

6

[x]

n

=hx

n

i

, n∈N

1Chuyên Phan Ngọc Hiển Cà Mau

(102)

7 Số số nguyên dương chia hết cho n không vượt x hx

n

i

8

x+a b

= x

b , n −→+∞

9 x>y⇔[x]>[y]

10 Nếu [x] = [y] |x−y|61

11 Trong hai số x y có số ngun số khơng phải số nguyên <

{x}+{y}<1

12 Với x y số thực ta có[2x] + [2y]>[x] + [y] + [x+y]>2([x] + [y])

13 Nếu max{{x},{y}}< 12

[2{x}] + [2{y}] = = [{x}+{y}] [2x] + [2y] + [x] + [y] + [x+y] = 2[x] + 2[y]

14 Nếu min{{x},{y}}< 12 6max{{x},{y}}62[x] + 2[y]

[2{x}] + [2{y}] = = [{x}+{y}] + [2x] + [2y] = [x] + [y] + [x+y] = 2[x] + 2[y] +

15 Nếu 12 6min{{x},{y}}thì

[2{x}] + [2{y}] = = [[{x}+{y}] + [2x] + [2y] = [x] + [y] + [x+y] + = 2[x] + 2[y] +

16 Vớix∈R ta có

{x}+1

= [2{x}]và

x+1

= [2x]-[x]

Hệ : Với số nguyên dương ta ln có hn

2

i

+

n+

=n

17 Với số tự nhiên n với số thực x∈R ta có n[x]6[nx]6n[x] +n−1

18 Với số thực x số nguyên với số ngun n ta ln có[x]+[n−x] =

n−1

19 Cho k1,k2, ,kn, n số nguyên dương Khi

k1 +k2+k3+ +kn >

k1+k2+ +kn

n

+n−1

20 Cho a b >2 số tự nhiên Khi [logba] + số chữ số số a viết hệ đếm số b

21 Giả sử r phần dư chia số nguyên m cho số nguyên dương n , m=pn+r

với r ∈ {0,1, , n−1} Khi

r=m−nhm n

(103)

22 Nếu p q số nguyên dương cho p

q khơng phải số ngun p

q >

p q

+

q

23 Cho q số tự nhiên, x số thực dương Có đúnghxqisố tự nhiên không vượt x chia hết cho q

24 Các qui tắc đổi chỗ (hoán vị), kết hợp phép toán cộng phép toán nhân; qui tắc kết hợp phép toán nhân phép toán cộng cho phần nguyên phần dư 25 Định lí Legendre: Số mũ số nguyên tố p phân tích tiêu chuẩn n! tính

theo cơng thức:

vp(n) =

+∞ X

i=1

n pi

26 Định lí Hermite : Với n∈N∗, x số thực ta có :

[nx] = [x] +

x+

n

+ +

x+ n−1

n

Bài tập bản

1 Phương trìnhx4−3x3−6 = 0 có nghiệm thực p q Tính [p] + [q].

2 Giải phương trình[x2] = [x]2

3 Giải phương trìnhhx

2

i

+hx

i

+hx

i

=x

4 Giải bất phương trình[x] +{x}< x−1

5 Cho x số thực dương thỏa √x+√x+ =√4x+ Chứng minh

∃n∈N∗ :

n2 +n

64n+ <(n+ 1)2

6 (THTT số 408 ) Giải phương trình x2−(1−[x])x+ 2011 =

7 (THTT số 424) Giải phương trình [x]3+ 2x2 =x3+ 2[x]2

8 ( THTT số 411) Tìm tất hàm số liên tục f : R −→ R thỏa mãn {f(x+y)} =

{f(x) +f(y)}với x, y ∈R

9 ( APMO 1993) Tìm tất giá trị khác hàm số

f(x) = [x] + [2x] +

5x

3

(104)

10 (THTT số 416) Chon >1số hữu tỉr1, r2, rn thỏa mãn 0< ri

1 2,

n

X

i=1

ri = hàm số

f(x) = [x] +

x+1

Hãy tìm giá trị lớn biểu thức P(k) = 2k−

n

P

i=1

f(kri)khi k chạy tập số nguyên

11 ( Romania MO 2003 ) ChoA=√4n2+n, n ∈

N Chứng minh

{A}6

4

12 (Austrian MO 1974, Hong Kong TST 1988) Chứng minh

h√

n+√n+ 1i =h√4n+ 2i

13 ( Canada MO 1987) Cho n số tự nhiên Chứng minh rằng:

h√

n+√n+ 1i =h√4n+ 1i =h√4n+ 2i+h√4n+ 3i

14 (Đề nghị Olympic 30/4, THPT Lê Quý Đôn Quảng Trị) Tìm tất nghiệm khơng ngun phương trình

x+ 96

x = [x] +

96 [x]

15 (Đề nghị Olympic 30/4, THPT Lê Quý Đơn Quảng Trị) Giải phương trình

8x+

+

4x−1

= 16x−7

16 (Sweden MO 1982) Với mỗin ∈N , xác định xem phương trình x2−[x2] ={x2} có

bao nhiêu nghiệm đoạn [1;n]

17 (VMO 1979) Tìm tất sốα cho phương trình x2−2x[x] +x−α = 0 có hai

nghiệm số phân biệt không âm

18 (Olympic Czech and Slovak, 1998) Tìm tất số thực x cho

x[x[x[x]]] = 88

19 (Belarusian Olympiad 1999) Chứng tỏ phương trình {x3}+{y3} = {z3} có vơ số nghiệm ngun

20 (Australian MO 1999) Giải hệ phương trình

   

  

x+ [y] +{z}= 200

(105)

Ứng dụng định lý Hermite định lý Legendre

Ứng dụng định lý Hermite qua tốn

Định lí Hermite : Với số tự nhiên n số thực x ta ln có :

[x] +

x+

n

+

x+

n

+ +

x+ n−1

n

= [nx]

hay

[nx] =

p−1

X

i=0

x+ i

n

(∗)

Hệ :

[x] +

x+1

= [2x] X

06i6j6n

x+i j

=n[x]

Đẳng thức (*) đẳng thức đẹp số học tổ hợp, ứng dụng nhiều tốn tính tổng liên quan đến phần nguyên Trước hết ta xét toán mở đầu:

Ví dụ 1: Cho n ∈R, m∈N, m>2, tính tổng

S =

∞ X

i=0

m−1

X

j=0

n+jmi

mi+1

Giải : Ta có

S =

∞ X

i=0

m−1

X

j=0

n mi+1 +

j m = ∞ X i=0 h n mi i

−h n

mi+1

i

= [n]

Vậy S = [n] Bài toán chứng minh đơn giản có ứng dụng hay nhiều tốn tính tổng, đặc biệt tốn có nhiều dấu P

Ta xét số toán ứng dụng cho bổ đề :

Bài toán 1: Chứng minh n−1

X

k=0

∞ X

i=0

m−1

X

j=1

2k+jmi

2i+1

= 2n−1

HD : Ứng dụng bổ đề ta có V T =

n−1

X

k=0

2k =

n−1

2−1 =

n−

1(ĐPCM) Bài toán 2: Chứng minh

p−1

X

k=0

∞ X

i=0

m−1

X

j=0

"

x+ kp +jmi

mi+1

#

= [px]

HD: Ứng dụng bổ đề ta có V T =

p−1

X

k=0

x+k

p

= [px]( Áp dụng định lí Hermite lần ) Bài tốn 3: Tìm m đểy với

y =

p−1

X

k=0

∞ X

i=0

m−1

X

j=0

"

Cn2+kp +jmi mi+1

# − [p 2] X k=0 ∞ X j=0

m−1

X

j=0

"

C2k p +jmi

mi+1

(106)

HD: Dễ thấy y = p

2(p−1)

2 −2

p−1

Để y p2(p−1) Điều dẫn ta tới p≡0( mod 2)hoặc p≡1( mod 4) Bài toán 4: ( IMO 1968) Tính tổng

∞ X

k=0

x+ 2k 2k+1

HD: Áp dụng bổ đề với j = 1, m= ta dễ dàng thu

∞ X

k=0

x+ 2k 2k+1

= [x] Bài tốn 5: Tính tổng

S =

m−1

X

i=0

ai+b am

HD : Ta có

S=

m−1

X

i=0

ai+b am −

m−1

X

i=0

ai+b am

=

m−1

X

i=0

ai+b am −

m−1

X i=0 b am+ i m ,

đến áp dụng bổ đề ta kết

Ứng dụng định lý Legendre số mũ nguyên tố

Định lí Legendre: Số mũ số nguyên tố p phân tích tiêu chuẩn n! tính theo cơng thức

vp(n) =

X

i>1

n pi

Chứng minh :

Ta có nhận xét n < pi thì

n pm

= 0, m >i Trong phân tích chuẩn n có

n p

bội số p

Do

n! =p[np]

n p

!A1 , (A1, p) =

Tương tự

n p

! =p " [n p] p #  h n p i p 

!A2 , (A2, p) =

Mà theo tính chất

[x]

n = hx n i ta có   h n p i p  = n p2

⇒n! =p[

n p]+

h n p2 i n p2

!A2

Lí luận tương tự với

n p2

!và tiếp tục

n pk

< p Cuối ta thu số mũ vp(n)của p phân tích chuẩn n! :

vp(n) =

(107)

trong số k thỏa mãn pk6n < pk+1 ( ĐPCM).

Vậy định lí chứng minh

Đây định lí có ứng dụng rộng tốn chứng minh chia hết tính số mũ nguyên tố phân tích chuẩn n! Ta mở đầu toán đề IMO 1972 : Ví dụ 1: Cho số tự nhiên m, n, chứng minh

(2m)!(2n)!

m!n!(m+n)! ∈N

HD: Ta tìm số mũ p phân tích chuẩn (2m)!(2n)! vàn!m!(m+n)!, sau chứng minhvp((2m)!.(2n)!)>vp(n!m!(m+n)!)

Giải : Ta có

vp((2m)!(2n)!) =

∞ X i=1 2m pi + ∞ X j=1 2n pj

vp(n!m!(m+n)!) =

∞ X i=1 m pi + ∞ X j=1 n pj + ∞ X k=1

m+n pk

Ta cần chứng minh

2m pk + 2m pk > m pk + n pk +

m+n pk

Theo tính chất 12 ta thấy BĐT => ĐPCM Vậy (2m)!(2n)!

m!n!(m+n)! ∈N

(đpcm) Các tập tương tự :

Bài tốn :CMR với m, n∈N ta ln có

(m+n)!

m!n! ∈N

HD: Áp dụng BĐT [x+y]>[x] + [y]

Bài toán :( USMO 1975) CMR vớin ∈N ta có

(5m)!(5n)!

m!n!(3m+n)!(3n+m)!

là số tự nhiên

HD: Chứng áp dụng BĐT [5x] + [5y]>[3x+y] + [x+ 3y]

Bài tốn :CMR với n ∈Nta có

12(5n)!

n!(n+ 1)!(n+ 2)!(n+ 3)!(n+ 4)!

là số tự nhiên

Tiếp theo ta qua dạng chứng minh chia hết, kết hợp với hệ đếm số Ví dụ : Cho số nguyên tố p Tìm n ∈N cho n! pn−1.

(108)

>p= ta cần chứng minh n! 2n−1 Với n lẻ tức n= 2k+ ta có :

v2((2k+ 1)!) =v2((2k)!) =

∞ X i=1 2k 2i < ∞ X i=1 2k

2i =

2k

2−1 = 2k =n−1

Suy số mũ phân tích n! nhỏ n-1, n! khơng chia hết cho 2n−1 Với n = 2k(2m+ 1) ta có :

v2(n!) =v2((2k(2m+ 1))!) =

∞ X

i=1

2k(2m+ 1)

2i

=

k−1

X

t=0

2i(2m+ 1) +

∞ X

t=0

2m+ 2t

<(2m+ 1)(2k−1) + 2m= 2k(2m+ 1)−1 = n−1

Suy n!không chia hết cho 2n−1

Với n = 2k ta có

v2(n!) =v2((2k)!) =

∞ X i=1 2k 2i =

k−1

X

i=1

2i= 2k−1 = n−1

Suy n! chia hết cho 2n−1.

Vậy n = 2klà giá trị cần tìm.

>p= làm tương tự ta thu khơng có giá trị n thỏa mãn >p >3 ta có kết khơng có giá trị n thỏa mãn Vậy có giá trị n= 2k, k∈

N ( ứng với p=2) thỏa đề Nhận xét : Từ ta có nhận xét nếun = 2k, k∈

N tất ước nguyên tố n! lớn n-1

Sau số toán sử dụng hệ đếm số tốn phần ngun Ví dụ 3: Giả sử m! = 2l(2k+ 1), m, k, p∈

N, chúng minh tồn vô hạn m cho:

2m−l= 11+ 22+ + 20142014

Giải:

Giả sử hệ đếm số , m biểu diễn dạng:

m=anan−1 a0 =

n

X

k=0

ak2k

Khi

l =vp(m!) =

∞ X i=0 hm 2i i = n X i=0 hm 2i i = n X i=0 " n X k=0

ak2k−i

# = n X k=0 " ak n X i=0

2k−i

(109)

Mặc khác ta có

ai n

P

i=k+1

2k−i

= ta có :

l= n X k=0 " ak k X i=0

2k−i

# = n X k=0 "

ak2k k X i=0 2i # = n X k=0

ak.2k

1−

2k+1

1− = n X k=0

ak.2k

2−

2k

= 2m−

n

X

k=0

ak

Suy

2m−l =

n

X

k=0

ak

Công việc cuối phải chứng minh tồn vô hạn số (ak)ni=0 cho

n

X

k=0

ak= 11+ 22+ + 20142014

Mà điều hiển nhiên Vậy tốn chứng minh Ta xét ví dụ tương tự sau:

Bài toán 1: Giả sử m! = 3l(3k+ 1), m, k, p ∈

N, chúng minh tồn vô hạn m cho:

m−2l = 1! + 2! + + 30!

HD : Làm tương tự trên, giả sửm =anan−1 a0 =

n

X

k=0

ak3k , sau biến đổi ta đượcm−2l= n

X

k=0

ak

Hàm có chứa phần nguyên

Trong tổ hợp, dạng tốn tính tổng đa dạng nhiều cách giải Một vần đề hay gặp tốn tính tổng tổng hàm có chứa phần ngun Sau tơi xin giới thiệu dạng toán đẹp phần với tốn mở đầu

Ví dụ 1: Cho p số nguyên tố lẻ, q số nguyên không chia hết cho p, f :N−→R , thỏa 1)f(x) không chia hết cho p , với mọix∈N

2)f(x) +f(p−x) chia hết cho p với x∈N

Chứng minh :

p−1

X

k=1

f(k).q p

= q

p

p−1

X

k=1

f(k)− p−1

(110)

Giải : Ta có qf(k)

p +

qf(p−k)

p ∈Z, mà

qf(k)

p ∈/Z nên

qf(p−k)

p ∈/ Z với mỗik = 1, p−1

Do từ tính chất phần lẻ ta có

06

qf(k)

p

+

qf(p−k)

p

<2

Suy

qf(k)

p

+

qf(p−k)

p

=

Lấy tổng giá trị từ đến p-1 ta có : p−1

X

k=1

qf(k)

p

+

p−1

X

k=1

qf(p−k)

p

=p−1

Suy

2

p−1

X

k=1

qf(k)

p

=p−1⇔

p−1

X

k=1

qf(k)

p

= p−1

2

Từ ta có

p−1

X

k=1

f(k)q

p

=

p−1

X

k=1

q

pf(k)−

p−1

X

k=1

q pf(k)

=

p−1

X

k=1

q

pf(k)− p−1

2

Suy đpcm Vậy

p−1

X

k=1

f(k)q

p

= q

p

p−1

X

k=1

f(k)− p−1

2

Việc áp dụng tốn dễ cần thayf(x)bằng hàm hay giá trị Bài tốn áp dụng:

Bài toán 1: Cho p,q số nguyên dương nguyên tố nhau, chứng minh : p−1

X k=1 kq p

= (p−1)(q−1)

HD giải : Chọn f(k) = k , theo tốn mở đầu ta có p−1

X k=1 kq p =

p−1

X

k=1

f(k)q p

= q

p

p−1

X

k=1

f(k)− p−1

2

Mà p−1

X

k=1

f(k) = p(p−1)

2 nên dễ dàng có

p−1

X k=1 kq p

= (p−1)(q−1)

Bài toán :Cho p số nguyên tố lẻ, tính p−1

(111)

HD: Chọn f(k) = k3, đó

p−1

X

k=1

f(k) =

p(p−1)

2

Bài toán 4: Cho p số nguyên tố lẻ, tính

S =

p−1

X

k=1

"

(−1)k+1k3

p

#

Áp dụng giải phương trình S = HD : Chọn f(k) = (−1)k+1k3, đó

n

X

k=1

f(k) =(n+ 1)

2

(2n−1)

khi n chẵn

n

X

k=1

f(k) = −n

2(2n+ 3)

4

khi n lẻ

Ta nhận thấy trên, tất cảdegf(k)đều lẻ ( đáp ứng điều kiện đề Cịn degf(k) chẵn sao? Trường hợp ta xét ví dụ sau:

Bài tốn :Với plà số nguyên tố có dạng 4k+ 1, tính

S =

p−1

X i=1 i2 p

Nhận xét : Ta thử làm giống Chọnf(k) = k2 Khi dễ thấy f(k) không chia hết

cho p f(k) +f(p−k) = k2+ (p−k)2

= 2k2−2pk+p2 cũng không chia hết cho p Điều này

mâu thuẫn với điều kiện đề cho Vậy ta làm Giải :

Trước hết chứng minh bổ đề

Bổ đề : Với p số nguyên tố thỏa p ≡1(mod4) số tự nhiên a với 16 a 6 p−1

2

luôn tồn số tự nhiên b thỏa mãn p+

2 6b 6p−1 a

2+b2 ≡0( mod p).

Chứng minh : Theo định lí Wilson ta có : (p−1)!≡ −1( mod p) Với k = 1,p−1

2 p−k ≡ −k( mod p)⇒k(p−k)≡ −k

2( mod p).

Kết hợp với giả thiết p≡1( mod 4)⇒ p−1

2 ≡0( mod 2), ta được:

−1≡(p−1)! ≡(−1)p

−1

p−1 ! =

p−1

!

2

(modp)

Đặt ϕ=

p−1

!⇒ϕ2 ≡ −1(modp)

Với 6a 6 p−1

2 ta chọn

p+

2 6b p−1 thỏa mãn b

(112)

tại Khi

a2+b2 ≡a2(p2+ 1)≡0(modp)

Suy ĐPCM

Vậy bổ đề chứng minh Áp dụng với 16i6 p−1

2

p+

2 6j 6p−1 ta thấy :

S =

p−1

X i=1 i2 p =

p−1 X i=1 i2 p +

i2j2

p

=

p−1

X

i=1

i2

p − p−1

2 =

(p−1)(p−2)

Vậy

S = (p−1)(p−2)

Tiếp tục đề thi Chọn đội tuyển IMO Việt Nam năm 2005 (Vietnam TST 2005) Bài toán (Vietnam TST 2005) : Cho plà số nguyên tố (p > 3) Tính:

a)S =

p−1

X

k=1

2k2

p −2 k2 p

nếu p≡1(mod4)

b)P =

p−1 X k=1 k2 p

nếu p≡1(mod8) Giải:

Trước hết ta chứng minh bổ đề :

Bổ đề :Với p số nguyên tố thỏa p≡1(mod4)thì số tự nhiên a với 16a 6 p−1

2

luôn tồn số tự nhiên b thỏa mãn p+

2 6b 6p−1 a

2+b2 ≡0(modp).

Bổ đề : Với x số thực [2x]−2 [x]

2 {x} < 06x <

2

Chứng :

Bổ đề : Xem bổ đề Bổ đề : Ta có x= [x] +{x} Suy

[2x]−2 [x] = [2 [x] + 2{x}]−2 [[x] +{x}] = [2{x}]−2 [{x}] = [2{x}]

Khi đó: +)

2 6{x}<1 [2{x}] = =>[2x]−2 [x] =

+) 06x <

2 [2{x}] = 0⇒[2x]−2 [x] =

Vậy bổ đề chứng minh Quay trở lại toán :

a) Ta thấy S có tất p−1

2 số hạng

(113)

Theo bổ đề với số tự nhiên a với 16a 6 p−1

2 tồn số tự nhiên

b thỏa mãn

p+

2 6b 6p−1 a

2

+b2 ≡0(modp)

Suy a2+ (p−b)2 ≡0( mod p)

Do tồn a0 ∈

1;p−1

thỏa a2+a02 ≡0(modp) Gọi x,y số số dư chia k2 cho p 16k 6 p−1

2

có giá trị lớn nhỏ p−1

2 Theo nhận xét x=y, x+y=

p−1

2 ⇒x=y=

p−1

4

Từ S =x.1 +y.0 = p−41 Vậy S = p−1

4

b) Do p≡1( mod 8) nên tồn a cho a2 ≡2( mod p) p≡1( mod 4) Ta có :

P =

p−1 X k=1 k2 p =

p−1

X

k=1

2k2 p − k2 p −

p−1

X

k=1

2k2 p −2 k2 p =

p−1

X

k=1

2k2 p − k2 p −S

Ta cần tính

p−1

X

k=1

2k2

p − k2 p

Ta có :

p−1

X

k=1

2k2

p − k2 p =

p−1

X

k=1

2k2 p −

k2 p

p−1

X

k=1

2k2 p − k2 p =

p−1

X

k=1

k2

p −

p−1

X

k=1

2k2

p − k2 p

Theo nhận xét tập hợp số dư chia k2, 1

6k 6 p−1

2 cho p trùng với tập hợp

các số dư chia 2k2,16k6 p−1

2 cho p tức

p−1

X

k=1

2k2

p − k2 p = Suy

p−1

X

k=1

2k2 p − k2 p =

p−1 X k=1 k2 p = p

2−1

24

Vậy P = p

2−1

24 −

p−1

2 =

(p−1)(p−5)

24

(114)

bài tốn tính tổng dạng Sau toán tương tự : Bài toán 7: Cho p≡1( mod 3), m khơng chia hết cho p, tính

p−1

X

i=1

3mi2 p

−m

3i2 p

HD : Làm tương tự ta kết (m−1)(p−1)

2

Bài toán 8: ( THTT số 430) Cho p số nguyên tố có dạng 4k+ , tính tổng p−1

X

k=1

2k2

p −2 k2 p

HD: Giải tương tự câu a toán cho kết p−1

2

Bài toán 9: Cho plà số nguyên tố có dạng 4k+ , tính tổng p−1

X

k=1

2k3

p −2 k3 p

HD : Bài ta sử dụng kết toán mở đầu , cho kết p−1

2

Nhận xét : Bài toán cho ta nhận xét tốn tổng qt tính tổng S =

p−1

X

k=1

2kn

p −2 kn p

với n = n = S = p−1

2 , với n=1 làm trường

hợp n=3 ta có kết tương tự Vậy ∀n ∈Nthi kết có khơng? Đáp án đúng, phần chứng minh xin dành cho bạn đọc

Ví dụ 2: Chứng minh với p số nguyên tố thỏa p ≡ 1(mod4), với hai hàm số f(x)

g(x) thỏa mãn

(

(f(x), p)) = (g(x), p)) = ∀x

Khi chứng minh p−1

X

i=1

g(x)f(x)i2

p

−g(x)

f(x)i2

p

= (g(x)−1)(p−1)

Chứng minh: Bài dễ dàng chứng minh toán mở đầu theo cách Bài toán câu a

Áp dụng toán hay cần chọn hàm thỏa mãn sử dụng Ví dụ : Chọnf(x) = 1, g(x) = 2,

p−1

X

i=1

2i2

p −2 i2 p

= (2−1)(p−1)

2 =

p−1

Bài tập tương tự:

Bài tốn 10: Cho p số ngun tố có dạng 4k+ 1, chứng minh

2

p−1

X

i=1

(p+ 1)i2

p

−(p+ 1)

i2

p

(115)

Bài toán 11: Cho p số nguyên tố có dạng 4k+ 1, chứng minh p−1

X

i=1

(2p2+ 2p+ 1)i2 p

−(2p2+ 2p+ 1)

i2 p

không số nguyên tố với p

Bài toán 12: Cho p số nguyên tố có dạng 4k+ Tìm pđể

S =

p−1

X

i=1

(2p+ 3).i2

p

−(2p+ 3)

i2

p

−25

là số phương

Hàm phần nguyên việc tính tổng chữ số

Định nghĩa

Giả sử n số tự nhiên Ta định nghĩaS(n) tổng chữ số n biểu diển hệ thập phân

Tính chất

1 Với n số nguyên dương ta có

S(n) = n−9

∞ X k=1 h n 10k i

CM: Trong hệ thập phân ta biểu diễn

n=amam−1 a0 = 10mam+ 10m−1am−1 + +a1.10 +a0

Khi

n−9

∞ X k=1 h n 10k i

= amam−1 a0|10−9

amam−1 a1|10 + amam−1 a2|10 + am

Suy n−9

∞ P k=1 n 10k = m P i=0

ai(10i−9(10i−1+ 10i−2+ + 1) = m

P

i=1

ai =S(n) Từ có đpcm

2 S(n)≡n(mod9)

CM: Ta nhận thấy

∞ X k=1 h n 10k i

là số nguyên nên9

∞ X k=1 h n 10k i

(116)

3 06S(n)6n

CM : Dễ thấyS(n)>0, từ

S(n) = n−9

∞ X

k=1

h n

10k

i

Suy S(n)6n ( dấu “=” xảy 06n 69) S(m+n)6S(m) +S(n)

CM : Ta có :S(m) +S(n) =m+n−9

∞ X

k=1

h m

10k

i

+

∞ X

k=1

h n

10k

i !

Áp dụng BĐT [a] + [b]6[a+b] ta

S(m) +S(n)>m+n−9

∞ X

k=1

m+n

10k

!

=S(m+n) (đpcm)

Dấu xảy phép cộng m+n khơng có nhớ S(m.n)6S(m).S(n)

Bạn đọc chứng minh thơng qua biểu diễn m, nvà mn hệ thập phân Từ ta có tổng quát :

6 S(X

i=1

ai)6

X

i=1

S(ai)

7 S(Y

i=1

ai)6

Y

i=1

S(ai)

Bài tập ví dụ

Ví dụ 1: Tìm n cho

S(n) = n2−2014n+

Giải :

Ta có 06S(n)6n⇔06n2 −2014n+ 56n

Giải hệ bất phương trình, kết hợp n ∈N ta đượcn = 2014 Vậy n = 2014

Ví dụ 2: Tìm n cho: n+S(n) +S(S(n)) = 2001 Giải :

Ta có n 62000⇒S(n)6S(1999) = 28⇒S(S(n))610⇒n >1972 Mà

3n≡n+S(n) +S(S(n))≡2001 ≡3(mod9)⇒n ≡1(mod9)

(117)

Bằng cách thử trực tiếp ta thấy giá trị n cần tìm n ∈ {1969,1972,1975} Ví dụ 3: Tìm n thỏa n+S(S(n)) = 2014

Bài toán xin dành cho bạn đọc

Ví dụ : (PTNK 2008) Với số nguyên dương n, gọi S(n) tổng chữ số n a) Chứng minh số n = 999 n = 2999 biểu diễn dạng a+b với

S(a) =S(b)

b) Chứng minh số n,999 < n < 2999 biểu diễn dạng a +b với

S(a) =S(b)

Giải :

a) Giả sử biểu diễn

>n= 999 : Ta có :a+b= 999, mà phép cộng khơng có nhớ nên S(a) +S(b) =S(a+b) =

S(999) = 27

Mà S(a) =S(b) nên 2S(a) = 27 (vô lý) >n= 2999 :

Tương phép cộng a+b=2999 khơng có nhớ nên ta có ĐPCM

b) Trước hết ta chứng minh 999< n <2999 tồn số tự nhiên k cho

S(k) +S(n˘k) số chẵn Thật vậy, S(n) số chẵn ta chọn k = Nếu S(n) lẻ, giả sử n =ba2a1a0, đób ∈ {1; 2} Do 999 < n <2999 n 6= 1999 (do S(n)lẻ) nên tồn

tại i choai <9 Chọn i lớn thoả mãn điều kiện Khi chọn k = 10i.(ai+ 1)

S(k) = ai+ 1cịn S(n−k) = S(n)˘ai˘1 + 9(phép trừ có nhớ tạo số vị trí giảm đơn vị vị trí trước đó) Từ suy S(k) +S(n−k) = ai+ +S(n)˘ai˘1 + =S(n) +

chẵn S(n) lẻ Bây giả sử ta tìm k cho S(k) +S(n−k) số chẵn Khi đặt

k=a3a2a1a0 n−k=b3b2b1b0

Do S(k) +S(n−k)chẵn nên số số isao choai+bi lẻ chẵn Với cặp số(i, j)sao cho ai+bi = 2ji+ 1, aj +bj = 2ki+ lẻ, ta đổi → ai0 =ki+ 1, b0i =ki, a0j = kj, b0j =kj+ Với số isao cho a1+b1 = 2ki, ta đổi →a0i =b

0

i =ki Khi dễ dàng nhận thấy

a03a02a01a00 +b03b02b01b00 =a3a2a1a0+b3b2b1b0 =n

S a0

3a02a01a00

=S b0

3b02b01b00

Từ ta có điều phải chứng minh

Ví dụ 5: (IMO 1975) Đặt A=S(44444444) B =S(A) TìmS(B) Giải :

Đặt N = 44444444 DoN <100004444 nên N không 4444.4<20000 số Từ

(118)

Mặt khác

4444≡(−2)(mod9)⇒N ≡24444 = 81431.2≡(−2)(mod9)

Do đóS(B)≡7( mod 9) (2) Từ (1) (2) ta cóS(B) = Bài tập tương tự:

Ví dụ 6: Đặt A=S(30.42012), vàB =S(A) Tính S(B).

Ví dụ 7: (VMO 20004) Tìm giá trị nhỏ S(n)khi n chạy tập bội 2003 HD : Sử dụng bổ đề :

• 1001 số nguyên dương nhỏ số số nguyên dương mmà 10m ≡ 1(mod 2003)

• Khơng tồn bội dương 2003 có dạng 10k+ 1 với k∈ N

• Tồn bội dương của2003 có dạng 10k+ 10h+ 1, với k, h∈ N Và sử dụng thêm tính chất S(n)ta đáp án minS(n) = Bài tập tự luyện :

Bài toán :Cho số tự nhiên N thỏa S(N) = 100, S(5N) = 50 Chứng minh N chẵn Hướng dẫn: Đặt M = 5N S(M) = 50 S(2M) = S(10N) = S(N) = 100 Suy phép cộng M +M = 2M phép cộng khơng nhớ

Bài tốn 2: Tìm n nhỏ cho n số tự nhiên liên tiếp tùy ý chọn số N mà S(N) vdots 13

Đáp án : 79

Bài tốn 3: Đặt a=S((29)1999);b =S(a);c=S(b) Tìm c.

Đáp án : c=8

Bài toán 4: CMR với n số tự nhiên ta ln có

S(8n)

S(n) >

Bài tốn 5: Cho a số chẵn khơng chia hết cho Chứng minh

lim

n→+∞S(a

n) = +∞

(119)

Bài toán :Cho n số nguyên dương thỏa mãn n! có 2002 chữ số 0tận Chứng minh n68024

Bài toán :Với n số nguyên , chứng minh

n+

+

n+ 4

+

n−1

=n

Bài toán :( Đài Loan 1998) Chứng minh với số nguyên dương m n, ta ln có:

(m, n) =

m−1

X

k=0

kn m

+m+n−mn

Bài toán : Tìm min{xn|16n6M} với M = 20142014và

(xn) :

  

 

x1 =M

xn+1 =

x+[xnM]

∀n>1

Bài tốn :Tìm số ngun dương n lớn cho 2014! chia hết cho7n.

Bài toán 6: (Canada 1998) Tim số số a thỏa mãn

ha

2

i

+ha

i

+ha

i

=a

Bài tốn 7: ( Hàn Quốc 1997)Tính tổng n

P

k=1

h√

ki theo n vàa=√n Bài toán :Cho (m, n) = với m chẵn Tính tổng

S= 2n +

n−1

X

k=1

(−1)[kmn ]

km n

Bài toán :(Balkan 1998 ) Tính số hạng khác dãy

k2

1998

:k= 1,2, ,1997

Bài toán 10 : Chứng minh tích n số ngun liên tiếp ln chia hết cho n!

Bài toán 11 : (APMO 2001) Tìm số nguyên N lớn cho số số thuộc tập hợp

{1,2, , N}và chia hết cho số số thuộc tập chia hết cho Bài toán 12 : Chứng minh với số nguyên dương n>3

(2n−1)(2n−2)(2n˘4) .(2n−2n−1) n!

Bài toán 13 : Chứng minh

(120)

Bài toán 14 : Cho n>2 số nguyên dương Chứng minh rằng: n

X

k=2

n2

k

=

n2

X

k=n+1

n2

k

Bài toán 15 : Tính

lim

n−→+∞

1

n

n

X

k=1

2n k

−2hn

k

i !

Bài toán 16 : (Định lý LTE) Gọivp(n) số mũ p khai triển thừa số nguyên tố n Cho p số nguyên tố lẻ, x, y số nguyên cho x, y không chia hết cho p x – y chia hết cho p, n số nguyên dương Chứng minh

vp(xn−yn) =vp(x−y) +vp(n)

Tài liệu tham khảo

1 Phần nguyên- Bài tập ứng dụng : Hoàng Xuân Thanh

2 Vẻ đẹp phần nguyên từ tính chất : Nhóm học sinh chun tốn trường KHTN Hà Nội

3 Hàm phần nguyên ứng dụng: Luận văn thạc sĩ toán học Tạ Duy Phượng Số học qua định lý toán : Trần Nam Dũng

5 Các hàm số học ứng dụng : Luận văn thạc sĩ Toán học, Đỗ Cao Sơn

Ngày đăng: 09/02/2021, 03:25

Tài liệu cùng người dùng

Tài liệu liên quan